MSK Flashcards

1
Q

A 32-year-old footballer sustains an avulsion injury to the anterior superior iliac spine during training. Which of the following muscles is likely to be affected? [B1 Q12]

a. Sartorius
b. Gracilis
c. Iliopsoas
d. Rectus femoris
e. Semimembranosus

A

Sartorius

Sartorius has its origin at the anterior superior iliac spine and inserts into the pes anserinus. A sartorius muscle injury can therefore cause an avulsion fracture of the anterior superior iliac spine. Gracilis has its origin at the inferior pubic ramus, and rectus femoris has its origin at the anterior inferior iliac spine.

Other avulsion sites and tendons
Rectus femoris – AIIS
All hamstrings – Ischial tuberosity
Iliopsoas – Lesser trochanter
Gluteus medius and minimus – Greater trochanter
Transversus abdominis – Iliac crest

How well did you know this?
1
Not at all
2
3
4
5
Perfectly
2
Q

A 13-year-old footballer complains of pain in the right groin after a tackle. Radiograph of the pelvis shows an avulsion fracture of the lesser tuberosity. Which muscle is attached to the lesser tuberosity? [B5 Q25]

a. Iliopsoas
b. Rectus femoris
c. Sartorius
d. Biceps femoris
e. Vastus medialis

A

Iliopsoas muscle

This is attached to the lesser trochanter.

How well did you know this?
1
Not at all
2
3
4
5
Perfectly
3
Q

A 52-year-old woman presents to her GP with a longstanding history of lower back pain which has suddenly worsened in severity over the past few days. An urgent MRI scan of the lumbar spine shows a right paracentral disc protrusion at the L4/L5 level. The disc impinges on the lateral recess at this level. The most likely nerve to be affected is the: [B2 Q26]

a. Cauda equina
b. Lumbar plexus
c. Right L4
d. Right L5
e. Right S1

A

Right L5

The right L5 nerve root is the most likely to be affected as it will be sitting in the right lateral recess at the L4/5 level. The L4 nerve root will be at the exit foramen and therefore if the protrusion affects only the lateral recess, then this nerve will already have exited and therefore not be affected.

How well did you know this?
1
Not at all
2
3
4
5
Perfectly
4
Q

An MRI of the ankle shows deep injury to the deltoid ligament. Which of the following belong to the deep components of the deltoid (medial collateral) ligament of the ankle? [B2 Q22]

a. Tibio-calcaneal ligament
b. Tibio-navicular ligament
c. Posterior superficial tibiotalar ligament
d. Anterior tibiotalar ligament (ATTL)
e. Tibio-spring ligament

A

D

The other answers all belong to the superficial components. The posterior deep tibiotalar ligament is also deep. The superficial and deep components function almost synergistically and stabilise against valgus and pronation as well as rotational force against the talus.

How well did you know this?
1
Not at all
2
3
4
5
Perfectly
5
Q

A 56-year-old woman slips off the pavement onto the road and her outstretched foot is run over by a passing car. She has immediate severe midfoot pain. Plain radiographs taken on arrival at the emergency department confirm a Lisfranc fracture dislocation of the midfoot. Which two bones does the Lisfranc ligament attach to? [B2 Q36]

a. First metatarsal and intermediate cuneiform
b. First metatarsal and medial cuneiform
c. Second metatarsal and medial cuneiform
d. Second metatarsal and intermediate cuneiform
e. First and second metatarsals to the medial and intermediate cuneiforms

A

Second metatarsal and medial cuneiform

The Lisfranc ligament attaches between the second metatarsal and medial cuneiform, which is why an injury to this ligament allows the second to fifth metatarsals to drift laterally once they have lost this stabilisation. This is therefore an unstable injury and requires rapid immobilisation. This is a vital injury to detect as long-term sequelae will often result from a delayed diagnosis.

How well did you know this?
1
Not at all
2
3
4
5
Perfectly
6
Q

Of the lateral fibrous structures contributing to the stability of the posterolateral corner of the knee, which is the most likely to be congenitally absent and not identified on MRI, being present in only approximately two-thirds of patients? [B4 Q6]

a. Lateral collateral ligament
b. Popliteus tendon
c. Popliteo-fibular ligament
d. Arcuate ligament
e. Fabello-fibular ligament

A

Arcuate ligament

The structures of the posterolateral corner of the knee have a very important role in maintaining the rotational stability of the knee joint.

The lateral collateral ligament forms the superficial layer, with the remainder of the structures comprising the deep layer. Injury is relatively common and results most frequently from a varus force on an extended joint.

The lateral collateral ligament and popliteus tendon are present in all joints, with the popliteofibular ligament being present in approximately 98%.

Both the arcuate ligament and fabellofibular ligaments are variable, with the former absent more frequently. Absence of one of these structures is often compensated for by hypertrophy of the other.

How well did you know this?
1
Not at all
2
3
4
5
Perfectly
7
Q

Following wrist arthrography by a single-compartment radiocarpal injection technique, contrast seen on MR arthrographic images in the midcarpal compartment can be explained by disruption of which of the following structures? [B4 Q98]

a. triangular fibrocartilage
b. lunotriquetral ligament
c. dorsal distal radioulnar ligament
d. flexor retinaculum
e. radio scapholunate ligament

A

Lunotriquetral ligament

The two most important intercarpal ligaments are the scapholunate and lunotriquetral ligaments. These are crescent-shaped with strong anterior and posterior zones and a relatively thin middle membrane.

Disruption of Scapholunate or lunotriquetral ligaments will result in communication of the radiocarpal compartment proximally with the midcarpal compartment distally.

Contrast material seen in the distal radioulnar joint indicates disruption to the triangular fibrocartilage complex or distal radioulnar ligaments.

Some authors advocate selective midcarpal injection as superior in delineating injury to the scapholunate and lunotriquetral ligaments, and a sequential technique of three injections has also been described.

How well did you know this?
1
Not at all
2
3
4
5
Perfectly
8
Q

A 20-year-old football player presents after injuring his right knee in a tackle. Plain radiographs show fracture of the tibial spine with lipohaemarthrosis. What structure is attached to the medial part of the anterior tibial spine? [B5 Q36]

a. Anterior cruciate ligament
b. Posterior cruciate ligament
c. Medial collateral ligament
d. Lateral collateral ligament
e. Medial meniscus

A

Anterior cruciate ligament

The anterior cruciate ligament is attached to the medial part of the tibial spine.

How well did you know this?
1
Not at all
2
3
4
5
Perfectly
9
Q

A 60-year-old presents with left groin pain. Ultrasound shows a 2 cm hypoechoic lesion bulging medial to the epigastric vessels on Valsalva manoeuvre and absent on rest. What is the most likely diagnosis? [B5 Q38]

a. Direct inguinal hernia
b. Indirect inguinal hernia
c. Obturator hernia
d. Spigelian hernia
e. Femoral hernia

A

Direct inguinal hernia

A direct inguinal hernia is seen medial to the inferior epigastric vessels whereas an indirect hernia is seen lateral to them.

How well did you know this?
1
Not at all
2
3
4
5
Perfectly
10
Q

A 40-year-old man presents with right groin pain. Ultrasound shows a 3 cm echogenic soft tissue mass distending the right inguinal canal on straining, and which goes away on relaxation. What is the most likely diagnosis? [B5 Q39]

a. Direct inguinal hernia
b. Indirect inguinal hernia
c. Femoral hernia
d. Obturator hernia
e. Lymph node

A

Indirect inguinal hernia

An indirect inguinal hernia protrudes through the internal inguinal ring and extends along the inguinal canal parallel to its long axis.

How well did you know this?
1
Not at all
2
3
4
5
Perfectly
11
Q

An 18-year-old football player presents with right groin pain after a tackle. The radiograph shows avulsion of the lesser trochanter. Which muscle is attached to the lesser trochanter? [B5 Q42]

a. Sartorius
b. Rectus femoris
c. Iliopsoas
d. Hamstrings
e. Adductor longus

A

Iliopsoas

Iliopsoas is attached to the lesser trochanter.

How well did you know this?
1
Not at all
2
3
4
5
Perfectly
12
Q

An 18-year-old male with fingernail dysplasia and a family history of renal failure is investigated for possible nail-patella syndrome. Which of the following radiographic findings is considered pathognomonic for this disorder? [B1 Q54]

a. Patellar hypoplasia
b. Lateral elbow hypoplasia
c. Posterior iliac horns
d. Calcaneo-valgus feet
e. Madelung deformity

A

Posterior iliac horns

Nail-patella syndrome (Fong Disease) (hereditary onycho-osteo-dysplasia) is an autosomal dominant condition characterized by:

Nail dysplasia
Patella hypoplasia
Elbow hypoplasia
Iliac horns present in 80% and are pathognomic.
They arise at the site of gluteus medius and project posterolaterally. Patella hypoplasia results in chronic knee pain and recurrent dislocations. Elbow hypoplasia is typically towards the lateral side of the joint. Madelung deformity and calcaneo-valgus feet are other features described in nail-patella syndrome. The most important non-orthopaedic condition is an immune complex nephropathy, which can result in end-stage renal failure. These patients are also at risk of open-angle glaucoma.

Fong Disease (Nail Patella Syndrome) [STATdx]

Synonyms:

  1. Hereditary Osteo-onycho-dysplasia
  2. Iliac Horn Syndrome
  3. Turner-Kieser syndrome

Pathology: Autosomal Dominance

Clinical Features:

  1. Knee pain and instability
  2. Characteristic Nail Deformities – abnormal/ triangular lunula, discoloured, pitting
    ridged, thickened nails, starts from thumb to little finger, worse on ulna side.
  3. Kidneys – proteinuria to nephrotic syndrome.
  4. Open angle glaucoma – peripheral vision loss.

Radiographic Features:

  1. Iliac horns – Pathognomonic, 80% of cases
    Symmetric, posterior-laterally from central ilium,
    Asymptomatic
    Palpable 2. Absent/hypoplastic patella
    Asymmetric
    Superolateral dislocation
    Knees appear flattened
    Overgrowth of medial femoral condyle
    Hypoplastic lateral femoral condyle
    Enlarged tibial tubercle
  2. Absent/hypoplastic radial head
    Asymmetric
    Radial head may subluxate or dislocate
    Hypoplastic capitellum and lateral condyle
    Prominent medial condyle
    Creates positive ulnar variance deformity at wrist - Cubitus valgus
    Limited motion
  3. Others
    Hypoplastic changes of shoulder, hip
    Talipes equinovarus and other foot deformities*
How well did you know this?
1
Not at all
2
3
4
5
Perfectly
13
Q

A 30-year-old woman presents to her general practitioner with fatigue and painful stiff knees.
She is subsequently found to be anaemic. Plain films show an Erlenmeyer flask deformity of
the distal femora with cortical thinning. There are no erosions. What is the most likely
underlying condition? [B4 Q2]

a. mucopolysaccharidosis
b. rheumatoid arthritis
c. Gaucher’s disease
d. Langerhans’ cell histiocytosis
e. thalassaemia major

A

Gaucher’s disease

Gaucher’s disease is the most common lysosomal storage disorder with an incidence of 1:50
000 (100 times more common in Ashkenazi Jews). It is caused by a defect of hydrolase acid b-
glycosidase, which results in accumulation of the fatty substance glucosylceremide within
macrophages in the reticuloendothelial system. It characteristically causes an Erlenmeyer flask
deformity of the distal femur or proximal tibia due to marrow infiltration. Patients may be
asymptomatic or present with anaemia, large joint stiffness or bone pain. Diagnosis is by bone
marrow aspirate. The mucopolysaccharidoses are a spectrum of lysosomal storage diseases that
typically present in infancy with a variety of overt symptoms and signs. Rheumatoid arthritis
can present with anaemia and joint stiffness, but marrow infiltration is not a feature on plain
film. Musculoskeletal manifestations of Langerhans’ cell histiocytosis most commonly affect
the skull (50%). Although Erlenmeyer flask deformity is seen in thalassaemia major,
presentation is within the first 2 years of life.

How well did you know this?
1
Not at all
2
3
4
5
Perfectly
14
Q

A 10-year-old white boy presented with mass in abdomen and bilateral hip pain. He was found to have splenomegaly and pancytopaenia. Pelvic radiograph suggests bilateral avascular necrosis of femoral heads. MRI shows diffuse low signal bone marrow on T1 and T2. What is the most likely diagnosis? [B5 Q11]

(a) Gaucher’s disease
(b) Sickle cell disease
(c) Perthes’ disease
(d) Leukaemia
(e) Multifocal histiocytosis

A

Gaucher’s disease

This is the commonest lipid storage disorder. Marrow infiltration leads to avascular necrosis in the femur, ankle, and humerus. Patients have splenomegaly, anaemia, and pancytopenia. Loss of normal remodelling of the femur results in Erlenmeyer flask’ deformity at the distal femur.

How well did you know this?
1
Not at all
2
3
4
5
Perfectly
15
Q

A 10-year-old male involved in an RTA is brought to the A&E department with a history of
severe right thigh pain. Plain radiograph demonstrates a transverse fracture in the mid-
diaphysis of the femur. Incidental note is made of bone osteopenia and undertubulation of the
femur with metaphyseal flaring producing Erlenmeyer flask deformity and coxa magna related
to previous avascular necrosis of the femoral head. What is the underlying bone disease? [B1
Q55]

A. Pyle’s disease.
B. Osteopetrosis.
C. Gaucher’s disease.
D. Fibrous dysplasia.
E. Ollier’s disease.

A

Gaucher’s disease.

All these conditions cause Erlenmeyer flask deformity and are associated with pathological
fractures. However, the history of previous avascular necrosis of femoral head suggests
Gaucher’s disease. Sickle-cell disease (SCD) may also cause all the above bone changes.

Gaucher’s disease is a rare familial metabolic disorder caused by deficiency of the enzyme F-
glucocerebrosidase. This leads to accumulation of glucocerebroside in reticuloendothelial cells
(macrophages) of the liver, spleen, and bone marrow.

The imaging findings include delayed growth, osteopenia, Erlenmeyer flask deformity,
metaphyseal notching of humeri, bone infarction/avascular necrosis, and pathological fractures.
Diffuse marrow replacement with low signal on T1WI is noted on MRI. Visceral
manifestations include hepatosplenomegaly and reticulonodular interstitial lung disease.

Gaucher’ Disease [STATdx]

Pathology: – Autosomal Recessive, Glucocerebrosidase deficiency, deposition in RE cells

General Features:
Organomegaly and pancytopenia, relative sparing of hepatocytes, low incidence of liver failure.
Acute bony crisis secondary to infarct
Other skeletal manifestation Erlenmeyer flask deformity of distal femora, pathological
fractures, vertebral collapse, and bony infarct.

Radiographic Findings:
* Vertebral endplate collapse, femoral Erlenmeyer flask deformity, avascular necrosis
(AVN), pathologic fractures
* Hepatosplenomegaly
* MRI Findings
o Early hip AVN
o Reduced marrow signal due to glucocerebroside deposition
o “Ballooning” of intervertebral discs due to vertebral endplate collapse
* Nuclear Med – “Cold” lesion in bone infarcts
* Best imaging tool
o Plain radiographs to characterize disease extent
o MR for early bone infarct detection

How well did you know this?
1
Not at all
2
3
4
5
Perfectly
16
Q

A 34-year-old man is admitted with sudden onset chest pain described as tearing in nature.
Clinical examination reveals a diastolic murmur consistent with aortic regurgitation.
Subsequent chest CT confirms ascending aortic dissection. He has a past medical history of
spontaneous pneumothorax. Despite a negative family history, an underlying diagnosis of
Marfan syndrome is suspected. Which of the following musculoskeletal manifestations is
required for this diagnosis to be made? [B1 Q59]

A. Joint hypermobility.
B. Pectus excavatum of moderate severity.
C. Reduced upper-to-lower segment ratio.
D. High arched palate.
E. Malar hypoplasia.

A

Reduced upper-to-lower segment ratio.
Marfan syndrome
* An autosomal dominant, multisystem connective tissue disorder.
* Approximately 25% are sporadic mutations.
* Mutation of the fibrillin-1 gene.
* There is a broad phenotype expression – diagnosis can be made with the Ghent
classification system.
* In the absence of a family history, the presence of two major criteria in two different organ
systems and a minor criterion in a third system supports a diagnosis of Marfan syndrome.

In this case, dissection of the ascending aorta is a major cardiovascular criterion and
spontaneous pneumothorax a minor pulmonary system criterion. Of the musculoskeletal
manifestations, reduced upper-to-lower segment ratio is a major criterion, the remaining
options are all minor criteria. Other musculoskeletal system major criteria include scoliosis
with a curvature greater than 20’, pectus carinatum, pectus excavatum requiring surgery,
acetabular protrusion, and medial displacement of the medial malleolus causing pes planus.

How well did you know this?
1
Not at all
2
3
4
5
Perfectly
17
Q

A 30-year-old African man presents with knee pain. Radiograph shows a serpiginous area of
lucency with sclerotic margins in the proximal metaphysis of tibia. MRI shows a ‘double-line’
sign on T2-weighted images. There is a linear area of low signal peripheral to a high signal
intensity inner border. A bone scan shows no uptake in the area. The most likely diagnosis is?
[B5 Q27]

(a) Bone infarct
(b) Osteomyelitis
(c) Enchondroma
(d) Non-ossifying fibroma
(e) Osteonecrosis

A

Bone infarct

The radiographic and MRI appearances described are typical for a bone infarct. These are
typically metaphyseal or diaphyseal in contrast to osteonecrosis. Bone scans may show
increased uptake in acute stages where revascularisation has occurred.

How well did you know this?
1
Not at all
2
3
4
5
Perfectly
18
Q

A 5-year-old boy presents with a history of walking difficulty. On examination he is noted to
have an antalgic gait and lower limb length discrepancy, with the right limb being shorter than
the left. Plain radiographs of the right leg show lobular ossific masses arising from the distal
femoral epiphysis and the talus, which resemble osteochondromas. What is the most likely
underlying diagnosis? [B1 Q66]

A. Dysplasia epiphysealis hemimelica (Trevor disease).
B. Multiple epiphyseal dysplasia.
C. Diaphyseal aclasis.
D. Dyschondrosteosis (Leri–Weil disease).
E. Klippel–Trenaunay–Weber syndrome.

A

Dysplasia epiphysealis hemimelica (Trevor disease).

This is an uncommon developmental disorder relating to the formation of an osteochondroma
type lesion at the epiphyses of usually a single lower extremity. The epiphyses most involved
are those on either side of the knee or ankle. Typically, it is only the medial or lateral side of
the epiphyses affected (medial: lateral 2:1). The disease is usually recognized at a young age
because of an antalgic gait, palpable mass, varus or valgus deformity, or limb length
discrepancy.

Dysplasia epiphysealis hemimelica – Trevor disease [Radiopaedia]

Pathology: Non-hereditary, Rare, Male Predilection.

Clinical Features: Young Children, Walking difficulty, Antalgic Gait, Discrepancy in leg
lengths

Radiological Features:

Osteochondroma arising from epiphyses, so epiphyseal signal. Widening of Joint Space.

Lower > Upper,
Distal > Proximal,
Medial > Lateral

Dysplasia epiphysealis multiplex – Fairbank disease [Radiopaedia]

Autosomal dominance
Delayed, irregular secondary ossification centres,
Flatting and squaring of epiphyses, eg. Thinning or lateral tibial epiphysis
Double-layered patella
Hypoplastic tibial and femoral condyles with shallow intercondylar notch

Hereditary Multiple Exostosis – Diaphyseal Aclasis [Radiopaedia]

Autosomal dominance
Multiple osteochondroma – typical appearance of osteochondroma.
Can be asymptomatic small lesions or large lesion with deformity.
Any bone except calvarium.

How well did you know this?
1
Not at all
2
3
4
5
Perfectly
19
Q

An orthopaedic surgeon in your hospital comes to your office to ask your advice on a 15-year-
old girl he is about to see at his clinic. Although limited clinical information is available, he
was able to find out that the patient has a congenital condition, which has resulted in her being
confined to a wheelchair. As she was complaining of a sore knee, an x-ray was carried out.
There is a long gracile femur and tibia, indicating under-tubulation of the bone. What is the
most likely cause for this appearance? [B1 Q70]

A. Dwarfism.
B. Gaucher’s disease.
C. Cerebral palsy.
D. Arthrogryposis multiplex congenital.
E. Juvenile RA (JRA).

A

Cerebral palsy.

As a radiologist you would obviously have been able to correct your orthopaedic colleague,
that long gracile bones are examples of over tubulation, not under tubulation. As such options
A, B, and E are not diagnostic considerations as these result in under tubulation and may cause
an Erlenmeyer flask abnormality. This phenomenon is further described elsewhere in this
chapter but causes of Erlenmeyer flask abnormality include anaemias (thalassaemia, SCD),
storage disorders (Gaucher’s, Niemann–Pick), and skeletal dysplasias (Pyle’s disease,
craniometaphyseal dysplasia, Melnick–Needles syndrome).

The most common cause of over-tubulation is in patients with diminished weight bearing
(cerebral palsy, myelomeningocele, arthrogryposis), with cerebral palsy being the most
common of these. JRA and Marfan syndrome can also cause this appearance.

Bone Tubulation [Radiopaedia]

the adult-type appearance of long bones with a diaphyseal narrowing or constriction that forms
due to periosteal bone resorption and endosteal bone formation at the metaphysis during
longitudinal bone growth at the physis.
Causes of over-tubulation
1. Osteogenesis imperfecta
2. Neurofibromatosis
3. Paralysis
4. Radiation therapy

Causes of under-tubulation [Erlenmeyar flask deformity]
1. Gaucher
2. Thalassaemia
3. Cranio-metaphyseal Dysplasia
4. Hereditary multiple exostosis

How well did you know this?
1
Not at all
2
3
4
5
Perfectly
20
Q

A 21-year-old patient attends the A&E department following a minor injury with a suspected
fracture. The request form states that the patient has osteogenesis imperfecta. It is noted that
the patient is of reduced stature and does not display any evidence of blue sclera, but that the
colouration of his sclera has faded over time. He has normal hearing. What subtype of
osteogenesis imperfecta does he likely have? [B1 Q72]

A. Type I.
B. Type II.
C. Type III.
D. Type IV.
E. Type V.

Type IV.

Osteogenesis imperfecta in an adult is almost always type I or IV. Type I is the most common.
Patients have can have normal stature and the characteristic blue sclera are seen in 90%.
Patients also often have hearing impairment. Type IV has variable bone fragility, from mild to
severe. Hearing impairment is less common, as is reduced stature. Blue sclerae are present in
children but are often absent after adolescence. Type II is universally fatal in the neonatal
period. Type III is also severe and often associated with reduced lifespan. Stature is
significantly reduced. In patients who survive to adolescence the blue sclera are also often
absent. Type V is not universally recognized but is like type IV.

A

Type IV.

Osteogenesis imperfecta in an adult is almost always type I or IV. Type I is the most common.
Patients have can have normal stature and the characteristic blue sclera are seen in 90%.
Patients also often have hearing impairment. Type IV has variable bone fragility, from mild to
severe. Hearing impairment is less common, as is reduced stature. Blue sclerae are present in
children but are often absent after adolescence. Type II is universally fatal in the neonatal
period. Type III is also severe and often associated with reduced lifespan. Stature is
significantly reduced. In patients who survive to adolescence the blue sclera are also often
absent. Type V is not universally recognized but is like type IV.

How well did you know this?
1
Not at all
2
3
4
5
Perfectly
21
Q

You are reviewing the x-rays of a child that are stored in your department’s museum.
Sequential radiographs have been taken as the child has aged and the appearances have become
more pronounced with time. The child has a form of dwarfism. On the CXR you notice ‘oar-
shaped’ ribs. The metacarpals are short and wide, but narrow proximally, giving a fan-like
appearance. The patient has a J-shaped sella turcica. The iliac wings are wide, but the iliac
bones narrow inferiorly. On the lateral lumbar spine, the vertebrae have central anterior beaks.
A clinical vignette mentions that the patient was not intellectually impaired. What condition
does the patient probably have? [B1 Q74]

A. Campomelic dysplasia.
B. Niemann–Pick disease.
C. Morquio syndrome.
D. Achondroplasia.
E. Hurler’s syndrome

A

Morquio Syndrome

The constellation of skeletal manifestations describes the characteristic appearance of dyostosis
multiplex. This pattern of skeletal abnormalities is seen with the mucopolysaccharidoses
(MPS), although it can also be seen with other storage disorders. Except for Hurler syndrome,
where the manifestations are present at 1 year of age, the skeletal manifestations progress as
the patients get older. Hurler’s and Morquio’s are the most common of the MPS conditions.
Amongst the MPS conditions, Morquio’s stands out as a favourite for single best answer (SBA)
and viva questions as it is the only MPS where the patient is not intellectually impaired. It also
displays a central anterior vertebral body beak, whereas the other conditions have an anterior
beak in the lower third of the vertebral body.

How well did you know this?
1
Not at all
2
3
4
5
Perfectly
22
Q

A 35-year-old man presents with increasing stiffness in his knee and soft tissue swelling around
the joint. Plain films show multiple areas of irregular cyst-like radiolucency in the distal femur.
There are no areas of abnormal calcification and there is no evidence of periarticular
osteoporosis. MR shows a low signal joint effusion on both T1 and T2 sequences. The most
likely diagnosis is: [B2 Q2]

a. Synovial osteochondromatosis
b. Pigmented villo-nodular synovitis
c. Osteoarthritis
d. Reiter’s syndrome
e. Osteomyelitis

A

Pigmented villo-nodular synovitis

Pigmented villo-nodular synovitis is a relatively rare condition which usually presents in the third or fourth decade. It is a monoarticular, painful disease which causes a decreased range of movement at the affected joint. It is most common at the knee (80%) followed by the hip, ankle, shoulder, and elbow. Haemorrhagic ‘chocolate’ effusion is characteristic.

Low signal effusion on all sequences at MR is characteristic. There is no calcification or osteoporosis, and joint space narrowing is a late feature.

Pigmented villonodular synovitis (PVNS) [STATdx]

  • Synonyms:
    o Teno-synovial Giant Cell Tumour, Intra-articular
    o Benign synovioma; focal nodular synovitis
  • Definitions
    o Low-grade fibro-histiocytic proliferation with hemosiderin deposits in synovial
    joints
  • 2 subtypes
    o Diffuse tenosynovial giant cell tumor (TSGCT)
    o Localized TSGCT
  • Location - Synovial joints
    o Knee: 80% of cases (infrapatellar fat pad > suprapatellar recess > posterior
    intercondylar notch > adjacent to posterior cruciate ligament)
    o Hip, ankle, elbow, shoulder, wrists, fingers, toes
  • Morphology
    o Diffuse TSGCT: widespread distribution corresponding to shape of joint and
    associated synovial spaces
    o Localized TSGCT: singular round, ovoid, or lobulated mass in synovial joint
  • Radiographic Findings
    o No mineralization
    o Effusion/joint distension
    o Cartilage preserved until late in process
  • CT Findings
    o May have increased attenuation related to hemosiderin deposition
    o Well-defined erosion with sclerotic margins
    o Synovium enhances on C+ imaging
  • MR Findings
    o Effusion
    o T1: predominantly low to intermediate SI (hemosiderin deposition)
    o Internal signal that shows fat is uncommon finding (from lipid-laden
    macrophages)
    o T2: predominantly low to intermediate SI; (hemosiderin deposition)
    o Gradient-echo: assess for blooming (hemosiderin deposition)
    o C+: expected to avidly enhance inhomogeneously, but enhancement is not
    present in all cases
  • Ultrasonographic Findings
    o Utility for intraarticular mass localization for biopsy
    o Confirms solid soft tissue lesion
    o Hypoechoic or mixed echogenicity ± hypervascularity
  • PET
    o May demonstrate increased FDG avidity
How well did you know this?
1
Not at all
2
3
4
5
Perfectly
23
Q

A 40-year-old man presents with right knee pain. Plain radiography shows a large joint effusion.
MRI of the knee shows multiple foci of low signal intensity seen in the synovium on T1, T2
and gradient-echo sequences. There is a moderate joint effusion. The most likely diagnosis is?
[B5 Q12]

(a) Haemangioma
(b) Pigmented villo-nodular synovitis
(c) Rheumatoid arthritis
(d) Synovial sarcoma
(e) Synovial chondromatosis

A

Pigmented villonodular synovitis

This is a benign pathology affecting usually the knee joint. It shows no calcifications,
osteoporosis or erosions (until late). MRI is diagnostic, the lesions returning low signal on all
sequences due to iron (haemosiderin)

How well did you know this?
1
Not at all
2
3
4
5
Perfectly
24
Q

A young man with limited range of movement at the shoulder joint, a webbed neck and plain
film findings of a hypoplastic scapula which is elevated and medially rotated with an associated
omovertebral bone is likely to have which associated syndrome? [B2 Q26]

a. Turner’s syndrome
b. Down’s syndrome
c. Klippel–Feil syndrome
d. Neurofibromatosis
e. Cleidocranial dysostosis

A

Klippel–Feil syndrome

The collective findings described are of a Sprengel deformity of the shoulder. This occurs
because of failure of descent of the scapula secondary leading to both cosmetic and functional
impairment. The male: female ratio is 3:1 and it is associated with Klippel–Feil syndrome, a
condition in which there is fusion of vertebral bodies, and renal anomalies.

How well did you know this?
1
Not at all
2
3
4
5
Perfectly
25
Q

A 17-year-old patient complains of lower thoracic back pain. Plain radiographs of the thoraco-
lumbar spine show wedging of multiple vertebrae at the thoraco-lumbar junction, multiple
limbus vertebrae, an increase in the AP diameter with a reduction in the sagittal height of
multiple vertebrae, and multiple endplate defects. What is the unifying diagnosis? [B2 Q31]

a. Scheuermann’s disease
b. Ankylosing spondylitis
c. Mycobacterium tuberculosis
d. Hyperparathyroidism
e. DISH

A

Scheuermann’s disease

These are the classical appearances of Scheuermann’s disease. This condition usually presents at puberty and consists of:
Vertebral wedging
Endplate irregularity
Narrowing of the intervertebral disc spaces

The most common location is in the lower thoracic and upper lumbar spine. Schmorl’s nodes are often present.

How well did you know this?
1
Not at all
2
3
4
5
Perfectly
26
Q

A young man presents to his GP complaining of longstanding back pain. He says he has been
diagnosed with a ‘syndrome’ in the past but cannot remember the details. Which of the
following signs is more likely to suggest a diagnosis of homocystinuria than Marfan’s
syndrome? [B2 Q35]

a. Arachnodactyly
b. Osteoporosis
c. Scoliosis of the spine
d. Autosomal dominant inheritance
e. Upward lens dislocation

A

Osteoporosis

Osteoporosis is a feature of homocystinuria and occurs in 75% of cases, often causing bowing
and fracture of the long bones. The other features are all more suggestive of Marfan’s syndrome.
Although arachnodactyly does occur in homocystinuria (in 30% of cases), it occurs in 100%
of people with Marfan’s syndrome. Homocystinuria has an autosomal recessive mode of
inheritance

How well did you know this?
1
Not at all
2
3
4
5
Perfectly
27
Q

A dental radiograph of a 47-year-old woman shows loss of the lamina dura of most of the teeth.
Which of the following would be a possible cause? [B2 Q45]

a. Osteopetrosis
b. Hypoparathyroidism
c. Scleroderma
d. Sickle cell anaemia
e. Myeloma

A

Scleroderma

The other causes of loss of the lamina dura include Cushing’s disease, Paget’s,
hyperparathyroidism, osteoporosis, osteomalacia, leukaemia, metastases and Langerhans’ cell
histiocytosis. Both osteopetrosis and hypoparathyroidism cause thickening of the lamina dura
of the teeth.

How well did you know this?
1
Not at all
2
3
4
5
Perfectly
28
Q

A 70-year-old woman with a history of dysphagia presents with multiple swelling in the hands.
Radiographs of the hands show widespread soft tissue calcification with terminal phalangeal
resorption. What is the most likely diagnosis? [B5 Q47]

(a) Systemic lupus erythematosus
(b) Scleroderma
(c) Dermatomyositis
(d) Psoriasis
(e) Calcium pyrophosphate deposition disease

A

Scleroderma

Scleroderma is the cutaneous manifestation of progressive systemic sclerosis. This causes
fibrosis and small vessel disease in several organs. In the hands, typically it causes terminal
phalangeal resorption due to pressure atrophy, soft tissue calcification and occasionally intra-
articular calcification.

How well did you know this?
1
Not at all
2
3
4
5
Perfectly
29
Q

A three-month-old boy presents with several small painful soft-tissue swellings which have
developed over the mandibular region and the right clavicle. Plain films show marked
periosteal new bone formation and localised soft tissue swelling. There is also bone expansion
with remodelling of old cortex. The most likely diagnosis is: [B2 Q54]

a. Caffey disease
b. Hypervitaminosis A
c. Infantile myofibromatosis
d. Scurvy
e. Kinky hair syndrome

A

Caffey disease

The most likely diagnosis is Caffey disease. This is a relatively rare self-limiting condition
which usually presents before six months of age. The mandible is the most common site and
accounts for 80% of cases, followed by the clavicle and the upper limb bones.

How well did you know this?
1
Not at all
2
3
4
5
Perfectly
30
Q

A three-year-old boy attends A&E with a history of a seizure. He has known congenital
cardiomyopathy. A chest radiograph shows sclerosis and expansion of several ribs. Previous
plain films have shown bone islands within the vertebrae and long bones and bone cysts within
the phalanges. Which of the following conditions would be likely to underly these findings?
[B2 Q59]

a. Down’s syndrome
b. Tuberous sclerosis
c. Sturge–Weber syndrome
d. Neurofibromatosis
e. Sarcoidosis

A

Tuberous sclerosis

This is a multi-system autosomal dominant disorder affecting the central nervous system, kidneys, lung and heart. The classic triad of facial angiofibroma, epileptic seizures and mental retardation is only seen in approximately 30% of patients.

Skeletal abnormalities in TS include:
sclerotic calvarial patches or ‘bone islands’
Thickening of diploe
Expansion and sclerosis of ribs
Periosteal thickening of long bones.

Gracile ribs are often seen in association with Down’s syndrome.

How well did you know this?
1
Not at all
2
3
4
5
Perfectly
31
Q

A 40-year-old man with short stature and normal intelligence has features including a large
head, and an injury to his left leg whilst playing football. On review of the plain film, no acute
bony injury is demonstrated. However, he is noted to have a disproportionately large fibula, a
champagne glass pelvic inlet and horizontal acetabulae. Which is the underlying diagnosis?
[B3 Q14]

A. Achondrogenesis
B. Homozygous achondroplasia
C. Heterozygous achondroplasia
D. Pseudoachondroplasia
E. Metatrophic dwarfism

A

Heterozygous achondroplasia

Patients with pseudoachondroplasia have a normal skull and medial beaking of the proximal
femoral neck. Achondrogenesis is lethal in utero/neonatal period, as is homozygous
achondroplasia. In metatrophic dwarfism, dumbbell shaped long bones and flattened vertebra
may be found

How well did you know this?
1
Not at all
2
3
4
5
Perfectly
32
Q

A 41-year-old male presents to the A&E department with knee pain following a fall at work.
Plain radiography does not demonstrate any fracture, but note is made of continuous, irregular
cortical hyperostosis along the lateral margin of the femur. What is the most likely diagnosis?
[B1 Q68]

A. Osteopoikilosis.
B. Fibrous dysplasia.
C. Engelmann disease.
D. Melorheostosis.
E. Osteopathia striata.

A

Melorheostosis.

The radiographic findings describe the ‘flowing candle-wax’ sign (continuous, irregular cortical hyperostosis in a long bone), which indicates melorheostosis, a non-hereditary sclerosing bone dysplasia of unknown aetiology. Patients are often asymptomatic, being discovered incidentally. It is most common in the long bones. The disease can overlap with other sclerosing bone dysplasias such as:
Osteopoikilosis (multiple ovoid bone islands)
Osteopathia striata (metaphyseal longitudinal striations).

Engelmann disease presents in childhood with neuromuscular dystrophy. Diaphyseal fusiform enlargement with cortical thickening is seen in the long bones. Fibrous dysplasia causes bone thinning.

How well did you know this?
1
Not at all
2
3
4
5
Perfectly
33
Q

Which of the following skeletal findings on plain radiographs is not typically associated with
achondroplasia? [B4 Q21]
a. short interpedicular distance
b. small foramen magnum
c. rhizomelia
d. horizontal acetabular roof
e. atlantoaxial instability

A

Atlanto-axial instability

Achondroplasia is the most commonly seen autosomal dominant rhizomelic dwarfism.
Rhizomelia refers to relative shortening of the proximal compared with the distal portion of
the limbs. Achondroplasia has widespread skeletal manifestations affecting the skull, chest,
spine, pelvis and extremities. Intelligence and motor function are normal. The most significant
complication is brain-stem or spinal cord compression due to spinal stenosis, which is caused
by alignment abnormalities and decreased spinal canal size due to short pedicles with a reduced
interpedicular distance. Atlantoaxial instability is defined as a predental space of 3 mm or more
in adults and 5 mm or more in children, or where there is considerable change between flexion
and extension. It is seen in inflammatory arthritides, in Down’s and Morquio’s syndromes, and
with retropharyngeal abscess in a child.

How well did you know this?
1
Not at all
2
3
4
5
Perfectly
34
Q

At which of the following skeletal locations does avascular osteonecrosis typically only occur
in the presence of an associated fracture? [B4 Q51]

a. medial tibial condyle
b. second metatarsal head
c. lunate
d. femoral head
e. proximal scaphoid pole

A

Proximal scaphoid pole

Osteonecrosis may be caused by two mechanisms: interruption of arterial supply, and intra- or
extra-osseous venous insufficiency. Interruption of vascular supply is usually associated with
a fracture, as seen in the proximal scaphoid following waist fractures. Femoral head
osteonecrosis can occur with subcapital fractures, or without fracture as in Legg–Calve´–
Perthes disease. Other common locations that may develop osteonecrosis without overt fracture
include the medial tibial condyle (Blount’s disease), metatarsal head (Freiberg’s infraction)
and the lunate (Kienbock’s disease). Radiographic findings often lag several months behind the injury or onset of symptoms, and MR is the most sensitive imaging modality. Radiographic
signs include focal radiolucencies, sclerosis, bone collapse and loss of joint space

How well did you know this?
1
Not at all
2
3
4
5
Perfectly
35
Q

Which of the following is not a recognized cause of myeloid hyperplasia (red marrow
reactivation/reconversion) in a 50-year-old adult? [B4 Q68]

a. sickle cell disease
b. smoking
c. chemotherapy
d. long-distance running
e. Gaucher’s disease

A

Gaucher’s disease

Marrow reconversion is the repopulation of yellow (fatty) marrow with haematopoietic cells, reconverting the fatty marrow to red marrow. This occurs when the haematopoietic capacity of the existing red marrow in an adult is insufficient. This can result from increased physiological requirement (long distance running), chronic anaemia (sickle cell disease) and chemotherapeutic treatment with granulocyte–macrophage colony-stimulating factor. The pattern of reconversion is predictable and it is the reverse of the age-related physiological conversion of red to yellow marrow. Reconversion begins in the axial skeleton and progresses distally through the appendicular skeleton, to end in the hands and feet. Knowledge and recognition of this pattern are important because neoplastic infiltration of adult yellow marrow in malignant disease may have similar MR appearances. However, malignant marrow replacement tends to have a more random distribution than reconversion; it may enhance with intravenous gadolinium, can cause cortical destruction and may extend into the soft tissues.

How well did you know this?
1
Not at all
2
3
4
5
Perfectly
36
Q

You are asked to image the pelvis with MRI for someone who has a hip arthroplasty. Which
of the following measures can be used to decrease the magnetic susceptibility artifact from the
joint prosthesis? [B1 Q33]

A. Use fast spin echo (FSE) imaging rather than GE imaging.
B. Choose a higher field strength magnet, e.g. 3 T rather than 1.5 T.
C. Position the long axis of the prosthesis perpendicular to the main magnetic field strength
(β0) direction if possible.
D. If fat saturation is to be employed, use spectral fat suppression rather than STIR imaging.
E. Increase the volume of the voxels (decrease spatial resolution).

A

Use fast spin echo (FSE) imaging rather than GE imaging.

Magnetization of the implant affects the local field gradient, proton dephasing, and spin
frequency, resulting in signal void, spatial distortion, and spurious high signal. The lack of a
180° rephasing pulse in GE sequences, as opposed to SE/FSE sequences, means that T2*
effects are not reversed, and a greater dephasing of spins occurs in GE than in SE/FSE
techniques. Thus, GE techniques have a greater sensitivity to magnetic susceptibility effects.
This is detrimental when imaging patients with metal prostheses, but can be used to advantage
in certain clinical situations, e.g. when identifying subtle haemorrhage due to the magnetic
susceptibility effects of iron in haemosiderin (a blood breakdown product)

How well did you know this?
1
Not at all
2
3
4
5
Perfectly
37
Q

A radiologist is reporting a 99m Tc bone scan and describes it as a ‘super-scan’. He can say this
because of reduced uptake in the: [B1 Q73]

A. brain
B. skeleton
C. kidneys
D. bowel
E. myocardium.

A

Kidneys

A super-scan refers to a99m Tc-labelled technetium IBS where there is diffuse increased osseous uptake with apparent reduced renal and soft tissue uptake. The appearance is commonly due to widespread osteoblastic bony metastases (e.g. prostate or breast carcinoma), but is also caused by non-malignant disease (e.g. renal osteodystrophy, hyperparathyroidism, osteomalacia, myelofibrosis, Paget’s disease). In metastatic disease there is usually higher uptake in the axial than the appendicular skeleton.

In IBS uptake is normally seen in bone, kidneys, and bladder, soft tissues (low levels), breasts (particularly in young women), and epiphyses (skeletally immature patients).

Uptake is seen in the myocardium (high), brain (high), and bowel (moderate) in FDG-PET scanning, not IBS; however myocardial uptake on IBS can be seen in cases of recent myocardial infarction and amyloidosis.

Note that poor renal function can often demonstrate reduced or absent renal visualization producing an appearance like a superscan (false positive), whereas urinary tract obstruction in prostatic carcinoma can increase renal activity and lead to false negative scans.

How well did you know this?
1
Not at all
2
3
4
5
Perfectly
38
Q

A 56-year-old woman who has had chronic wrist pain since a fall several months previously is
referred for an MR arthrogram of her wrist with a suspected triangular fibrocartilage complex
(TFCC) tear. Which of the following would be the best sequence for visualising a TFCC tear?
[B2 Q56]

a. T1 axial
b. T2 coronal
c. Gradient echo sagittal
d. T2 sagittal
e. T1 sagittal

A

T2 coronal

The best sequence would be a T2 or T2* image for detecting a tear. This is also a useful plane
in which to assess for ulnar variance; positive ulnar variance has an association with
perforations. The central portion of the articular disc is not well vascularised and therefore a
tear in this portion will heal poorly. The peripheral portion, however, has been vascularised.

How well did you know this?
1
Not at all
2
3
4
5
Perfectly
39
Q

You are reporting an MRI knee on a patient with moderately severe osteoarthritis (OA), as
diagnosed on plain film radiography. The patient describes significant knee pain. Which of the
following statements best describes the relationship between symptoms, plain film findings,
and MRI findings? [B1 Q27]

A. The MRI findings correlate well with the severity of findings on plain film radiography.
B. MRI findings correlate well with the patient’s symptoms.
C. Plain film findings correlate well with the patient’s symptoms, unlike MRI.
D. Plain film and MRI both correlates well with the severity of the patient’s symptoms.
E. Symptoms, plain film findings, and MRI findings do not have a significant association with
each other.

A

The MRI findings correlate well with the severity of findings on plain film radiography.

MRI has been shown to correlate well with the severity of OA as depicted on plain film
radiography. Neither MRI nor plain film appearances are significantly associated with the
patient’s symptoms.

How well did you know this?
1
Not at all
2
3
4
5
Perfectly
40
Q

You are reporting an MRI knee on a patient with moderately severe osteoarthritis (OA), as
diagnosed on plain film radiography. The patient describes significant knee pain. Which of the
following statements best describes the relationship between symptoms, plain film findings,
and MRI findings? [B1 Q27]

A. The MRI findings correlate well with the severity of findings on plain film radiography.
B. MRI findings correlate well with the patient’s symptoms.
C. Plain film findings correlate well with the patient’s symptoms, unlike MRI.
D. Plain film and MRI both correlates well with the severity of the patient’s symptoms.
E. Symptoms, plain film findings, and MRI findings do not have a significant association with
each other.

A

The MRI findings correlate well with the severity of findings on plain film radiography.

MRI has been shown to correlate well with the severity of OA as depicted on plain film
radiography. Neither MRI nor plain film appearances are significantly associated with the
patient’s symptoms.

How well did you know this?
1
Not at all
2
3
4
5
Perfectly
41
Q

You are discussing OA with a rheumatologist. He/she is curious to know what radiological
findings seen in early disease are associated with progressive, as opposed to stable, arthritis.
All of these are associated with OA, but which is least likely to indicate progressive disease?

A. Increased uptake on isotope bone scan.
B. Grade 1 osteophytosis in the knee.
C. Osteochondral defect.
D. A focal area of high signal on T2WI and STIR in the subchondral bone.
E. A serpiginous subchondral line that is low signal on T2WI and T1WI, with an adjacent
T2WI high signal line.

A

Grade 1 osteophytosis in the knee.

While the Kellegren and Lawrence grading of OA is the most widely used scale for grading
OA on plain films, early osteophyte formation is not definitively ‘arthritic’ change. Studies
have shown that patients with this type of early change infrequently progress to developing
more severe disease. Recent research in OA is focused on the impact of the subchondral bone
on the disease, rather than hyaline cartilage. This is evidenced by increased uptake on bone
scan being closely linked to progressive OA, even in patients with relatively normal joints on
plain film. The foci of high signal on T2WI and STIR are bone marrow lesions that if persistent
can indicate pathology in the subchondral bone, which can lead to arthritic change. The
serpiginous line describes the classical finding of avascular necrosis. Both this and
osteochondral defects lead to progressive OA change.

How well did you know this?
1
Not at all
2
3
4
5
Perfectly
42
Q

A 55-year-old female presents to the rheumatologists with a history of episodic swollen red
joints over the previous 2 years. She also complains of left hip pain. The patient’s rheumatoid
factor is not known at the time of requesting the radiographs. There is no other past medical
history. The rheumatologists have requested bilateral hand and pelvis x-rays. The hand x-rays show bilateral asymmetric disease affecting the distal and proximal interphalangeal (IP) joints.
In the affected distal IP joints there are central erosions, adjacent sclerosis, and marginal
osteophytes. The first carpometacarpal joint in the left hand shows loss of joint space with
osteophyte formation. The scaphoid-trapezium joint in the right hand also shows loss of joint
space and adjacent sclerosis. In the left hip there is nonuniform loss of joint space, with
associated subchondral cyst formation. What is the main differential? [B1 Q37]

A. RA.
B. Psoriatic arthritis.
C. Erosive OA.
D. Calcium pyrophosphate deposition (CPPD) arthropathy.
E. Ankylosing spondylitis

A

Erosive OA.

If the presence of erosions is ignored the pattern of disease is typical of OA. Erosive, or
inflammatory, OA shows typical OA distribution, but with central erosions in the affected joint
spaces in the distal IP, and less commonly proximal IP, joints. These central erosions in
combination with marginal osteophytes give the classical gull-wing appearance to affected
joints. Whilst individual joint appearance can be identical to psoriatic arthropathy, which can
precede the development of the skin disease, the overall pattern is atypical, making it less likely.
Also, while the distribution of CPPD arthropathy is often identical to OA, it would not cause
erosions, and would usually be associated with chondrocalcinosis.

How well did you know this?
1
Not at all
2
3
4
5
Perfectly
43
Q

An 80-year-old woman complains of pain in both hands. Radiography of the hands shows
bilateral central articular ‘seagull’ erosions affecting the interphalangeal joints of fingers in
both hands. Mild periarticular osteoporosis is seen. The most likely diagnosis is? [B5 Q18]

(a) Osteoarthritis
(b) Erosive osteoarthritis
(c) Calcium pyrophosphate deposition disease
(d) Gouty arthropathy
(e) Rheumatoid arthritis

A

Erosive osteoarthritis

Central articular erosion with a ‘seagull’ pattern, ankylosis and periarticular osteoporosis is
typical of erosive osteoarthritis. This is seen in older women and is usually limited to hands,
particularly affecting the proximal interphalangeal joints.

How well did you know this?
1
Not at all
2
3
4
5
Perfectly
44
Q

A patient presents to their GP with a complex history of acute episodes of severe tender
inflamed joints, around the knee. At present the patient has joint stiffness which is most
pronounced in the evenings and mild joint pain. The patient has a past medical history of
hypothyroidism. A plain film is requested which shows chondrocalcinosis and moderate
degenerative change in the lateral tibiofemoral compartment and the patellofemoral
compartment. Regarding CPPD disease, which of the following statements is the most
appropriate? [B1 Q47]

A. The presence of chondrocalcinosis indicates a radiological diagnosis of pseudogout.
B. Pseudogout syndrome is the most common means of presentation for this disease.
C. Disproportionate involvement of the patellofemoral joint is the most frequently seen
radiographic finding.
D. The presence of crystals displaying positive birefringence at polarized light microscopy
allows for the definitive diagnosis of pyrophosphate arthropathy.
E. The presence of hypothyroidism is associated with the diagnosis.

A

The presence of hypothyroidism is associated with the diagnosis.

Disorders associated with CPPD are the four Hs:
Hyperparathyroidism
Haemochromatosis
Hypothyroidism
Hypomagnesaemia.

Use of nomenclature in this disorder is confused.

Pseudogout is the clinical presentation of an acutely inflamed joint due to calcium pyrophosphate crystal deposition, and as such is not a radiological diagnosis. Pseudogout syndrome is the dominant feature in only 10–20% of cases of CPPD. Another 10–20% of cases are asymptomatic, whilst most present with symptoms identical to OA, with occasional flares.

Disproportionate involvement of the patellofemoral joint is a characteristic feature but is only occasionally seen. Pyrophosphate arthropathy is a description of the pattern of disease present due to crystal deposition, and as such is not a diagnosis made by analysis of joint aspirate.

How well did you know this?
1
Not at all
2
3
4
5
Perfectly
45
Q

A 55-year-old man presents with pain and swelling in the left big toe. The plain radiograph
shows periarticular erosions with sclerotic margins and overhanging edges in the first
metatarsophalangeal joint. The joint space is preserved and there is moderate surrounding soft
tissue swelling. The most likely diagnosis is? [B5 Q13]

(a) Rheumatoid arthritis
(b) Erosive osteoarthritis
(c) Gouty arthritis
(d) Psoriatic arthropathy
(e) Calcium pyrophosphate deposition disease.

A

Gouty arthritis

Periarticular erosions
sclerotic borders
overhanging margins
preserved articular surface

Rheumatoid arthropathy has non-proliferative marginal erosions, symmetrical distribution and joint space narrowing. Psoriasis show progressive joint destruction with erosions. Erosive osteoarthritis is symmetrical with erosions are in the centre of the articular surface.

Calcium pyrophosphate deposition disease is polyarticular, and shows chondrocalcinosis and joint-space narrowing.

How well did you know this?
1
Not at all
2
3
4
5
Perfectly
46
Q

A 28-year-old male presents with soft tissue swelling, pain, and reduction of motion in the
small joints of his hands. Plain films of the hands show erosions at the (MCP) joints and distal
interphalangeal joints with periosteal reaction and enthesophytes. What is the most likely
diagnosis? [B1 Q2]

A. Psoriatic arthropathy.
B. RA.
C. SLE.
D. Haemochromatosis.
E. Calcium pyrophosphate dihydrate crystal deposition disease.

A

Psoriatic arthropathy.

Bone involvement before skin changes is evident in up to 20% of cases. Nail pitting or
discolouration is common and correlated with the severity of the arthropathy. Five distinct
manifestations have been described: oligoarthritis, polyarthritis (predominately distal IP joints),
symmetric type (resembling RA), arthritis mutilans, and spondyloarthropathy. The
characteristic distribution involves the small joints of the hands and feet, with or without
spondyloarthropathy. Involvement in the hands tends to include distal IP as well as MCP or
PIP joints, with early tuft resorption and distal IP erosive disease. The erosions become so
severe that a ‘pencil-in-cup’ deformity and telescoping of the joint may occur. Bone density
can be normal, and the joint distribution is asymmetric. Similarly, sacroiliitis is asymmetric,
unlike ankylosing spondylitis and syndesmophytes, which are non-marginal and asymmetric;
in ankylosing spondylitis they are marginal and asymmetric. The spondyloarthropathy of
psoriatic arthropathy is indistinguishable from that of reactive arthritis, the clinical scenario
(rash vs uveitis/urethritis) providing the diagnosis.

The arthropathies of CPPD disease and haemochromatosis are essentially identical
radiographically. Chondrocalcinosis is commonly seen in the wrist (triangular fibrocartilage)
and knee (menisci). The joints most affected are the knee, wrist and second and third MCPs of
the hand: the IP joints tend to be spared. Early disease shows erosive change. More advanced
disease demonstrates sclerosis, osteochondral fragments, and osteophytes. Subchondral cysts
are common and large. RA is rarely found in the distal IP joints and periosteal reaction is not
a feature. SLE is usually non-erosive and affects the MCP joints.

How well did you know this?
1
Not at all
2
3
4
5
Perfectly
47
Q

An 18-year-old male patient presents to the rheumatologists with a history of proximal right
tibial pain and sternal pain. The patient has a history of psoriasis and is also being seen by the
dermatologists with palmoplantar pustulosis. Plain films of the sternum indicate sclerosis of
the manubrium and erosive disease in the sternoclavicular joint. Plain films of the tibia show a
lucent lesion in the proximal tibial metaphysis with associated periosteal reaction. An MRI
shows high signal on STIR in the proximal tibial metaphysis, with a cortical defect. This area
enhances on the T1 post gadolinium images, as does the periosteal region. A bone biopsy of
the region is negative except for inflammatory cells. What is the most likely diagnosis? [B1
Q22]

A. Psoriatic arthropathy.
B. Synovitis, acne, pustulosis, hyperostosis, and osteitis (SAPHO) syndrome.
C. Chronic recurrent multifocal osteomyelitis (CRMO)
D. Chronic osteomyelitis.
E. Aseptic necrosis.

A

SAPHO syndrome.

The most common presentation for this condition is acne and palmoplantar pustulosis with
changes in the sterno-costoclavicular region where hyperostosis is seen, often with some
erosive change. The condition can also cause several manifestations in the spine, ranging from
focal hyperostosis of one or more lumbar vertebrae, to a syndrome strikingly like psoriatic
spondylo-arthropathy. In the appendicular skeleton hyperostosis is commonly seen. SAPHO
can also give rise to manifestations identical to osteomyelitis, but with no causative organism.
In this respect there is overlap with CRMO syndrome. Patients are frequently human leucocyte
antigen (HLA) B27 positive, and psoriasis frequently coexists, leading some authors to suggest
that SAPHO is a variant of psoriatic arthropathy, although this is not widely accepted.

How well did you know this?
1
Not at all
2
3
4
5
Perfectly
48
Q

A 30-year-old man visits his general practitioner complaining of recent onset of acne and
discharging pustules on his palms. He has a history of several years of pain and swelling at the
medial end of his right clavicle. Radiographs of the shoulder demonstrate hyperostosis and
early ankylosis of the sternoclavicular joint. What is the most likely diagnosis? [B4 Q90]

a. SAPHO syndrome
b. suppurative osteomyelitis
c. psoriatic arthritis
d. Reiter’s disease
e. recurrent multifocal osteomyelitis

A

SAPHO syndrome

Synovitis, acne, pustulosis, hyperostosis and osteitis. (SAPHO) syndrome is a term encompassing several disease entities that demonstrate an association between rheumatological and cutaneous lesions. There may be a delay of several years between the onset of osseous symptoms and cutaneous manifestations. It is thought to be similar to chronic recurrent multifocal osteomyelitis in children. The dominant radiographic abnormality is new and bizarre bone proliferation, with the sternoclavicular joint affected in 70–90% of cases.

How well did you know this?
1
Not at all
2
3
4
5
Perfectly
49
Q

A 44-year-old female patient presents to the rheumatologists with a history of multiple painful
joints for 2 years. She has synovitis clinically, confirmed on ultrasound, which involves the
MCP joints bilaterally. PA and Norgaard views of the hands are requested and show small
erosions in the distal radio-ulnar joint and the piso-triquetral joint, but no erosions at the MCP
joints. There is widening of the scapholunate interval on the right side. There is ankylosis of
the capitate to the hamate on the left. There is periarticular osteoporosis. Which of these
features is atypical of RA?

A. Symmetrical disease.
B. Synovitis on ultrasound but no erosions radiographically.
C. Erosions noted in the radio-ulnar joint and radio-carpal joint preceding MCP erosions.
D. Bony ankylosis of the carpal bones.
E. Periarticular osteoporosis.

A

Bony ankylosis of the carpal bones.

Whilst fibrous ankylosis of the carpal and tarsal bones does occur, bony ankylosis is extremely
rare in RA. It is, however, common in JRA. There are a number of unusual findings, which if
present should indicate a diagnosis other than RA. Productive bone change (e.g. periostitis or
enthesopathy) is extremely unusual. Osteophytes are also uncommon in the absence of
advanced associated osteoarthritic change. The exception to this is the distal ulna, a feature
known as ulnar capping. The other features are all typical of RA.

How well did you know this?
1
Not at all
2
3
4
5
Perfectly
50
Q

You are carrying out an MRI on a patient with a known history of RA. The patient has minimal
erosions on plain film, but severe arthralgia. She is being considered for biologic therapy. The
clinicians have requested an MRI of her hands. This reveals symmetrical disease in both hands
with areas of high signal on T2WI and low signal on T1WI around the triangular fibrocartilage
complex (TFCC), the radio-carpal joint (RCJ), and the distal radio-ulnar joint (DRUJ). The
abnormal areas at the TFCC and RCJ enhance following administration of gadolinium, the
DRUJ does not. A delayed T1WI sequence displays uniform enhancement in all joints. What
do these findings indicate? [B1 Q12]

A. Hyper-vascular pannus at the TFCC and RUJ, with fibrous pannus at DRUJ.
B. Fibrous pannus at the TFCC and RUJ, with joint effusion at DRUJ.
C. Hyper-vascular pannus at the TFCC and RUJ, with joint effusion at DRUJ.
D. Fibrous pannus at the TFCC and RUJ showing differential enhancement.
E. Fibrous pannus at the TFCC and RUJ with hyper-vascular pannus at DRUJ.

A

Hyper-vascular pannus at the TFCC and RUJ, with joint effusion at DRUJ.

Hyper-vascular pannus is intermediate to high signal on T2WI and low signal on T1WI. It also
enhances, retaining enhancement on delayed imaging. Joint effusions can be difficult to
differentiate from hyper-vascular pannus on pre-contrast imaging. Following enhancement,
they show only delayed enhancement. Fibrous pannus is low signal on all sequences.

How well did you know this?
1
Not at all
2
3
4
5
Perfectly
51
Q

A patient who is HIV positive presents with knee and ankle pain and swelling. Clinical
examination is otherwise unremarkable. Initial radiographs reveal only a joint effusion. The
complaint resolves after 4 weeks. What is the most likely diagnosis? [B1 Q34]

A. Septic arthritis.
B. Psoriatric arthritis.
C. HIV-associated arthritis.
D. Acute symmetric polyarthritis.
E. Hypertrophic pulmonary osteoarthropathy (HPOA).

A

HIV-associated arthritis.

This is oligoarticular, asymmetric and peripheral. It primarily affects the knees and ankles. It
has a short duration of 1–6 weeks; radiography may reveal a joint effusion. Acute symmetric
polyarthritis also occurs in HIV. It behaves clinically like RA, but patients are negative for
rheumatoid factor. Features that help differentiate it from RA are periostitis and proliferative
new bone formation. Occasionally an erosive variety with little or no proliferative bone
formation occurs. Psoriatric arthritis has a higher prevalence among AIDS patients than in the
general population. HPOA is associated with P carinii pneumonia (PCP) in AIDS. Plain films
reveal periosteal reaction. Kaposi’s sarcoma uncommonly affects the bone but does so most commonly in Africa. Non-
Hodgkin lymphoma (NHL) is the second most common tumour in HIV infection. It can
produce lytic, sclerotic, or mixed lesions with a wide zone of transition; they are usually lytic.
Other musculoskeletal complications in AIDS include infections (cellulitis, osteomyelitis,
septic arthritis, pyomyositis, necrotising fasciitis), Reiter’s syndrome, undifferentiated
spondyloarthropathy, polymyositis, osteonecrosis (especially of the femoral head),
osteoporosis, rhabdomyolysis, and anaemia.

How well did you know this?
1
Not at all
2
3
4
5
Perfectly
52
Q

A 30-year-old man presents with backache and morning stiffness. Examination reveals loss of
spinal movement, uveitis, and upper zone end inspiratory fine crepitations on auscultation.
Which of the following statements is most correct in relation to the radiological features of the
underlying condition? [B1 Q39]

A. Romanus lesions (anterior or posterior spondylitis) are a late feature.
B. Syndesmophytes are better depicted on MRI than plain film.
C. Ankylosis involves the vertebral edges or centre.
D. Sacroiliac joint widening is not a feature.
E. Enthesitis appears as low signal within the ligaments on STIR imaging.

A

Ankylosis involves the vertebral edges or centre.

The question refers to ankylosing spondylitis. Ankylosis involves the vertebral edges or centre,
with bony extension through the disc. The former is thought to be secondary to a Romanus
lesion, the latter an Andersson lesion. Romanus lesions are irregularities and erosions involving
the anterior and posterior edges of the vertebral endplates and are the earliest changes of
spondylitis depicted on conventional radiographs. On MRI an Andersson lesion is depicted as
disc-related signal-intensity abnormalities of one or both vertebral halves of a discovertebral
unit. They are often hemispherically shaped. MRI is better than conventional radiography at
depicting Romanus lesions, Andersson lesions (spondylodiscitis), and most other abnormalities,
although ankylosis is equally well detected by both modalities.

Syndesmophtyes are difficult to detect on MRI. Plain radiography is superior in this respect
because of its superior spatial resolution; syndesmophtyes are seen as bony outgrowths of the
anterior vertebral edges. They occur in 15% of the vertebrae of patients. Apical pulmonary
fibrosis affects 1% of patients. Sacroiliac (SI) joint erosion and widening is an early feature,
and this may initially be more prominent on the iliac side of the joint, as the cartilage on that
side is normally thinner. Later in the disease, sclerosis and ankylosis occur and the SI joints
become symmetrically fused.

Enthesitis is most prominently seen when the interspinal ligaments, those that extend between
the spinous processes, and the supraspinal ligaments are affected. Ligamentous involvement is
characterized by an increased signal intensity on either STIR images or contrast-enhanced
T1WI fat saturated sequences. It may be associated with osteitis of adjacent bone marrow in
the spinous processes. Arthritis of the synovial joints (e.g. facet joints) and insufficiency
fractures (often spontaneous or after minor trauma) are also features of the seronegative
spondylarthritides.

How well did you know this?
1
Not at all
2
3
4
5
Perfectly
53
Q

A 24-year-old male patient is referred from the rheumatologists with a history of back pain and
hip pain. Plain films are carried out. These show bilateral sacroiliitis with erosive change on the iliac side on the left, but sacral and iliac erosions on the right. The imaging of the spine
reveals large non-marginal syndesmophytes in the thoracolumbar spine with a relatively
normal lower lumbar spine. The patient also complains of foot pain and plain films reveal
evidence of a retrocalcaneal bursitis with erosion of the calcaneus. Hand x-rays reveal small
erosions asymmetrically in the distal IP joints in both hands. What is the most likely diagnosis?
[B1 Q52]

A. Ankylosing spondylitis.
B. Reactive arthritis.
C. Psoriatic arthritis.
D. Erosive OA.
E. Adult Stills disease.

A

Psoriatic arthritis.

Ankylosing spondylitis causes a symmetrical sacroiliitis. The syndesmophytes associated with
this are marginal and fine. It also typically progresses superiorly from the lumbar spine. Both
reactive arthritis and psoriatic arthritis cause an asymmetric sacroiliitis and the
syndesmophytes are usually centred on the thoracolumbar spine and are non-marginal and
bulky. Retrocalcaneal bursitis and erosions, whilst more common in reactive arthritis, can
occur in psoriatic arthritis, and reactive arthritis would uncommonly affect the hands. Also,
with all other factors being equal, psoriatic arthritis is much more common than reactive
arthritis, even without the skin manifestations, which are absent in up to 20% at presentation.

How well did you know this?
1
Not at all
2
3
4
5
Perfectly
54
Q

In a 21-year-old man with symptoms of chronic back pain, pain in his feet, particularly the
great toe and metatarsophalangeal joints, and bilateral sacroiliitis on plain films, the most likely
diagnosis is: [B2 Q7]

a. Ankylosing spondylitis
b. Gout
c. Inflammatory bowel disease-related arthropathy
d. Reiter’s syndrome
e. Psoriatic arthritis

A

Reiter’s syndrome

Reiter’s syndrome is the association of urethritis, conjunctivitis and mucocutaneous lesions.
Sacroiliitis is usually bilateral but often persists asymmetrically. There is an association with
the HLA B27 antigen. Reiter’s has a predilection for the great toe and metatarsophalangeal
joints.

How well did you know this?
1
Not at all
2
3
4
5
Perfectly
55
Q

A routine pre-operative chest X-ray in a 62-year-old woman shows bilateral erosion of the
distal clavicles. Which one of the following conditions might be responsible? [B2 Q62]

a. Hypoparathyroidism
b. Rheumatoid arthritis
c. Langerhans’ cell histiocytosis
d. Ankylosing spondylitis
e. Sarcoidosis

A

Rheumatoid arthritis

Myeloma, hyperparathyroidism, metastases, cleidocranial dysplasia and Gorlin basal cell
nevus syndrome all cause absence of the outer end of the clavicle. Destruction of the medial
end of the clavicle is caused by metastases, infection, lymphoma, eosinophilic granuloma,
rheumatoid arthritis, and sarcoma.

How well did you know this?
1
Not at all
2
3
4
5
Perfectly
56
Q

A 15-year-old boy presents with pain and swelling in the hands. Radiographs show periarticular
osteopenia, loss of joint space at the metacarpophalangeal joints and widened bases in the
proximal phalanges. A periosteal reaction is seen in the metacarpal bones. What is the most
likely diagnosis? [B5 Q44]

(a) Juvenile rheumatoid arthritis
(b) Psoriatic arthropathy
(c) Scleroderma
(d) Systemic lupus erythematosus
(e) Dermatomyositis

A

Juvenile rheumatoid arthritis

The condition is usually seen in young people before the age of 16 years. In the hand, the
metacarpophalangeal and interphalangeal joints are usually affected. Chronic synovitis causes
enlargement of bones and epiphyses. Malalignment and subluxation are common. An
important feature of juvenile arthritis is periosteal reaction affecting the metacarpal and
phalangeal shafts.

How well did you know this?
1
Not at all
2
3
4
5
Perfectly
57
Q

A 57-year-old man with increasing pain and stiffness in his hands and feet and worsening back
pain presents to his GP. Plain films of his hands show sclerosis of the terminal phalanges and
several ‘pencil-in-cup’ erosions. There is destruction of the interphalangeal joint of his right
great toe with exuberant periosteal reaction. There is also erosion of the posterior margin of
the calcaneus. The most likely diagnosis is: [B2 Q40]

a. Reiter’s syndrome
b. Ankylosing spondylitis
c. Rheumatoid arthritis
d. Psoriatic arthritis
e. Osteoarthritis

A

Psoriatic arthritis

This is usually HLA B27 positive and is associated with skin and nail changes in most cases.
The hands are often described as having sausage digits, and erosions with ill-defined margins
are characteristic. Sacroiliitis is often present and most often bilateral. Within the axial skeleton,
there is often large bulky vertically orientated soft-tissue ossification giving a ‘floating’
osteophyte appearance.

How well did you know this?
1
Not at all
2
3
4
5
Perfectly
58
Q

Which of the following favours a diagnosis of rheumatoid arthritis rather than tuberculous
arthritis? [B3 Q32]

A. Periarticular osteopenia
B. Marginal erosion
C. Relatively late sparing of joint space
D. Joint effusion
E. Uneven and thick synovial proliferation

A

E

The remaining answers are also seen in TB arthritis. Even and thin synovium, large bone
erosions, rim enhancement around bone erosion and extra-articular cystic masses are more
frequently seen in TB arthritis

How well did you know this?
1
Not at all
2
3
4
5
Perfectly
59
Q

Ivory phalanx, tuft resorption, and pencil-in-cup deformity of the middle phalynx in a patient
with asymmetric arthritis are most likely to represent: [B3 Q49]

A. Psoriatic arthritis
B. Haemachromatosis
C. Rheumatoid arthritis
D. Reiter syndrome
E. Scleroderma

A

Psoriatic arthritis

Hands are more commonly affected in psoriatic arthropathy, whereas feet are more commonly
affected in Reiter’s

How well did you know this?
1
Not at all
2
3
4
5
Perfectly
60
Q

In a 50-year-old woman with arthralgia, which of the following favours rheumatoid arthritis
over Systemic Lupus Erythematosus (SLE)? [B3 Q48]

A. Prominent subluxation of metacarpal phalangeal joints (MCPJ)
B. Usually bilateral and symmetrical
C. Absence of erosion
D. Radiographically similar to Jaccoud’s arthropathy
E. Hyperextension of Distal Interphalyngeal (DIP) and flexion of Proximal Interphalyngeal
(PIP) joints

A

Hyperextension of DIP J and flexion of PIP J

A-D are features seen in SLE.

How well did you know this?
1
Not at all
2
3
4
5
Perfectly
61
Q

On plain radiographs of the hands in a middle-aged male patient complaining of bilateral joint
pain and swelling, which single feature is most likely to support a diagnosis of psoriatic arthritis
over rheumatoid arthritis? [B4 Q15]

a. new bone formation
b. joint space loss
c. periarticular osteoporosis
d. periarticular erosions
e. soft tissue swelling

A

New bone formation is the hallmark finding of psoriatic arthritis and is not seen in rheumatoid
arthritis. Conversely, periarticular osteoporosis is seen in rheumatoid but is not a feature of
psoriatic arthritis. Both conditions may cause soft tissue swelling (typically a sausage digit in
psoriatic arthritis), joint space loss and erosions, which are marginal in psoriatic and marginal
and/or central in rheumatoid arthritis. Another distinguishing factor is the distribution of
involved joints in the hands, which is typically, but not always, interphalangeal in psoriatic and
metacarpophalangeal in rheumatoid arthritis.

How well did you know this?
1
Not at all
2
3
4
5
Perfectly
62
Q

Which of the following features is not a recognized primary musculoskeletal manifestation of
the CREST (calcinosis, Raynaud’s phenomenon, oesophageal involvement, sclerodactyly and
telangiectasia) syndrome? [B4 Q66]

a. digital oedema
b. calcinosis
c. acro-osteolysis
d. osteoporosis
e. joint erosions

A

Osteoporosis

CREST syndrome represents the limited form of the autoimmune connective tissue disorder
scleroderma. Five-year survival rate is 50–67%. The two other types of sclerodermas are
generalized (also called systemic sclerosis) and localized (morphoea). Common findings are
digital soft-tissue oedema with sclerodactyly (tapered soft tissues), acro-osteolysis
(autoamputation) and calcinosis. There may be associated arthritis that shows erosions (also
seen in the ribs) or terminal phalanx resorption, with joint space narrowing a late sign.
Osteoporosis is not usually a feature except in the context of disuse.

How well did you know this?
1
Not at all
2
3
4
5
Perfectly
63
Q

A young man complains of early morning back pain and stiffness, and undergoes plain
radiographs followed by MRI of the whole spine. Which single feature is most likely to suggest
a diagnosis of psoriatic arthritis over ankylosing spondylitis? [B4 Q32]

a. syndesmophytes
b. para-syndesmophytes
c. asymmetrical sacroiliitis
d. ankylosis
e. patchy bone marrow oedema

A

Para-syndesmophytes

Seronegative spondyloarthritis is an umbrella term for inflammatory joint or spinal
conditions that are not associated with rheumatoid factor or rheumatoid nodules. There are five
described subgroups: ankylosing spondylitis, psoriatic arthritis, arthritis associated with
inflammatory bowel disease, reactive arthritis (e.g. Reiter’s syndrome) and an undifferentiated
subgroup.

The subgroups may overlap both clinically and radiologically, and the diagnosis is more easily
made based on clinical history and examination. Imaging plays a limited role in differentiation,
particularly early in the disease when there can be considerable overlap of appearances.
The main exception is the identification of para-syndesmophytes, which are seen almost
exclusively in psoriatic arthropathy. In addition, bone marrow oedema can involve the entire
vertebral body in psoriatic arthritis, which may be a further useful distinguishing feature.
Undifferentiated spondyloarthritis is diagnosed when there is no clinical or radiological
evidence of sacroiliitis. All types may eventually progress to ankylosis.

How well did you know this?
1
Not at all
2
3
4
5
Perfectly
64
Q

An elderly female patient has plain radiographs performed in an outpatient clinic for bilateral
painful, stiff hips, which demonstrate joint space narrowing. Which additional feature is more
likely to support a diagnosis of rheumatoid arthritis rather than osteoarthritis? [B4 Q58]

a. eccentric joint space loss
b. soft tissue swelling
c. subchondral sclerosis
d. subchondral cysts
e. protrusio acetabuli

A

Protrusio acetabuli

Even or eccentric joint space reduction representing cartilage loss is seen in both types of
arthritis and not a distinguishing diagnostic feature. Although osteoarthritis is said to be
classically eccentric, this is difficult to assess accurately on many hip radiographs, as they are
not routinely taken in the upright, weight-bearing position. Subchondral sclerosis and cysts are
typically associated with degenerative osteoarthritis. Although soft-tissue swelling is a feature
of rheumatoid arthritis, the depth of the hip joint and the copious surrounding soft tissues mean
that any synovial swelling is unlikely to be appreciated clinically or radiologically. Subtle
osteophytes (in osteoarthritis) or erosive change/osteoporosis (in rheumatoid arthritis) can
distinguish between the two entities. An inflammatory cause should be considered in young
adults with hip pain and, if protrusio or other abnormalities are found, the sacroiliac joints
should be examined

How well did you know this?
1
Not at all
2
3
4
5
Perfectly
65
Q

A 57-year-old female patient with a history of multiple myeloma is referred for imaging due
to a history of arthralgia primarily affecting the hands. The patient describes early morning
stiffness that eases through the day. The clinicians report a finding of synovitis clinically. Blood
results have revealed a raised ESR. Hand x-rays are carried out which reveal sharply defined
intra-articular marginal erosions at the MCP joints of the index and middle fingers bilaterally.
The joint spaces are well preserved. There are also well marginated subchondral cysts noted in
the carpal bones, again with joint space preservation. Soft tissue nodules are noted around the
wrist joints, which are not calcified. There is no evidence of juxta-articular osteopenia. No
osteophytes are noted. What diagnosis is most strongly suggested by these findings? [B1 Q64]

A. Gout.
B. CPPD.
C. RA.
D. Amyloidosis.
E. Wilson’s disease.

A

Amyloidosis.

There are a lot of conditions that are capable of mimicking RA. In these cases, a few key
features can help reach a diagnosis. The classic finding in gout is of non-marginal erosions, as
opposed to those described. Nevertheless, marginal erosions can occur with gout. An RA-type
picture in the presence of non-marginal erosions or calcified soft-tissue nodules (tophi) should
suggest this diagnosis. CPPD gives a more productive pattern of arthritis, such as seen with
OA, affecting the radio-carpal joint. Thus, it is often suspected when the appearance is of OA
with a ‘funny distribution’. Amyloidosis is suggested first by the history of MM. Involvement
of the hands is more commonly seen in amyloid secondary to prolonged dialysis but can be
seen when the amyloid is secondary to MM, when the wrists are often affected. Amyloid can
closely resemble RA in its distribution and the pattern of erosions. However, three important
features can help differentiate: amyloidosis classically preserves the joint space, is not usually
associated with periarticular osteopenia, and amyloidosis causes well-demarcated subchondral
cyst formation in excess to that expected from the degree of joint disease.

How well did you know this?
1
Not at all
2
3
4
5
Perfectly
66
Q

A patient under joint care of cardiology and nephrology has bulky soft tissue nodules, well-
marginated erosion, preserved joint spaces involving the wrists, elbows, shoulder and hip joints.
Which is the most likely diagnosis? [B3 Q50]

A. Multicentric reticulohistiocytosis
B. Amyloidosis
C. Ochranosis
D. Wilson’s disease
E. Haemachromatosis

A

B.

Multiple sites can be involved and there is characteristic preservation of joint spaces

How well did you know this?
1
Not at all
2
3
4
5
Perfectly
67
Q

Which is the most characteristic feature in haemochromatosis? [B3 Q19]
A. Chondrocalcinosis
B. Small subchondral cysts with a fine rim of sclerosis
C. Symmetric joint space narrowing
D. Generalised osteopenia
E. Hook-like osteophytes on the radial aspects of the metacarpal heads

A

Hook like osteophytes on the radial aspects of the metacarpal heads

Although all the answers are features, hook-like osteopohytes are the most characteristic.
Arthropathy is seen in 50%, and chrondrocalinosis in more than 60%, with knees being most
commonly affected.

How well did you know this?
1
Not at all
2
3
4
5
Perfectly
68
Q

A 70-year-old man has plain radiographs of the hands and knees for joint pain and swelling,
which show joint space narrowing and chondrocalcinosis. Which additional finding would
support a diagnosis of haemochromatosis over pseudogout? [B4 Q73]

a. periarticular calcium deposition
b. metacarpal hooked osteophytes
c. eccentric joint space narrowing
d. large subchondral cysts
e. intra-articular loose bodies

A

Metacarpal hooked osteophytes

Haemochromatosis is excess iron deposition in the tissues and can be either primary (autosomal
recessive genetic disorder) or secondary to ineffective erythropoiesis or iron overload. Skeletal
manifestations include osteoporosis (which is proportional to the extent of iron deposition),
small subchondral cysts, arthropathy (50%), concentric joint space narrowing,
chondrocalcinosis, and characteristic hooked osteophytes on the radial aspect of the metacarpal
heads. Other organs can be affected by iron deposition, most commonly the brain, liver,
pancreas and spleen

How well did you know this?
1
Not at all
2
3
4
5
Perfectly
69
Q

In imaging of focal bone lesions in the appendicular skeleton, which of the following
radiographic features is most likely to indicate an aggressive or malignant process? [B4 Q17]

a. cortical expansion
b. lytic process
c. periosteal reaction
d. multiple lesions
e. wide zone of transition

A

Wide zone of transition

The zone of transition relates to the interface between the tumour margin and the host bone. It
is an extremely important discriminator, particularly for lytic lesions. Lesions with a well-
defined margin (and therefore narrow zone of transition) are described as geographic and are
usually non-aggressive, whereas those with a wide zone of transition are termed ‘permeative’
and are often malignant or aggressive (such as in osteomyelitis). Cortical expansion without
destruction is seen in many benign or slow-growing conditions such as fibrous cortical defect
and aneurysmal bone cyst. Many bone lesions, both benign and aggressive, are lytic. Periosteal
reaction does not indicate an aggressive lesion as such, but the pattern of reaction can do so.
Multiplicity is not an indicator of malignancy, as it can be seen in benign and self-limiting
processes (such as multiple enchondromatosis and neurofibromatosis). Equally, a solitary
lesion may be malignant

How well did you know this?
1
Not at all
2
3
4
5
Perfectly
70
Q

Of the following types of periosteal reaction, select the one most likely to indicate a benign
process? [B4 Q24]

a. soap-bubble
b. sunray
c. hair-on-end
d. laminated
e. Codman’s triangle

A

Soap-bubble

Periosteal reactions are usually a radiographic manifestation of underlying bone disease. The
term ‘soap bubble’ refers to expansion of the cortex without destruction by a lytic bone lesion.
The intact cortex usually indicates a benign process, whereas cortical destruction is associated
with malignant or aggressive lesions. Sunray and hair-on-end reactions are spiculated forms of
periosteal reaction that occur following periosteal elevation by tumour, with tumour preventing
the subperiosteal space from filling with new bone. Laminated or ‘onion-skin’ reaction occurs
with both malignant and benign processes and indicates an intermittent or cyclical process.
Codman’s triangles are formed by elevation and then destruction of the periosteum. They are
usually related to malignant tumours but can also be formed by aggressive benign processes.

How well did you know this?
1
Not at all
2
3
4
5
Perfectly
71
Q

The presence of punctate, ring-like or arcuate calcification in a lytic bone lesion on plain
radiography is most commonly associated with which of the following matrix types? [B4 Q19]

a. osteoblastic
b. fibrous
c. cartilaginous
d. cellular
e. mixed

A

Cartilaginous

Chondroid tumour matrix may or may not calcify, but, if it does, the pattern is characteristically
in arcs or circles and is sometimes described as ‘popcorn’. Osteoid matrix when calcified is
usually dense and homogeneous like a cloud. Calcified fibrous matrix has a characteristic
ground-glass appearance, whereas a cellular tumour usually does not show matrix calcification.
A mixed matrix will show mixed characteristics.

How well did you know this?
1
Not at all
2
3
4
5
Perfectly
72
Q

A middle-aged man with no significant medical history undergoes a radiograph of the pelvis
for localized tenderness following a fall. Multiple longitudinally orientated, 2–10 mm rounded
densities similar to cortical bone are seen throughout the cancellous bone, in a diffuse
symmetrical pattern concentrated around the acetabulum. There is no fracture. What is the most
likely diagnosis? [B4 Q8]

a. osteopathia striata
b. osteopetrosis
c. bone metastases
d. melorrheostosis
e. osteopoikilosis

A

Osteopoikilosis

Osteopoikilosis is a rare condition causing multiple enostoses (bone islands), which are
asymptomatic and usually of no clinical significance. They represent deposits of normal
cortical bone within the cancellous bone. Osteopathia striata (Voorhoeve’s disease) is similar
to osteopoikilosis in appearance and is usually asymptomatic, but it consists of linear
longitudinal or sunburst striations rather than rounded densities. Osteopetrosis causes
generalized increase in bone density, whereas melorrheostosis is a cortical process giving a
‘flowing wax’ appearance, usually affecting only one side of the affected bone. While
metastases are plausible, the patient would probably be symptomatic and have evidence or a
history of a primary tumour

How well did you know this?
1
Not at all
2
3
4
5
Perfectly
73
Q

A 20-year-old man presents with an increasingly painful right thigh which is worse at night.
Plain films of the area show a lucent area measuring approximately 8–9mm in the distal femur
surrounded by extensive sclerosis. The most likely diagnosis is: [B2 Q4]

a. Osteoblastoma
b. Giant cell tumour
c. Brodie’s abscess
d. Osteoid osteoma
e. Chondroblastoma

A

Osteoid osteoma

This most commonly presents in the second and third decades. The male: female ratio is 2.5:1.
Classically it presents with increasing pain which is worse at night and often relieved with
aspirin. Spinal lesions often lead to painful scoliosis. Almost any site in the body may be
affected but the most common regions are the lower limb and spine

How well did you know this?
1
Not at all
2
3
4
5
Perfectly
74
Q

A 20-year-old athlete presented with chronic leg pain relieved with aspirin. The plain
radiograph shows 2–3 cm area of sclerosis and cortical thickening in the midshaft tibia. CT
shows a 1 cm lytic lesion with central mineralisation. MRI demonstrates a bone oedema pattern
but normal surrounding soft tissues. The most likely cause of underlying pathology is? [B5 Q9]

(a) Stress fracture
(b) Ewing’s sarcoma
(c) Osteomyelitis
(d) Enchondroma
(e) Osteoid osteoma

A

Osteoid osteoma

This typically presents as a small lytic lesion with central mineralisation surrounded by reactive
sclerotic area. On bone scans, the central nidus shows intense uptake surrounded by region of
lesser activity (double density sign). MRI demonstrates bone oedema pattern and typically
relieved with aspirin.

Stress fractures are transverse and linear. Ewing’s sarcoma and infection must be considered if
soft tissue involvement is seen on MRI.

How well did you know this?
1
Not at all
2
3
4
5
Perfectly
75
Q

An osteoid osteoma is thought to be the cause of painful scoliosis in a 20-year-old man. Which
of the following is the single best answer regarding osteoid osteomas? [B3 Q23]

A. The nidus appears sclerotic with surrounding lucency on CT
B. Most found in long bones of the lower limbs
C. The nidus does not enhance on CT
D. Reactive sclerosis around the nidus is uncommon
E. The nidus demonstrates decreased activity on bone scintigraphy

A

Most found in long bones of the lower limbs

Although osteoid osteomas can occur in any bone, they are most common in the
metadiaphyseal femur and tibia. The nidus appears lucent on radiographs, intensely active on
bone scan – with surrounding well-defined luceny (double doughnut sign), isointense to muscle
on T1 and variable SI on T2

How well did you know this?
1
Not at all
2
3
4
5
Perfectly
76
Q

A 33-year-old man presents with a 2-year history of a hard lump on the left middle f inger. A
radiograph shows a 2 cm, well-defined, round, densely sclerotic lesion attached to the cortex
of the proximal phalanx of the left middle finger. No cortical erosion or periosteal reaction is
seen. A bone scan shows no tracer uptake. The most likely diagnosis is? [B5 Q35]

(a) Enostosis
(b) Osteoma
(c) Parosteal osteosarcoma
(d) Osteochondroma
(e) Myositis ossificans

A

Osteoma

The best diagnostic clue is the densely sclerotic, well-defined lesion attached to the parent bone.
Latent lesions show no tracer uptake

How well did you know this?
1
Not at all
2
3
4
5
Perfectly
77
Q

A 19-year-old man presents to his general practitioner with a sudden onset of painful scoliosis.
His pain improves with prescribed aspirin while awaiting MRI. MRI reveals a localized area
of inflammatory change in the left pedicle of L1. Subsequent CT shows marked sclerosis in the same region with a 5 mm, cortically based central lucency. What is the most likely cause?
[B4 Q35]

a. plasma cell cytoma
b. osteosarcoma
c. osteoid osteoma
d. Brodie’s abscess
e. lymphoma

A

Osteoid osteoma

Osteoid osteoma accounts for 12% of benign neoplasms of bone. It is most located in the cortex
of long bones (50% in the femur and tibia) with 15% in the spine, typically the pedicle. It rarely
exceeds 15 mm in size. Young men are most affected, with pain as the predominant presenting
feature due to the extensive inflammatory reaction and vascularity of the lesion. With spinal
lesions this results in a painful positional scoliosis, though most patients experience
improvement of the pain with salicylates. The lucent central area or nidus represents the
underlying pathological process, with the surrounding sclerosis representing reactive
inflammatory change in normal bone. Treatment traditionally was surgical curettage, but
radiologically guided percutaneous radiofrequency ablation is now used.

How well did you know this?
1
Not at all
2
3
4
5
Perfectly
78
Q

The case of a 22-year-old male with typical clinical and radiographic features of osteoid
osteoma is discussed at the musculoskeletal multidisciplinary team meeting regarding
treatment planning. A decision is made to offer radiofrequency ablation (RFA). You have been
asked to consent the patient for the procedure. Which of the following statements is true? [B1
Q40]

A. Up to six repeat procedures may be required.
B. It is performed under local anaesthetic.
C. Biopsy is necessary to confirm diagnosis prior to treatment.
D. Complete symptom relief is seen in 90% after initial therapy.
E. RFA treatment of vertebral osteoid osteomas is contraindicated.

A

Complete symptom relief is seen in 90% after initial therapy.

Osteoid osteoma is a benign, but painful, bone tumour typically found in the lower limbs of
children and young adults. The use of CT-guided RFA is a safe and effective technique and is
the treatment of choice over open surgical approaches. Complete relief of symptoms is
observed in approximately 90% after initial therapy and is reported up to 100% for secondary
procedures. The procedure is performed under general or spinal anaesthesia. A typical clinical
history of night pain relieved by non-steroidal analgesia and radiographic features of the central
nidus are considered sufficiently diagnostic to proceed with RFA. The procedure should not be
performed if there are doubts regarding diagnosis. Osteomas within the spine are potentially
treated by RFA if the nidus is >1 cm from the dura/neural structures. Since most spinal
osteomas are located within the posterior elements, however, RFA is generally unsuitable.

How well did you know this?
1
Not at all
2
3
4
5
Perfectly
79
Q

A 25-year-old man presents with a 4-month history of increasing dull lower back ache. He is
otherwise systemically well. He has no neurological signs. An x-ray of the lumbar spine
demonstrates a slight scoliosis, with an enlarged sclerotic left pedicle of L3. A subsequent CTscan shows a 3-cm lucent focus within the left pedicle of L3, which has expanded the bone.
There is surrounding sclerosis. What is the most likely underlying diagnosis? [B1 Q5]

A. Osteoid osteoma.
B. Enostosis.
C. Osteoblastoma.
D. Osteomyelitis.
E. Intracortical haemangioma.

A

Osteoblastoma.

Osteoblastoma is similar both clinically and histologically to osteoid osteoma, but there are some differences that aid in distinguishing these two entities. Clinically osteoblastoma is typically less painful than osteoid osteoma and does not respond as well to Aspirin. An osteoblastoma in the neural arch of the spine is more likely to cause neurological signs, as these lesions are typically larger and more expansile than osteoid osteoma. The lucent nidus seen in osteoid osteoma is usually less than 1.5–2 cm in size, whereas the nidus in osteoblastoma is usually larger than 2 cm at diagnosis and has less surrounding sclerosis. The nidus may or may not have a calcific focus within, in both these diagnoses.

The appearance described in the question is the subgroup of osteoblastoma that has similar features to osteoid osteoma. Other appearances on imaging of osteoblastoma include an expansile lesion with multiple small calcifications and a peripheral sclerotic rim or, more rarely, an aggressive appearance with osseous expansion, bone destruction, infiltrating soft tissue, and intermixed matrix calcification.

An enostosis (or bone island) may be giant (greater than 2 cm) but should be well defined and densely sclerotic. It is possible a bone abscess could cause a lytic lesion with surrounding sclerosis, but the patient is systemically well, making infection less likely. A bone abscess is also unlikely to be as expansile as the lesion described. Intracortical haemangioma is a very rare diagnosis, usually within the cortex of a long bone, such as the tibia. On CT there is a hypoattenuating lesion, with spotty internal calcification or a ‘wire-netting’ appearance.

How well did you know this?
1
Not at all
2
3
4
5
Perfectly
80
Q

A 26-year-old man presents with dull pain in the left thigh not relieved with salicylates.
Radiograph shows a 3 cm expansile lytic lesion in the mid-shaft of the left femur, which shows
reactive sclerosis and periosteal reaction. The bone scan shows intense tracer uptake. The most
likely diagnosis is? [B5 Q34]

(a) Osteoid osteoma
(b) Osteoblastoma
(c) Osteosarcoma
(d) Osteomyelitis
(e) Aneurysmal bone cyst

A

Osteoblastoma

Also called giant osteoid osteoma when measuring > 2 cm.

Osteoid osteoma are < 2 cm, have a predilection for the axial skeleton and usually respond to
salicylates. Osteosarcomas show cortical destruction, mineralised matrix, and soft tissue mass.
Aneurysmal bone cysts show fluid–fluid levels and no matrix calcification.

How well did you know this?
1
Not at all
2
3
4
5
Perfectly
81
Q

In a 19-year-old male with painful scoliosis, a well-defined 3cm geographic osteolytic lesion
is seen in the right posterior seventh rib with slight expansion and sharp sclerotic margins. CT
is performed and shows punctate calcification within the lesion and adjacent sclerotic bone.
MRI shows low intermediate T1 and intermediate-high T2 signal with bone marrow oedema.
Which is the most likely diagnosis? [B3 Q39]

A. Giant cell tumour
B. Fibrous dysplasia
C. Enchondroma
D. Osteoblastoma
E. Aneurysmal bone cyst

A

Osteoblastomas

Osteoblastomas share clinical and histological features with osteoid osteomas and 3-12% occur
in ribs.

How well did you know this?
1
Not at all
2
3
4
5
Perfectly
82
Q

A 30-year-old woman undergoes plain radiographic imaging of the hand for a palpable, painful
hard lump on the dorsum. Plain radiographs show a well-defined bony mass applied closely to
the diaphysis of the second metacarpal. CT shows a wide-based pedunculated lesion with a
perpendicular orientation to the diaphysis, no cartilage cap, and a matrix of mature trabeculated
bone. What is the most likely diagnosis? [B4 Q54]

a. osteochondroma
b. multiple osteo-cartilaginous exostoses
c. bizarre paraosteal osteo-chondromatous proliferation
d. Codman’s tumour
e. dysplasia epiphysealis hemimelica

osteochondroma are the absence of angulation away from the nearby physis and a wide base

A

Bizarre paraosteal osteo-chondromatous proliferation

Bizarre paraosteal osteochondromatous proliferation (also known as Nora’s lesion) is a rare
condition usually seen in adults in the third and fourth decades of life. Osteochondroma-like
lesions are seen most at the proximal and middle phalanges, followed by the metacarpals and
metatarsals. A relationship to trauma has been suggested but not proven. Other locations that
may be affected include the long bones (especially those of the upper extremity), skull and jaw.
It is thought to be a similar process to that which gives rise to lesions in myositis ossificans,
reactive periostitis and subungual exostosis. On plain radiographs, a well-defined bony mass is
seen attached to the surface of the parent bone. Features differentiating this from

How well did you know this?
1
Not at all
2
3
4
5
Perfectly
83
Q

A 25-year-old man presents with a painful knee. A plain film reveals a lucent area with a wide
zone of transition in the distal femoral metaphysis. MRI reveals fluid–fluid levels. What is the
most likely diagnosis? [B1 Q29]

A. Aneurysmal bone cyst.
B. GCT.
C. Osteosarcoma.
D. Chondroblastoma.
E. Osteoblastoma.

A

Osteosarcoma

The telangiectatic variety of osteosarcoma does show fluid–fluid levels, as does malignant fibrous histiocytoma or any necrotic bone tumour. Telangiectatic osteosarcoma is highly vascular and contains necrotic tissue and blood, with tumour located only along the periphery and septa. MRI will thus reveal enhancing nodularity in the latter locations; this finding will be absent in the case of ABC or GCT. In addition to those mentioned in the stem, the plain film findings include bone expansion and cortical breakthrough.

Unlike the other lesions, osteoblastoma does not demonstrate fluid–fluid levels on MRI; it is more common in the posterior elements of the spine than in the long bones.

ABC, GCT, and chondroblastoma have a narrower zone of transition on plain film than telangiectatic osteosarcoma.

GCT is subarticular.

Chondroblastoma is epiphyseal

How well did you know this?
1
Not at all
2
3
4
5
Perfectly
84
Q

A 22-year-old man presents to his GP with pain in his right knee which is gradually worsening
in severity and is relatively resistant to analgesia. MRI of the knee demonstrates an area of
geographic bone destruction in the distal femur with a wide zone of transition. There is marked
aneurysmal dilatation of the bone and a fluid-fluid level is present within the lesion. The most
likely diagnosis is: [B2 Q11]

a. Plasmacytoma
b. Simple bone cyst
c. Giant cell tumour
d. Telangiectatic osteosarcoma
e. Parosteal osteosarcoma

A

Telangiectatic osteosarcoma

With the MRI finding described, the most likely explanation is that the lesion is a telangiectatic
osteosarcoma. This is a rare type of osteosarcoma with a mean age at presentation of 20 years.
The most common site is around the knee (62%). Fluid-fluid levels are also seen in giant cell
tumours and aneurysmal bone cysts.

How well did you know this?
1
Not at all
2
3
4
5
Perfectly
85
Q

Which of the following indicates telangiectatic osteosarcoma (TOS) rather than an aneurismal
bone cyst (ABC)? [B3 Q26]

A. Enhancing septa without nodularity on MR
B. Marked expansile remodelling of bone
C. Cortical thinning
D. Presence of osteoid matrix with septal regions on CT
E. Presence of haemorrhagic spaces

A

Presence of osteoid matrix with septal regions on CT

Thick peripheral septa with nodularity, presence of an osteoid matrix within nodular or septal
regions, and aggressive growth features such as cortical destruction indicate TOS rather than
ABC.

How well did you know this?
1
Not at all
2
3
4
5
Perfectly
86
Q

A 30-year-old man undergoes MRI of the whole of the left lower limb and pelvis for a mid-
femoral destructive lytic lesion identified on radiography that is thought to represent a primary
bone tumour. MRI shows that the disease is confined to the femur with a 5 cm diaphyseal lesion
and a 1 cm proximal metaphyseal skip lesion. No enlarged lymph nodes are identified. CT
scans of the chest, abdomen and pelvis show two metastatic nodules in the lower lobe of the
left lung. Subsequent biopsy confirms the diagnosis of osteosarcoma. The cancer is correctly
staged as which of the following? [B4 Q78]

a. T1 N0 M0
b. T1 N0 M1a
c. T2 N0 M1b
d. T3 N0 M1a
e. T3 N0 M1b

A

T3 N0 M1a

Complete staging of primary bone sarcomas is unusual in that it also incorporates the
histological staging once tissue diagnosis via biopsy or surgical resection is available. The local
TNM classification is T1 (single lesion less than 8 cm), T2 (single lesion over 8 cm) or T3 (skip
lesions of any size). Nodal staging is N0 (no nodes) or N1 (any number of metastatic nodes).
Metastatic spread is staged, accordingly, as M0 (no metastases), M1a (metastases to lung) or
M1b (any other distant site). Once histology is available, tumours can be staged I–IV

How well did you know this?
1
Not at all
2
3
4
5
Perfectly
87
Q

Of the following subtypes of osteosarcoma, which is associated with the most favourable 5-
year survival? [B4 Q53]

a. multicentric
b. periosteal
c. paraosteal
d. telangiectatic
e. soft tissue

A

Paraosteal

Osteosarcoma is the second most common primary malignancy of bone after multiple myeloma,
accounting for 15% of all primary bone tumours. It usually affects those aged 10–30. Ninety-
five per cent are of the primary osseous type and, of these, paraosteal osteosarcoma has the
most favourable 5-year survival rate of 80%. Other osteosarcomas of the primary osseous type
include periosteal (5-year survival rate 50%) and telangiectatic (less than 20%). Multicentric
refers to synchronous osteoblastic osteosarcomas at multiple sites. It occurs exclusively in
children aged 5–10 and carries an extremely poor prognosis. The soft-tissue type is rare,
representing only 1.2% of all soft-tissue tumours. These lesions are primary soft-tissue tumours
with no attachment to bone. Death occurs within 3 years in most cases, tumour size being the
major predictor of outcome

How well did you know this?
1
Not at all
2
3
4
5
Perfectly
88
Q

A 25-year-old woman attends A&E after falling onto her right hand. A plain film of her hand
is taken to exclude fracture. No bony injury is seen. On examination, however, there is painless
swelling of the right index finger which she says has been present for a few weeks. Incidental
note is made of a small central lesion within the medullary cavity of the middle phalanx of the
index finger. There is no cortical breakthrough or periosteal reaction but there is bulbous
expansion of the bone with thinning of the cortex. The lesion contains dystrophic calcifications.
This is most likely to represent: [B2 Q38]

a. Giant cell tumour of the tendon sheath
b. Unicameral bone cyst
c. Brown tumour
d. Enchondroma
e. Epidermal inclusion cyst

A

Enchondroma

This lesion is most likely to be an enchondroma. This is a benign cartilaginous growth in the
medullary cavity and is usually asymptomatic. It most commonly occurs in the small bones of
the hands and wrist but may also occur in the proximal humerus and proximal femur.
Epidermoid inclusion cysts are usually in the distal phalangeal tuft and there is often a history
of trauma. A bone cyst would be unusual in the phalanges.

How well did you know this?
1
Not at all
2
3
4
5
Perfectly
89
Q

A 20-year-old woman presents with pain after injury to the index finger. The radiograph shows
a 2 cm lytic lesion with a matrix containing calcifications. There is endosteal scalloping of the
cortex with cortical expansion, but no cortical breach. The most likely diagnosis is? [B5 Q33]

(a) Bone infarct
(b) Enchondroma
(c) Chondrosarcoma
(d) Juxtacortical chondroma
(e) Epidermoid cyst

A

Enchondroma

Radiographic features are typical and diagnostic. A bone infarct appears as a serpiginous area
with sclerotic margins and no endosteal scalloping.

Chondrosarcoma shows periosteal reaction and an associated soft tissue mass.

How well did you know this?
1
Not at all
2
3
4
5
Perfectly
90
Q

Plain radiographs of the hands in a young woman are performed for unilateral deformity. These
show multiple lytic lesions in the medullary cavities of the tubular bones with cortical
expansion and matrix mineralization, and associated Madelung deformity. The changes are
unilateral. What is the most likely diagnosis? [B4 Q23]

a. Maffucci’s syndrome
b. Ollier’s disease
c. Trevor’s disease
d. Lichtenstein–Jaffe´ disease
e. Morquio’s syndrome

A

Ollier’s disease

Ollier’s disease or multiple enchondromatosis is characterized by the presence of benign
intraosseous cartilaginous tumours. The estimated prevalence of the disease is 1 in 100 000.
The distribution and number of lesions are variable but are often unilateral and monomelic.
Complications include pain, skeletal deformities, limb length discrepancy (including
Madelung’s deformity) and the potential risk of malignant change to chondrosarcoma in 20–
50% of cases. The condition in which enchondromas are associated with haemangiomas is
known as Maffucci’s syndrome. Neither is usually inherited. Trevor’s disease is an epiphyseal
dysplasia, whereas Lichtenstein–Jaffe´disease is another name for fibrous dysplasia. Morquio’s
syndrome is one of the lysosomal storage disorders known as the mucopolysaccharidoses.

How well did you know this?
1
Not at all
2
3
4
5
Perfectly
91
Q

A 21-year-old man presents with multiple swellings and focal areas of bluish discoloration in
both hands. Plain radiograph of the hands show multiple, welldefined, expanded lytic lesions
in the metacarpals. These lesions show stippled calcifications in the matrix and cause cortical
thinning. Multiple small round calcifications are seen in the surrounding soft tissues. T he most
likely diagnosis is? [B5 Q17]

(a) Ollier’s disease
(b) Maffucci’s syndrome
(c) Metastases
(d) Diaphyseal aclasis
(e) Kaposi’s sarcoma

A

Maffucci’s syndrome

This diagnosis is a combination of multiple enchondromatosis and soft tissue haemangiomas.
Hand and foot involvement is common and severe. The soft tissue calcifications are phleboliths
from haemangiomas.

In Ollier’s disease, there is absence of haemangioma and phleboliths.

How well did you know this?
1
Not at all
2
3
4
5
Perfectly
92
Q

A 29-year-old woman presents with a painful right knee which has been worsening over the
previous few weeks. A plain film of the right knee shows an oval expansile lesion with a
radiolucent centre in the metaphyseal region of the proximal tibia. There is a sclerotic margin
and geographic bone destruction. There are internal septations and stippled calcification. There
is no periosteal reaction. The most likely diagnosis is: [B2 Q55]

a. non-ossifying fibroma
b. Chondroblastoma
c. Giant cell tumour
d. Chondromyxoid fibroma
e. Chondrosarcoma

A

Chondromyxoid fibroma

This is most seen in the second and third decades and the most common site is the long bones, most often the proximal tibia and distal femur. Non-ossifying fibroma is usually asymptomatic. The appearances of a chondroblastoma would be similar but this would most likely be epiphyseal in location and usually presents in a slightly younger age group.

How well did you know this?
1
Not at all
2
3
4
5
Perfectly
93
Q

A 16-year-old boy fell playing football and hurt his left knee. He has some difficulty weight-
bearing and presents to the A&E department. An x-ray of his left knee is performed. This
demonstrates a small joint effusion, but no fracture is seen. An approximately 3-cm diameter,
well-defined lucent bony lesion, with a thin sclerotic margin, is identified within the proximal
epiphysis of the tibia. No internal calcification is evident on plain x-ray. What is the most likely
diagnosis for this abnormality? [B1 Q23]

A. Chondromyxoid fibroma.
B. Enchondroma.
C. GCT.
D. Chondroblastoma.
E. Chondrosarcoma

A

Chondroblastoma.

These are radiolucent lesions that typically occupy the epiphysis of long bones in younger
people, usually before skeletal maturity. They tend to be less than 4 cm in size, with
approximately threequarters having a sclerotic border and one-third a calcified matrix seen on
plain radiographs.

Chondromyxoid fibromas are rare benign tumours occurring in predominantly the second and
third decades of life. They characteristically have sclerotic margins and appear lobulated or
‘bubbly’. They usually arise in the metaphysis of long bones with occasional diaphyseal
extension.

GCTs tend to occur in young adulthood following skeletal maturity. Patients usually present
with pain. The lesion is purely lytic, typically with well-defined, but non-sclerotic, margins.
When present in long bones, the lesions are typically metaphyseal, extending across a fused
epiphysis to a subarticular location. Periosteal reaction is atypical, but expansile remodelling,
cortical penetration, and soft-tissue extension may be seen.

How well did you know this?
1
Not at all
2
3
4
5
Perfectly
94
Q

An 18-year-old student who fell two stairs and landed on her left knee attends A&E
complaining of generalised knee pain but can weight bear. No acute bony injury is
demonstrated on plain film, however a pedunculated lesion arising from the femoral
metaphysis and extending away from the knee joint is seen. The lesion shows continuity with
both the marrow and the cortex. The most likely diagnosis is: [B2 Q28]

a. Osteochondroma
b. Osteoblastoma
c. Osteoid osteoma
d. Chondroblastoma
e. Chondromyxoid fibroma

A

Osteochondroma

The description is classic for an osteochondroma or osteocartilagenous exostosis. These lesions are the most common benign growths of the skeleton, are usually found incidentally and are usually asymptomatic unless complications arise. Complications include fracture, vascular compromise, bursa formation and malignant transformation into chondrosarcoma.

How well did you know this?
1
Not at all
2
3
4
5
Perfectly
95
Q

A 12-year-old boy presents with a hard lump around his right knee. A radiograph shows a bony
projection from the medial part of the tibial metaphysis with continuity of the cortex and
medulla of the tibia. What is the most likely diagnosis? [B5 Q32]

(a) Osteochondroma
(b) Parosteal osteosarcoma
(c) Chondrosarcoma
(d) Periosteal osteosarcoma
(e) Juxtacortical myositis ossificans

A

Osteochondroma

The best diagnostic feature of osteochondroma is continuity of the bony cortex and medulla
with the parent bone.

How well did you know this?
1
Not at all
2
3
4
5
Perfectly
96
Q

On MRI performed for a tender osteochondroma of the femoral metaphysis in an adult, which
feature is most useful in determining the presence of malignant change? [B4 Q42]

a. thickness of the cartilage cap
b. lesion size
c. compression of local nerves
d. fracture of the stalk
e. bursa formation

A

Thickness of the cartilage cap

Osteochondromas are the commonest bone tumours and are considered developmental
exostoses rather than true neoplasms. They represent 20–50% of benign and 10–15% of all
bone tumours. They are made up of cortical and medullary bone and an overlying cartilage cap.
The cortex and medulla of the osteochondroma are continuous with the underlying host bone.
They are typically orientated away from an adjacent distal joint. Lesions are frequently solitary,
but multiple lesions are seen in hereditary multiple exostoses, an autosomal dominant
syndrome. Malignant transformation occurs in 1% of solitary lesions and in 3–5% of patients
with hereditary multiple exostoses. After skeletal maturity, continued lesion growth,
particularly of the cartilage cap, is suggestive of malignant transformation
. Although benign
lesions may reach 10 cm in size, the cartilage cap should not exceed 1.5 cm after skeletal
maturation. Any bone that develops by enchondral ossification may develop an
osteochondroma, the long bones of the lower extremity being most frequently affected.

How well did you know this?
1
Not at all
2
3
4
5
Perfectly
97
Q

A 20-year-old man is referred for suspected malignant transformation of an osteochondroma.
Which of the following is a cause for concern in an osteochondroma? [B3 Q17]

A. New lucency
B. Reduced scintigraphic activity
C. Growth before physeal closure
D. Asymptomatic nature
E. Cartilagenous cap > 0.5cm

A

New lucency

Concerning features for Osteochondroma malignant transformation include:

New lucency
Increased scintigraphic activity
Gowth after skeletal maturation
Pain after puberty
Cortical destruction
Cartilaginous cap > 1.5cm.

How well did you know this?
1
Not at all
2
3
4
5
Perfectly
98
Q

You are reviewing a plain film of pelvis of a 70-year-old woman with recent hip pain. She has
a past medical history of bronchial carcinoid. You notice thick, coarsened trabeculae of the left
iliac bone
, but in comparison to a previous film there is an area of cortical destruction with
‘ring-and-arc’ calcification
. There is no adjacent periosteal reaction. Which of the following is
the most significant pathology present? [B1 Q58]

A. Paget’s disease.
B. Chondrosarcoma.
C. Osteosarcoma.
D. Chondroblastoma.
E. Lung metastasis.

A

Chondrosarcoma
The findings describe development of chondrosarcoma in an area of Paget’s disease. While
osteosarcoma is more common than either malignant fibrous histiocytoma or chondrosarcoma
in Paget’s disease, the ‘ring-and-arc’ calcification in the vignette indicates chondroid rather
than osteoid calcification. Sarcomatous transformation in Paget’s is rare, occurring in
approximately 1% of cases, but should be suspected if there is new focal pain or swelling. Such
lesions, even osteosarcomas, are usually lucent. Periosteal reaction is often absent due to the
rapidity of bone destruction.

Other complications of Paget’s disease include those related to osseous weakening (deformity
and fracture), arthritis, neurological entrapment, and both benign and malignant GCT.

Chondroblastoma may have internal chondroid calcification (60%) but is a well-defined,
benign, lucent lesion with a sclerotic rim occurring in the epiphyses of children and young
adults.
Bronchial carcinoid metastases are usually purely osteoblastic (i.e., sclerotic, not lucent).

How well did you know this?
1
Not at all
2
3
4
5
Perfectly
99
Q

Which of the following favours chondroma rather than chondrosarcoma? [B3 Q34]

A. Size > 3cm
B. Pain
C. Age > 30
D. Location in hands and feet
E. Permeation into soft tissues

A

Location in hands and feet

Chondromas are usually < 3cm, painless and in younger patients in the peripheral skeleton

How well did you know this?
1
Not at all
2
3
4
5
Perfectly
100
Q

Plain radiographs of the femur performed for pain reveal a centrally located lucent lesion in
the medulla with a partially calcified matrix. Which of the following features favours a
diagnosis of chondrosarcoma over enchondroma? [B4 Q100]

a. arc-and-ring matrix calcification
b. ground-glass matrix
c. multiple lesions
d. deep endosteal scalloping
e. lesion size over 5 cm

A

Deep endosteal scalloping

Distinction of enchondroma and intramedullary chondrosarcoma in the appendicular skeleton
proximal to the metacarpals/-tarsals is difficult radiologically. A series of 187 patients showed
that chondrosarcoma was associated with endosteal scalloping, with scalloping involving more
than two-thirds of the extent of the lesion being strongly suggestive of malignancy. Other
powerful discriminating factors identified as favouring chondrosarcoma were cortical
destruction, soft-tissue mass, periosteal reaction, radionuclide uptake at scintigraphy and pain
associated with the lesion. Chondrosarcoma also tended to be larger with a mean size of 10 cm
compared with 6.7 cm for enchondroma. A ground-glass matrix with arcuate calcification is
characteristic of both types of cartilaginous lesion. Multiple lesions may be seen in both
malignancy and enchondromatosis (Ollier’s disease).

How well did you know this?
1
Not at all
2
3
4
5
Perfectly
101
Q

A middle-aged woman, known to suffer from polyostotic fibrous dysplasia, presents with a
palpable, 3 cm, soft-tissue mass in the upper left thigh. MRI shows a relatively homogeneous,
smooth, well-defined lesion located in an atrophic quadriceps muscle, which returns low signal
on T1W images and high signal on T2W images. Following administration of intravenous
gadolinium, the lesion shows moderately intense heterogeneous enhancement. Whatis the most
likely pathological nature of the soft-tissue lesion? [B4 Q59]

a. soft tissue myxoma
b. malignant fibrous histiocytoma
c. soft-tissue cavernous haemangioma
d. multiple lipomatosis
e. rhabdomyosarcoma

A

Soft tissue myxoma

The association of fibrous dysplasia and soft tissue myxoma is well established and is
commonly termed Mazabraud’s syndrome. The key is identifying the relationship between the
bone and soft-tissue pathology, with the osseous features of fibrous dysplasia usually preceding
the formation of a soft-tissue mass. The condition is non-familial and more commonly affects
women, the thigh being the most common location. Typical MRI appearances are of a well-
defined lesion with signal intensity like water, and often a fat rind or adjacent muscle high
signal on T2W images is seen. Although uncommon, there have been reported cases of
malignant change into osteosarcoma

How well did you know this?
1
Not at all
2
3
4
5
Perfectly
102
Q

An 18year old man undergoes a TcMDP bone scan to investigate pain in the right hip. A ‘hot’
lesion is seen in the right proximal femur. No other lesions are seen. Which of the following
lesions would appear as ‘hot’ on a Tc MDP bone scan? [B2 Q53]

a. Osteopoikilosis
b. Fibrous cortical defect
c. Acute fracture within 12 hours of injury
d. Fibrous dysplasia
e. Haemangioma

A

Fibrous dysplasia

The most common site of monostotic fibrous dysplasia is the ribs, followed by proximal femur
and craniofacial bones. Three-quarters of cases present before age 30. Other benign lesions
causing a ‘hot’ on bone scan include Paget’s disease, brown tumours, aneurysmal bone cysts,
osteoid osteoma and chondroblastoma. Acute fractures are not usually ‘hot’ until after the first
24–48 hours.

How well did you know this?
1
Not at all
2
3
4
5
Perfectly
103
Q

A 70-year-old man undergoes CT of the skull for investigation of clinically apparent
macrocephaly confirmed on skull radiography. You are asked by the referring clinician to
review the images. Which finding is most likely to support a diagnosis of fibrous dysplasia
over Paget’s disease? [B4 Q91]

a. widened diploe
b. asymmetrical involvement of the skull
c. sparing of the paranasal sinuses
d. osteoporosis
e. ground-glass medulla

A

Ground-glass medulla

A ground-glass appearance is characteristic of fibrous dysplasia and is the most useful
discriminating factor. Other features of fibrous dysplasia of the skull that can help distinguish
it from Paget’s disease are symmetry of distribution, presence of a soft-tissue mass, cyst-like
changes, thickness of the cranial cortices, and involvement of the paranasal sinuses, maxilla,
sphenoid, orbits, and nasal cavity.

How well did you know this?
1
Not at all
2
3
4
5
Perfectly
104
Q

A 34-year-old man has an MRI of the lumbar spine for lower back pain. This is normal apart
from a focal lesion present in the L4 vertebral body. This is reported as a vertebral
haemangioma. Which of the following MRI characteristics does this lesion most likely have?
[B1 Q53]

A. decreased T1, decreased T2, decreased STIR
B. decreased T1, decreased T2, increased STIR
C. decreased T1, increased T2, decreased STIR
D. increased T1, decreased T2, decreased STIR
E. increased T1, decreased T2, increased STIR
F. increased T1, increased T2, decreased STIR
G. increased T1, increased T2, increased STIR

A

increased T1 increased T2 decreased STIR

The variable proportions of vascular and fatty soft-tissue elements influence the MRI
appearance of haemangiomas. Lesions with a predominantly fatty matrix show high signal
intensity on T1WI, intermediate to high signal intensity on T2WI, and loss of signal on STIR
or fat suppressed T2WI. If the vascular elements predominate, the lesions appear hypointense
on T1WI and extremely hyperintense on STIR and T2WI. If MRI is inconclusive, CT may be
helpful in identifying the typical pattern of haemangiomatous bone replacement, such as the
honeycomb, ‘soap bubble’ or ‘sunburst’ appearance.

How well did you know this?
1
Not at all
2
3
4
5
Perfectly
105
Q

A 40-year-old woman with lung cancer and multiple liver metastases presents with back pain
and left L5 radiculopathy. Plain radiography demonstrates prominent vertical striations in L4
vertebral body. MRI scans demonstrated a sequestrated disc pressing the left L5 nerve root in
the lateral recess. The L4 vertebral body shows a well-defined lesion, returning high signal on
T1 and T2. The lesion in the L4 vertebral body is? [B5 Q8]

(a) Intraosseous ganglion
(b) Metastatic deposit from lung cancer
(c) Haemangioma
(d) Osteoid osteoma
(e) Schmorl’s node

A

Haemangioma

Vertebral haemangiomas are visualised with fine or coarse vertical striations, commonly seen
in vertebral bodies. CT shows a dotted appearance in a fatty matrix. On MRI, they present as
lesions, returning high signal on T1 and T2 due to their high fat content.

Metastatic lesions are Intermediate signal on T1 and high on T2. Schmorl’s node affects the
end plates.

How well did you know this?
1
Not at all
2
3
4
5
Perfectly
106
Q

A middle-aged woman undergoes an MRI of the lumbar spine for longstanding lower back
pain. She has no specific neurological signs and is otherwise well. MRI shows some lower
lumbar spine facet joint arthropathy and a 22cm well-defined rounded lesion in the L3 vertebral body. This displays high signal on both the T1 and T2 sequences. The most likely explanation
for this lesion is: [B2 Q13]

a. Discitis
b. Lymphoma
c. Myeloma
d. Metastatic deposit
e. Haemangioma

A

Haemangioma

This is most likely to be a benign haemangioma. These are relatively common lesions seen as
incidental findings on spinal imaging. High signal on T1 imaging is indicative of the presence
of fat within the lesion. All the other conditions would give a low-signal lesion on T1 imaging.

How well did you know this?
1
Not at all
2
3
4
5
Perfectly
107
Q

Plain radiographs of the spine in a 40-year-old man performed following a roadtraffic collision
reveal a slightly expanded midthoracic vertebral body with coarse vertical trabeculations.
Subsequent CT shows a ‘polka-dot’ appearance to the same vertebral body in the axial plane.
What is the most likely disorder affecting the vertebra? [B4 Q31]

a. aneurysmal bone cyst
b. osteoid osteoma
c. haemangioma
d. compression fracture
e. osteopoikilosis

A

Haemangioma

Metastatic disease, myeloma and lymphoma are the most common malignant spinal tumours,
and haemangioma is the most common benign tumour of the spine. The appearances described
are characteristic of a vertebral haemangioma. On MRI, these lesions typically appear of
mottled low-to-high signal on T1W images depending on the degree of fat present, and of very
high signal on T2W images. Other primary osseous lesions of the spine are more unusual but
may exhibit characteristic imaging features that can help develop a differential diagnosis.
Radiological evaluation of a patient who presents with osseous vertebral lesions often includes
radiography, CT and MRI. The complex anatomy of the vertebrae means that CT is more useful
than conventional radiography for evaluating lesion location and assessing bone destruction.
The diagnosis of spinal tumours is based on patient age, topographic features of the tumour
and lesion pattern as seen on imaging.

How well did you know this?
1
Not at all
2
3
4
5
Perfectly
108
Q

A 35-year-male presents with pain in the thigh. A plain radiograph reveals an eccentric
expansile lucent lesion without a sclerotic margin but with a narrow zone of transition in the
distal femoral metaphysis and epiphysis, which extends to the joint surface. What is the most
likely diagnosis? [B1 Q6]

A. Osteosarcoma.
B. Giant cell tumour (GCT).
C. Metastasis.
D. Aneurysmal bone cyst.
E. Fibrous dysplasia.

A

Giant cell tumour (GCT).

This is the classical description and location of a GCT. They occur age 20–40 years, in the long
bones and occasionally the sacrum and pelvis. They are lucent, eccentric, and expansile but do
not usually produce sclerosis and produce a periosteal reaction in less than a third of patients.
They may have a multiloculated appearance. They originate in the metaphysis but extend to
the subchondral surface in the skeletally mature. MRI often reveals fluid–fluid levels and some
low signal on T2WI due to haemosiderin or collagen deposition. The major differential
diagnosis is an aneurysmal bone cyst (ABC), but this classically has a sclerotic margin
and
usually occurs under 30 years of age (75% occur before the age of 20 years). Fibrous dysplasia
would usually present at a younger age in the metaphysis with extension into the diaphysis; a
trabeculated/ground-glass appearance is typical with a thick sclerotic margin and endosteal
scalloping. Metastasis would be relatively rare at this age and there is no mention of a primary
tumour. Osteosarcoma would have a more aggressive appearance with a wide zone of transition,
periosteal reaction, cortical destruction, and soft tissue extension.

How well did you know this?
1
Not at all
2
3
4
5
Perfectly
109
Q

A 26-year-old woman presents with a 2-year history of an enlarging soft tissue mass in her left
thumb adjacent to the interphalangeal joint. An x-ray of the left thumb shows a soft tissue
swelling with a large well-defined erosion seen affecting the distal metaphysis of the proximal
phalanx. There is no soft tissue calcification or evidence of arthropathy at the interphalangeal
joint. A subsequent MRI scan shows a 3.5-cm well defined soft-tissue mass, which is low signal
on T1WI and enhances post administration of gadolinium. The lesion is low signal on T2WI
and gradient echo (GE) imaging. What is the most likely diagnosis? [B1 Q9]

A. Ganglion cyst.
B. Peripheral nerve sheath tumour.
C. Lipoma.
D. GCT of the tendon sheath.
E. Soft tissue haemangioma.

A

GCT of the tendon sheath.

A GCT of the tendon sheath is a nodular form of PVNS. These tumours are intimately associated with a tendon sheath and are most located in the hand. They usually manifest as a small slow-growing mass, with or without pain. Radiographs may show no abnormality or non-aggressive remodelling of the adjacent bone. In MRI, these lesions are typically hypo- or isointense to muscle on T1WI and T2WI, owing to abundant collagen and haemosiderin, often with enhancement. This is like the findings of diffuse intra-articular PVNS, when the extent of haemosiderin deposition may cause hypointense nodules on T2WI and blooming artifact on gradient echo (GE) sequences. It must be stated that the degree of haemosiderin content may not always be enough to cause marked hypo-intensity on T2WI in GCT of the tendon sheath.

A ganglion cyst could occur in this location and be related to a tendon sheath, but on MRI it is typically hyperintense on T2WI secondary to its fluid component. There may be thin rim enhancement of the wall post administration of gadolinium. Peripheral nerve sheath tumours are typically hyperintense on T2WI with variable contrast enhancement. Lipomas are similar in signal characteristic to subcutaneous fat on MRI, i.e. hyperintense on both T1WI and T2WI.

A soft-tissue haemangioma may contain phleboliths on plain radiographic imaging. On MRI, haemangiomas may be well circumscribed or have poorly defined margins, with varying amounts of increased T1WI signal owing to either reactive fat overgrowth or haemorrhage. Areas of slow flow are typically hyperintense on T2WI, while rapid flow can demonstrate a signal void on images obtained with a non-flow-sensitive sequence.

How well did you know this?
1
Not at all
2
3
4
5
Perfectly
110
Q

A 34-year-old woman presents with pain and swelling of the right knee over the previous 2
months. Plain films demonstrate a well circumscribed, expansile, lytic lesion eccentrically
located in the subarticular region of the right distal femur. The lesion has a narrow, non-
sclerotic zone of transition. What is the most likely diagnosis? [B4 Q1]

a. giant cell tumour
b. enchondroma
c. fibrous cortical defect
d. fibrous dysplasia
e. aneurysmal bone cyst

A

Giant cell tumour

Most giant cell tumours occur in patients with closed epiphyses, and although they may
originate in the metaphysis, lesions typically involve the epiphysis and abut the subarticular
surface. They are classically eccentrically located lesions with a narrow zone of transition, no
sclerosis, and no internal matrix mineralization. Giant cell tumours tend to be locally
aggressive, with a high recurrence rate after initial treatment. Enchondromas are the
commonest benign cystic lesion of the phalanges, though they are also seen in the long bones.
However, those in the long bones almost always contain calcified chondroid matrix.
Aneurysmal bone cysts are often seen as an eccentric lytic expansile lesion, but patients are
nearly all under the age of 30. Monostotic fibrous dysplasia is more commonly seen in the
proximal femur than distally, and lesions tend to have a sclerotic margin. Fibrous cortical
defects are asymptomatic lesions seen in children, which usually regress spontaneously, so they
are only rarely seen after the age of 30. They typically appear as lytic lesions with a thin
sclerotic border in the metaphysis of a long bone.

How well did you know this?
1
Not at all
2
3
4
5
Perfectly
111
Q

Considering the imaging features of extraarticular Pigmented Villonodular Synovitis (PVNS):
[B3 Q29]

A. Typically manifests as a soft tissue mass in 20% of cases
B. Osseous abnormalities are present in the vast majority
C. Extrinsic erosion is the most common osseous abnormality
D. Radiographs are normal in 80% of cases
E. Extensive erosions are more common in the knee

A

Extrinsic erosion is the most common osseous abnormality

Manifests as a soft tissue mass in 50-70% of cases, with normal radiographs in 20% and
osseous abnormalities in 5-25%. The most common osseous abnormality are extrinsic erosions
which are more often present in the ankle and foot.

How well did you know this?
1
Not at all
2
3
4
5
Perfectly
112
Q

Which is the most common location for giant cell tumour? [B3 Q18]

A. Proximal tibia
B. Proximal femur
C. Distal femur
D. Proximal fibula
E. Patella

A

Distal femur

50-65% occur around the knee but are rare in the patella

How well did you know this?
1
Not at all
2
3
4
5
Perfectly
113
Q

A 40-year-old mother presents with knee pain. Plain radiographs show an eccentric, expansile,
lytic lesion with a narrow zone of transition in the lateral femoral condyle extending to
subarticular bone. The lateral cortex is thinned but no periosteal reaction or sclerosis is seen.
MRI shows a well-defined bony lesion with intermediate signal on T1 and mixed signal on T2
and multiple fluid–fluid levels. The most likely diagnosis is? [B5 Q10]

(a) Benign fibrous histiocytoma
(b) Giant cell tumour
(c) Telangiectatic osteosarcoma
(d) Brodie’s abscess
(e) Simple bone cyst

A

Giant cell tumour

Typical features on radiograph are a lytic, eccentric, expansile, subarticular lesion with ‘soap
bubble’ appearance (fluid–fluid level on MRI).

Benign fibrous histiocytoma and telangiectatic osteosarcoma can also show fluidfluid levels
but are metaphyseal lesions not extending to the subarticular surface. A simple bone cyst is
unilocular and situated away from the articular surface.

How well did you know this?
1
Not at all
2
3
4
5
Perfectly
114
Q

A 15-year-old boy attended the Accident & Emergency Department with ankle pain after a
twisting injury 7 days previously. The history suggests there has been ill-defined swelling and
ache for a few weeks. The plain radiograph shows a fracture in the distal fibula, with lamellar
periosteal reaction. There appears to be an associated soft tissue bulge. What is the most likely
diagnosis? [B5 Q5]

(a) Fracture with large haematoma
(b) Neuroblastoma metastasis
(c) Lymphoma
(d) Ewing’s sarcoma with fracture
(e) Osteomyelitis

A

Ewing’s sarcoma with fracture

For a simple fracture with haematoma, this case has presented too early for a periosteal reaction.
Neuroblastoma metastasis could be considered in a child less than 5 years old, and lymphoma
should be considered in patients over 30 years age. Osteomyelitis could have been possible if
there had been a previous history of localised pain, fever etc.

How well did you know this?
1
Not at all
2
3
4
5
Perfectly
115
Q

Which of the following causes parallel spiculated (hair-on end) rather than divergent speculated
(sunray) periosteal reaction? [B3 Q7]

A. Osteosarcoma
B. Ewing’s sarcoma
C. Sigmoid colon cancer metastasis
D. Hemangioma
E. Meningioma

A

Ewing’s sarcoma

Other causes include syphilis and infantile cortical hyperostosis.

How well did you know this?
1
Not at all
2
3
4
5
Perfectly
116
Q

Which of the following is a cause of a mixed sclerotic/lytic lesion with a button sequestrum?
[B3 Q12]

A. TB
B. Ewing’s sarcoma
C. Eosinophilic granuloma
D. Metastases
E. Osteosarcoma

A

Eosinophilic granuloma

Eosinophilic granuloma is a cause of a mixed sclerotic/lytic lesion with a button sequestrum

How well did you know this?
1
Not at all
2
3
4
5
Perfectly
117
Q

Which of the following demonstrates the most uptake on PET/CT? [B3 Q44]

A. Primary bone lymphoma
B. Osteosarcoma
C. Chondrosarcoma
D. Enchondroma
E. Osteochondroma

A

A Greatest FDG uptake occurs in primary bone lymphoma and Ewing’s sarcoma. Osteosarcoma demonstrates moderate uptake. Most benign bone lesions are non-FDG avid, except for high giant cell containing tumours (Giant Cell Tumors (GCT), osteoblastomas, aneurysmal bone cysts) and fibrous lesions (fibrous dysplasia)

How well did you know this?
1
Not at all
2
3
4
5
Perfectly
118
Q

A ‘fallen fragment’ seen within a lytic bone lesion is most commonly associated with which
of the following? [B4 Q13]

a. aneurysmal bone cyst
b. unicameral (simple) bone cyst
c. giant cell tumour
d. eosinophilic granuloma
e. chondroblastoma

A

Unicameral (simple) bone cyst

The fallen fragment is virtually pathognomonic for a simple bone cyst. It represents a fragment
from a pathological fracture through the lesion, which has fallen to lie in a dependent location
in the cyst matrix

How well did you know this?
1
Not at all
2
3
4
5
Perfectly
119
Q

A 10-year-old boy presented with fracture of the left proximal humerus sustained during a
tackle in a football match. Plain radiographs show a pathological fracture and underlying lytic
lesion in the metaphysis of the proximal humerus. The lesion shows endosteal scalloping and
a small bone fragment in the floor of the cyst. MRI features include intermediate signal on T1
and high signal on T2 with a fluid–fluid level. What is the most likely diagnosis of the
underlying bony lesion? [B5 Q4]

(a) Unicameral bone cyst
(b) Lymphoma
(c) Aneurysmal bone cyst
(d) Telangiectatic osteosarcoma
(e) Giant cell tumour

A

Unicameral bone cyst

This is usually asymptomatic unless fractured. The lesion usually is 2–3 cm in size, usually in
the metaphysis, with its long axis parallel to the long axis of the bone. There may be endosteal
scalloping of the bone and a ‘fallen-fragment sign’ (if fractured, centrally dislodged fracture
fragment falls into a dependent position in the cyst).

How well did you know this?
1
Not at all
2
3
4
5
Perfectly
120
Q

A 16-year-old boy presents with a slowly enlarging, painful swelling in his left lateral chest
wall. A CXR shows an expansile lucent lesion arising from the lateral aspect of the left seventh
rib. An MRI scan is performed for further evaluation, and this demonstrates a lobulated, thin-
walled multiseptated lesion with fluid–fluid levels, the dependent layer of which are
hyperintense on T1WI. What is the most likely diagnosis? [B1 Q43]

A. Fibrous dysplasia.
B. Aneurysmal bone cyst.
C. Enchondroma.
D. Chondroblastoma.
E. Cystic angiomatosis.

A

Aneurysmal bone cyst.

ABC accounts for approximately 5% of primary rib lesions, excluding myeloma. The
radiological findings and age described in the question are classical for this lesion.
Approximately 75% of patients are <20 years of age. The key findings on MRI are the fluid–
fluid levels due to the settling of degraded blood products within the cysts. Fluid–fluid levels
may also be a feature of other lesions, including GCT and chondroblastoma, but the thin, well-
defined margins of an ABC should help to distinguish it from other lesions, particularly at this
young age group.

Fibrous dysplasia is the most common benign rib lesion. The radiographic appearances are
variable but may show unilateral fusiform enlargement and deformity with cortical thickening
and increased trabeculation of one or more ribs. The matrix may appear lytic, may demonstrate
a ground-glass appearance, or rarely be sclerotic. Amorphous or irregular calcifications may
be seen within the lesion on CT, and MRI shows low to intermediate signal on T1WI and
variable T2WI signal.

Enchondromas more typically arise in the anterior cartilaginous portion of the rib. Radiographs
reveal a lobulated, well-demarcated osteolytic lesion that demonstrates mild expansion and
well-defined, sclerotic margins. There is typically matrix calcification and CT is more sensitive
at detecting this when the calcification is subtle. MRI shows T2WI hyperintense foci that
appear to coalesce with one another and reflect the high fluid content of hyaline cartilage.

Chondroblastoma of a rib is reported in the literature but would be exceedingly rare and
typically occurs in the epiphyses of long bones. Cystic haemangiomatosis is a rare disease of
disseminated multifocal haemangiomatous or lymphangiomatous lesions in the skeleton and is
usually an incidental asymptomatic finding.

How well did you know this?
1
Not at all
2
3
4
5
Perfectly
121
Q

A 17-year-old girl presents with pain in the distal forearm which has worsened over the last six
to eight weeks. Plain films show an eccentric lytic radiolucency in the distal radius with a soap-
bubble appearance. The most likely pathology is: [B2 Q1]

a. Enchondroma
b. Aneurysmal bone cyst
c. Simple bone cyst
d. Fibrous dysplasia
e. Chondroblastoma

A

Aneurysmal bone cyst

Aneurysmal bone cyst is most common in females and 75% occur under 20 years of age. The
classic presentation is of pain of relatively acute onset with a rapid increase in severity over 6–
12 weeks. Common locations include the spine, with a slight preponderance for the posterior
elements, and the metaphysis of long bones – femur, tibia, humerus, and fibula. The lesion is
usually expansile with thin internal trabeculations giving it the characteristic soap-bubble
appearance.

How well did you know this?
1
Not at all
2
3
4
5
Perfectly
122
Q

A 35-year-old man sprains his right ankle and attends the A&E department. An x-ray of the
right ankle is performed. This does not show any evidence of a fracture, but the lateral view
does demonstrate a well-defined radiolucent lesion with a faint sclerotic margin in the mid calcaneus. There is some central calcification within the lesion. What is the most likely
diagnosis? [B1 Q28]

A. Simple bone cyst.
B. Normal variant.
C. Enchondroma.
D. Intraosseus lipoma.
E. Bone infarct.

A

Intraosseous lipoma.

This is usually asymptomatic and discovered as an incidental finding in adults between 30 and
60 years. The calcaneus is the most common site for intraosseous lipoma, accounting for
approximately 32% of cases. The key radiographic features are as described. The central
dystrophic calcification seen in approximately 62% of cases is considered pathognomic.

The major differential diagnosis of this lesion is a simple bone cyst, although this would not
contain central calcification. Unlike bone cysts elsewhere, it is seen at this site into adulthood.
Also within the differential diagnosis is a pseudo lesion within the calcaneus, caused by a
relative paucity of trabecular bone at the same location. Again, central calcification would not
be a feature in this phenomenon.

Enchondroma is typically a well-defined osteolytic lesion with central calcification, but it
usually has a predilection for tubular bones and would be exceedingly rare in the calcaneus.
The described appearances are not typical for bone infarct.

How well did you know this?
1
Not at all
2
3
4
5
Perfectly
123
Q

A 53-year-old woman attends A&E with a short history of dull right heel pain. She is otherwise
fit and well and there is no history of trauma. Plain radiographs of the right foot and ankle
reveal a 2cm expansile non-aggressive lesion in the calcaneum. It has a thin, well-defined
sclerotic border. There is no periosteal reaction but there is a small calcified central nidus. The
most likely cause of the lesion is: [B2 Q58]

a. Aneurysmal bone cyst
b. Intra-osseous lipoma
c. Lipoblastoma
d. Fibrous dysplasia
e. Desmoplastic fibroma

A

Intra-osseous lipoma

The calcaneum is a common location for an intra-osseous lipoma. They do, however, also occur
in the extremities, skull and mandible. There is no periosteal reaction unless there is an
associated fracture. Imaging features would be like those of a unicameral bone cyst. They are
often asymptomatic but can present with localised bone pain.

How well did you know this?
1
Not at all
2
3
4
5
Perfectly
124
Q

A 34-year-old sedentary male office worker presents with a 2-month history of heel pain. A
radiograph demonstrates a well-defined lytic lesion in the calcaneum. T his produces mild
expansion with endosteal scalloping and has a central ossified nodule. On MRI, the lesion is
high signal on T1 and T2. What is the most likely diagnosis? [B5 Q3]

(a) Giant cell tumour
(b) Fibrous cortical defect
(c) Intraosseous lipoma
(d) Osteoid osteoma
(e) Solitary bone cyst

A

Intraosseous lipoma

This is an expansile, non-aggressive lesion usually seen in metaphyses. It may contain a focal
area of dystrophic calcification within (secondary to fat necrosis). On MRI, the lesion returns
fat signal on all sequences (high signal on T1 and T2, and low signal on STIR).

How well did you know this?
1
Not at all
2
3
4
5
Perfectly
125
Q

A 50-year-old man who has been previously well presents with low back pain. Plain film
reveals an osteolytic midline lesion in the lower sacrum containing secondary bone sclerosis
in the periphery, as well as amorphous peripheral calcifications. A lateral film shows anterior
displacement of the bladder and rectum. He subsequently develops faecal incontinence. No
additional lesions were discovered after imaging of the whole spine. What is the most likely
diagnosis? [B1 Q1]

A. Osteomyelitis.
B. Ewing’s sarcoma.
C. Chordoma.
D. Myeloma.
E. Sacrococcygeal teratoma.

A

Chordoma.

Plain film is very insensitive for detecting sacral lesions. Metastatic disease is much more
common in the sacrum than primary malignancy. Chordoma is the most common primary
sacral lesion.

It is derived from the embryonic remnants of the notochord and is thus almost always found in
the midline or a paramedian location with respect to the spine. It is most found in the sacrum
(50%), clivus (35%), and vertebrae (15%).
A chordoma manifests as a destructive, lytic lesion,
commonly with internal calcifications, at both plain radiography and CT. A large presacral
soft-tissue component is usually present, as are soft-tissue components within the sacrum and
sacral canal. Symptoms can include pain, sciatica, and rectal bleeding as well as other bowel
and bladder symptoms, reflecting compromise of sacral nerves. The tumours can extend across
the adjacent disc space and sacroiliac joint.

On MRI, chordomas demonstrate low to intermediate signal intensity on T1WI and prominent
increased signal intensity on T2WI. Enhancement of the soft-tissue components is variable, yet
often moderate, on both CT and MR images. Chordomas demonstrate a prominent vascular
stain at angiography. They are locally aggressive and develop in locations that do not permit
easy surgical cure. There is an almost 100% recurrence rate; tumour seeding along biopsy tracts
and surgical incisions can lead to multicentric local recurrences. Metastasis occurs in 5–43%
to liver, lung, regional lymph nodes, peritoneum, skin, and heart. The 5-year survival rate is
66% for adults.

Osteomyelitis in the sacrum is most often due to contiguous spread from a suppurative focus
and we are told this patient was previously well. Ewing sarcoma would occur at a younger age
(peaking at 15 years, 90% manifest between the ages of 5 and 30). In the case of myeloma it
would be atypical for the rest of the spine to be uninvolved. The sacrococcygeal region is the
most common location of teratomas discovered in infancy. It is only rarely discovered in
adulthood. Teratomas are composed of a mixture of cystic and solid components.

The other lesions to be included in the differential diagnosis of such a sacral mass are metastasis,
sarcomas, GCT, chondrosarcoma, and ependymoma.

How well did you know this?
1
Not at all
2
3
4
5
Perfectly
126
Q

A 65-year-old man undergoes radiographs of the lumbar spine and pelvis for lower back pain.
A destructive lytic lesion is identified in the midline of the inferior sacrum with internal areas
of calcification. Subsequent MRI reveals a heterogeneous lesion replacing much of the sacrum,
which returns moderate low signal on T1W and high signal on T2W images, with a soft-tissue
component extending into the presacral soft tissues. The lesion shows patchy moderate
enhancement with intravenous gadolinium. What is the most likely diagnosis? [B4 Q94]

a. metastasis
b. giant cell tumour
c. aneurysmal bone cyst
d. chordoma
e. plasmacytoma

A

Chordoma

Chordomas arise from notochordal rests and therefore almost always occur in the midline. They
are the most common primary malignant sacral tumour and account for 2–4% of all malignant
tumours of bone. They are found at all ages but most commonly occur in the fourth to seventh
decades of life. Approximately half develop in the sacrococcygeal region. There is usually a
large soft-tissue component, and the tumour may extend across the intervertebral disc space or
sacroiliac joint. Overall, the most common sacral lesion is metastasis due to the high red
marrow content, but other primary malignant lesions include myeloma, Ewing’s sarcoma and
lymphoma. The most found benign tumours are giant cell tumours and aneurysmal bone cysts.
Despite being relatively common in the rest of the spine, haemangiomas and osteoid osteomas
are rare.

How well did you know this?
1
Not at all
2
3
4
5
Perfectly
127
Q

Plain film, CT and MRI are performed for the investigation of suspected chordoma. Which is
the best answer? [B3 Q24]

A. Radiographic appearances show sacral osteosclerosis
B. Coarse calcification often present with associated soft tissue
C. Areas of low attenuation within a mass on CT
D. Intermediate SI on T2
E. Arise from the spinal canal

A

Areas of low attenuation within a mass on CT

Chordoma usually appears as a low attenuation mass on CT

How well did you know this?
1
Not at all
2
3
4
5
Perfectly
128
Q

A 60-year-old man with several months’ history of back pain, worse when sitting, and with no
bowel or bladder symptoms, undergoes evaluation with MRI. This shows a lobulated presacral
mass, low SI on T1 with several areas of high SI within it, most likely to represent areas of
calcification and haemorrhage. Which is the most probable diagnosis? [B3 Q25]

A. Chordoma
B. Chondrosarcoma
C. Myxopapillary ependymoma
D. Metastasis
E. Giant cell tumour

A

Chordoma

This is the most common primary malignant tumour of the sacrum. Giant cell tumours are the
second most common cause and are indistinguishable from chordomas on MRI.

How well did you know this?
1
Not at all
2
3
4
5
Perfectly
129
Q

A 24-year-old woman presents with worsening frontal headaches and a sixth nerve palsy. A
non-enhanced CT shows a lesion situated within the clivus with associated bony destruction;
there is soft-tissue extension into the nasopharynx. MRI shows a large inter-osseous mass
which is isointense to brain T1-weighted imaging and hyperintense on T2. The most likely
diagnosis is: [B2 Q47]

a. Sphenoid sinus cyst
b. Meningioma
c. Nasopharyngeal carcinoma
d. Metastasis
e. Spheno-occipital chordoma

A

Spheno-occipital chordoma

The most likely cause is a spheno-occipital chordoma. This is associated with bony destruction
in 90% of cases and is most usually within the clivus. Other sites include the sella, petrous
temporal bone, floor of middle cranial fossa and jugular fossa. Sacrococcygeal chordoma is the
most common subtype of chordoma and is usually located with the fourth or fifth sacral
segments. Vertebral/spinal chordoma accounts for only 15–20% of all chordomas and is most
often situated in the cervical spine.

How well did you know this?
1
Not at all
2
3
4
5
Perfectly
130
Q

A 60-year-old woman undergoing follow-up CT under the care of the oncologist develops a
new expansile lytic lesion. Which of the following primary tumours usually causes an expansile
lytic metastasis? [B3 Q6]

A. Cervix
B. Uterus
C. Ovary
D. Thyroid
E. Rectum

A

Thyroid

Renal cell carcinoma also causes expansile lytic metastases

How well did you know this?
1
Not at all
2
3
4
5
Perfectly
131
Q

lytic lesions within the scapula and clavicle secondary to metastatic malignant spread. Which
of the following is most likely to be the primary site of malignancy? [B4 Q12]

a. renal
b. breast
c. cervical
d. colon
e. bronchus

A

Renal

The common cancers that typically metastasize to bone are breast, lung, thyroid, renal and
prostate. Due to the high prevalence of colon cancer, even though only a relatively small
proportion metastasizes to bone, it forms a significant proportion of bone metastases. Prostatic
metastases are typically sclerotic, whereas breast deposits are mixed. Colonic bone metastases
are usually lytic, with renal metastases typically lytic and expansile due to their highly vascular
nature. Other less frequent sources of lytic expansile metastases include thyroid, melanoma
and phaeochromocytoma.

How well did you know this?
1
Not at all
2
3
4
5
Perfectly
132
Q

A 35-year-old male presents with a history of backache. Plain radiograph demonstrates
reduction in the lumbar L2/3-disc space with mild endplate irregularity. An MRI of lumbar
spine is carried out for further assessment. What feature on MRI is useful in differentiating
discitis from modic type I endplate change? [B1 Q31]

A. Reduction in the disc height.
B. Low signal change in the endplate on T1WI.
C. High signal change in the endplate on T2WI.
D. Mild irregularity of the endplates.
E. High signal within the disc on T2WI.

A

High signal within the disc on T2WI.

Degenerative disc disease with associated degenerative modic type 1 endplate change (endplate
oedema) can mimic discitis. All the mentioned changes are seen in both conditions except high
signal within the disc on T2WI, which is seen in infection/discitis. In contrast, the degenerated
disc is of low signal due to loss of hydration.

In addition, disc enhancement and paravertebral inflammatory tissue, soft-tissue mass, and fluid
collection are associated with infection.

How well did you know this?
1
Not at all
2
3
4
5
Perfectly
133
Q

A 56-year-old male is admitted under the orthopaedic team with increasingly severe lower back
pain which started three weeks ago. MRI demonstrates an oedematous L4/5 intervertebral disc,
marked loss of disc material and oedematous adjacent endplate changes. There is associated
paravertebral inflammatory tissue and a small amount of pus within the residual disc space.
The findings are consistent with infective discitis. What is the most likely causative organism?
[B2 Q 33]

a. Mycobacterium tuberculosis
b. Streptococcus pyogenes
c. Staphylococcus aureus
d. Escherichia coli
e. Salmonella

A

Staphylococcus aureus

The most common cause of infective discitis is Staphylococcus aureus, which gives the above
typical findings. The only other relatively common cause is Mycobacterium tuberculosis,
which typically spares the disc until late and usually has a large amount of associated pus.

How well did you know this?
1
Not at all
2
3
4
5
Perfectly
134
Q

A 65-year-old Asian man with a long history of back pain is investigated with MRI. Which is
a feature of tuberculosis rather than pyogenic vertebral collapse? [B3 Q10]

A. Rapid progression
B. Marked osteoblastic response
C. Less sclerosis
D. Less collapse
E. Small or no paravertebral abscess

A

C.

Other features of tuberculosis include slow progression, marked collapse and a large
paravertebral abscess, with or without calcification.

How well did you know this?
1
Not at all
2
3
4
5
Perfectly
135
Q

A 16-year-old girl who has recently moved from India to the UK has back pain. Considering
tuberculous spondylitis: [B3 Q30]

A. 10% of skeletal TB involves the spine
B. Infection usually begins in the posterior part of the vertebral body
C. Medial bowing of the psoas shadow on plain film may indicate an abscess
D. The upper thoracic spine is most affected
E. Calcification within a psoas abscess is highly likely to represent TB

A

Calcification within a psoas abscess is highly likely to represent TB

50% of skeletal TB involves the spine, with the lower thoracic and upper lumbar regions being
most affected. It usually begins at the anterior vertebral body. A psoas abscess may cause lateral
bowing of the psoas shadow on plain film.

How well did you know this?
1
Not at all
2
3
4
5
Perfectly
136
Q

A lumbar spinal MRI is performed on a young man of south-east Asian origin for back pain
and pyrexia of unknown origin. It reveals an anterior paraspinal soft-tissue mass at levels L1
to L3 centred at the L2–3 intervertebral disc. It is located deep to and displaces the anterior
longitudinal ligament and extends into the left psoas muscle. The mass returns intermediate
signal on T1W images and high signal on T2W images. There are oedematous changes in the
adjacent vertebral bodies, but the intervertebral discs are spared. What is the most likely
infectious organism? [B4 Q27]

a. Mycobacterium tuberculosis
b. Actinomyces
c. HIV
d. Staphylococcus aureus
e. Aspergillus fumigatus

A

Mycobacterium tuberculosis

The musculoskeletal system is affected in only 1–3% of tuberculous infections, but the spine
is the most common skeletal location affected, accounting for 50% of musculoskeletal
tuberculosis. Tuberculous spondylitis (or Pott’s disease) can result in significant neurological
sequelae. A history of pulmonary infection may or may not be present. The infection usually
begins in the anterior vertebral body via haematogenous spread. The intervertebral discs are
frequently involved, and the loose internal structure of the disc allows the infection to
disseminate more widely, often resulting in paraspinal or psoas abscess. Calcification within
the abscess is very specific for tuberculosis.
The disease process often leads to vertebral
collapse with gibbous deformity
and obliteration of the disc space. However, elevation of the
anterior ligaments by subligamentous abscess allows tracking superiorly and inferiorly, and
classically spares the disc. Tuberculosis characteristically results in little reactive sclerosis or
periosteal reaction, which helps to distinguish it from pyogenic infections.

137
Q

On radiographs and MRI of the spine performed for lower back pain with clinical signs of
radiculopathy, which of the following features favours a diagnosis of discitis rather than
degenerative disc disease? [B4 Q88]

a. vacuum phenomenon in the discs
b. reduced disc space
c. intermediate signal posterior to the vertebral body on T1W images
d. vertebral endplate low signal on T1W images
e. Schmorl’s nodes

A

Intermediate signal posterior to the vertebral body on T1W images

Intermediate signal in the extradural space on T1W images is the most common appearance of
extradural abscess formation. The most common primary focus of infection is discitis, but
abscess formation may also be spontaneous. Patients particularly at risk are those with a history
of diabetes mellitus, intravenous drug use, trauma, haemodialysis, or recent surgery
(particularly dental). MRI features of extradural abscess include iso- or slight hyperintensity
on T1W images when compared with the spinal cord. High signal on T2W and proton density
sequences makes it difficult to differentiate abscess from CSF, but these sequences are useful,
as osteomyelitis and paravertebral abscess are well visualized as high-signal lesions.
Administration of intravenous gadolinium contrast characteristically demonstrates diffuse
enhancement of the solid component of the abscess.

138
Q

A 60-year-old diabetic man with a 7-day-old compound fracture of the right tibia and fibula
develops fever and septicaemia. Radiography of the leg shows a fracture of the mid shaft of
tibia and fibula, along with extensive air in the soft tissues extending to the ankle and knee.
What is the most likely diagnosis? [B5 Q48]

(a) Air secondary to compound fracture
(b) Aerobic bacterial infection
(c) Clostridium infection
(d) Staphylococcus infection
(e) Beta-haemolytic streptococci

A

Clostridium infection

Gas in the soft tissues after compound fractures is a manifestation of infection. Gas gangrene
after Clostridium infection is a classic example and causes extensive oedema, necrosis of
tissues with gas production resulting in a severe toxic state.

Other gas-forming organisms include anaerobic bacteria, coliforms and Bacteroides.

139
Q

A 75-year-old male with a history of backache undergoes plain radiographs of the lumbar spine,
which demonstrate diffuse bone sclerosis. An MRI demonstrates diffuse low signal intensity
of bone marrow on all sequences with no architectural distortion. The MRI planning sequence
demonstrates splenomegaly. What is the diagnosis? [B1 Q11]

A. Sickle-cell anaemia.
B. Lymphoma.
C. Osteoblastic metastasis.
D. Myelofibrosis.
E. Myeloma.

A

Myelofibrosis

This is associated with collagen proliferation in the marrow, which may be primary or
secondary/ reactive following other myeloproliferative disorders. The highly structured
collagen matrix has tightly bound protons that do not resonate or produce significant signal. As
a result, the marrow is of diffuse low signal intensity on all sequences. There is also no
architectural distortion.

In contrast, myeloma, metastases, and lymphoma are generally associated with focal or
multifocal signal abnormalities. Sickle-cell anaemia causes bone sclerosis secondary to bone
infarcts, which have an irregular appearance.

In myelofibrosis, splenomegaly is secondary to extramedullary hematopoiesis. The presence
of splenomegaly is helpful in limiting the differential diagnoses. Sickle-cell anaemia is
associated with a small, sometimes calcified, spleen due to auto-infarction.

140
Q

A 56-year-old man has a 6-week history of dull discomfort just above his right ankle. A plain
ankle radiograph is performed, and this demonstrates a relatively ill-defined area of lucency in
the distal tibial metaphysis. An underlying aggressive lesion is suspected, and the patient is
referred for an MRI of the distal right leg. This shows a rather serpiginous-shaped lesion in the
distal right tibia. A parallel rim of hypo- and hyperintensity is seen on one of the imaging
sequences, which is very helpful in confirming that the lesion is secondary to meta-diaphyseal
osteonecrosis rather than a neoplasm. On which imaging sequence is this parallel rim most
likely to be seen?

A. GE T2*WI.
B. Fast spin echo (SE) T1WI.
C. Fast SE T2WI.
D. Fast SE STIR.
E. Fast SE T1WI post gadolinium.

A

Fast SE T2WI.

The parallel rim of hypo- and hyperintensity seen on T2WI refers to the ‘double line’ sign,
which is almost pathognomic of osteonecrosis. It is most associated with avascular necrosis of
the femoral head but can be seen in osteonecrosis at other sites on MRI. The ‘double line’ sign constitutes a hyperintense inner border (inflammatory response of bone with granulation tissue),
with a hypointense periphery (reactive bone interface).

The characteristic plain radiographic pattern of meta-diaphyseal osteonecrosis is that of a
serpentine ring-like band of sclerosis that separates a central necrotic zone of variable lucency
from surrounding normal marrow, although this pattern is a relatively late manifestation of
osteonecrosis. Earlier in the course of the disease, osteonecrosis may result in a poorly defined
region of lucency within the medullary space, a feature that may be indistinguishable from a
lytic neoplastic process on x-ray. MRI is then very useful in these cases by showing the
serpentine low signal rim of the lesion on T1WI. On T2WI, the rim of the lesion may have low
signal, high signal, or both (the ‘double line’ sign), the latter being the most specific sign for
osteonecrosis.

141
Q

A 35-year-old female with a history of flushing, pruritis, and diarrhoea is referred for a small
bowel series. A barium study demonstrates irregular diffuse thickening of small bowel folds.
There is also diffuse osteosclerosis. Laboratory tests reveal elevated serum tryptase level. What
is the diagnosis? [B1 Q16]

A. Mastocytosis.
B. Intestinal lymphangectasia.
C. Amyloidosis.
D. Waldenstrom’s macroglobulinaemia.
E. Whipple’s disease.

A

Mastocytosis.

This is a rare disorder characterized by proliferation of mast cells in the skin, bone marrow,
liver, spleen, lymph nodes, and small bowel. Histamine released from mast cells is responsible
for the associated symptoms of episodic flushing, pruritis, hypotension, and diarrhoea. The
serum tryptase level is also elevated in mastocytosis.

Whilst the other conditions mentioned could also cause diffuse thickening of small bowel folds,
the associated clinical laboratory findings and osteosclerosis are diagnostic of mastocytosis.

142
Q

A 75-year-old man presents with bone pain. Investigations reveal anaemia, renal impairment,
hypercalcaemia, proteinuria, and a monoclonal gammopathy. He undergoes radiological
investigation. Which of the following is most correct in relation to the radiological features of
the disease process? [B1 Q24]

A. 25% bone destruction must occur on plain film before a lesion will be apparent on plain
film.
B. 75% of patients will have positive radiographic findings on plain film.
C. MRI typically reveals general hypo intensity of bone marrow on T2WI sequences.
D. MRI typically reveals general hyperintensity of bone marrow on T1WI sequences.
E. The use of PET is inappropriate for imaging recurrent disease.

A

75% of patients will have positive radiographic findings on plain film

The clinical vignette alludes to a diagnosis of multiple myeloma (MM) and 50% bone
destruction must occur before a myelomatous lesion will be visible on plain film. Four distinct forms of bony involvement have been described in MM: (1) plasmacytoma, a solitary lytic
lesion that predominately affects the spine, pelvis, skull, ribs, sternum, and proximal
appendages, (2) diffuse skeletal involvement (myelomatosis), which classically manifests as
osteolytic lesions with discrete margins and uniform size, (3) diffuse skeletal osteopenia,
without well-defined lytic lesions, which predominately involves the spine (multiple
compression fractures may be seen with this pattern), (4) sclerosing myeloma, which results in
sclerotic bony lesions, and is associated with POEMS syndrome (polyneuropathy,
organomegally, endocrinopathy, monoclonal gammopathy and skin changes).

As well as typically producing general hyperintensity on T2WI and hypointensity on T1WI in
diffuse myelomatous involvement, plasmacytomas may produce focal areas of hypointensity
on T1WI/hyperintensity on T2WI. STIR imaging allows better assessment of marrow
involvement; fat-suppressed post contrast studies can demonstrate enhancement in focal or
diffuse disease. These findings are not specific for MM and may be seen in spinal metastases.
However, MM is suspected whenever MR images depict an expansile focal mass, multiple
focal masses in the axial skeleton, diffuse marrow involvement, particularly at known sites of
normal hematopoiesis, or multiple compression fractures in a patient with no known primary
malignancy. 99m Tc-based bone scanning under-appreciates the extent of disease. FDG-PET has
been used to study relapsing patients in whom recurrent disease is not easily detectable with
routine imaging. PET, in this instance, has been found to aid in the detection of unsuspected
sites of medullary and extramedullary disease.

143
Q

A 55-year-old female with a history of pulmonary sarcoidosis presents with pain in both hands.
Which of the following findings on plain radiograph of the hands is atypical of skeletal
sarcoidosis? [B1 Q36]

A. Cyst-like radiolucencies.
B. Joint space narrowing.
C. Bone erosions.
D. Subcutaneous soft tissue nodules/mass.
E. Lace-like pattern of bone destruction.

A

Joint space narrowing.

Around 5–10% of patients with sarcoidosis demonstrate skeletal involvement. The phalanges
in the hands and feet are most affected.

Joint space narrowing is unusual in sarcoidosis, unless neuropathic changes develop.

Typical radiographic changes include:

Cyst-like radiolucencies
‘Lace-like’ pattern of bone destruction
Bone erosions
Subcutaneous soft-tissue mass.

144
Q

A 76-year-old man presents with hip and pelvic pain. He has a history of renal cell carcinoma
treated by radiofrequency ablation and has been treated on multiple occasions with heparin for
thromboembolic disease. Plain films are non-contributory but a 99m Tc bone scan reveals
increased thoracic kyphosis and increased uptake in the body and bilateral alae of the sacrum
in an H configuration. What is the most likely diagnosis? [B1 Q49]

A. Brown tumour.
B. Multiple myeloma.
C. Metastasis from renal cell carcinoma.
D. Chordoma.
E. Insufficiency fractures.

A

Insufficiency fractures.

This patient has developed osteoporosis due to heparin administration (and with age). This has
resulted in thoracic kyphosis and the ‘H’ sign of increased uptake within the sacral body and
alae, which is classical of insufficiency fractures of the sacrum. Often there will have been
relatively minor trauma that will not be reported by the patient, and there may be associated
pubic rami fractures. Radiotherapy to the area (e.g., in gynaecological malignancy) is another
predisposing factor.

Multiple myeloma, metastases from renal cell carcinoma, and chordoma are typically
osteolytic and result in osteopenia at isotope bone scan (IBS), although the investigation has
poor sensitivity for myeloma. Brown tumours do cause increased uptake on IBS, but we are
not given a history of renal failure or hyperparathyroidism to explain their presence. There is
no history given to suggest infection.

Bone metastases which cause an increased uptake on IBS are breast, prostate, lymphoma,
pulmonary carcinoid, mucinous GI, and bladder tumours. Renal cell carcinoma, thyroid, and
melanoma typically cause photopoenia.

145
Q

A 60-year-old woman with recent history of falls presents with bilateral hip and low back pain.
MRI of the pelvis shows bilateral sacral ala fractures and parasymphysial pubic fractures with
marginal sclerosis. There is increased tracer uptake on bone scan of these regions. The most
likely diagnosis is? [B5 Q23]

(a) Multiple myeloma with pathological fractures
(b) Insufficiency fractures
(c) Traumatic fractures
(d) Metastatic disease with unknown primary
(e) Lymphoma

A

Insufficiency fractures

Presence of bilateral sacral ala fractures with pubic fractures in this clinical context is virtually
diagnostic of insufficiency fractures. Presence of marginal sclerosis at the fractures suggests
chronic changes

146
Q

A 45-year-old female undergoes aggressive chemotherapy for bone metastases followed by
bone marrow transplantation. Which of the following findings on MRI indicates recurrent
metastatic disease instead of rebound hematopoietic marrow? [B1 Q51]

A. Intermediate signal on T1WI.
B. High signal on T2WI.
C. Loss of signal on out-of-phase GE images.
D. High signal on STIR images.
E. Increased conspicuity on prolonged time-to-echo (TE) images.

A

Increased conspicuity on prolonged TE images.

Distinguishing recurrent metastases from rebound hematopoietic marrow is difficult on
standard T1WI and T2WI sequences because both have intermediate signal intensity on T1WI
and high signal intensity on T2WI. They may also occur in same anatomic regions.

Out-of-phase GE imaging is useful in differentiating the two. Most neoplastic processes replace
the marrow elements such as fat, osseous trabeculae, and hematopoietic elements, but
hyperplastic red marrow does not. Out-of-phase imaging allows detection of intralesional fat
by demonstrating a drop in signal intensity compared to in-phase images.

Lengthening of echo results in loss of signal of rebound red marrow due to T2* dephasing,
while the water-laden metastatic foci become more conspicuous.

147
Q

A 45-year-old female is being investigated. She has a history of connective tissue disease. You
are reviewing her imaging and trying to decide which connective tissue disease she has. Her
hand x-rays reveal distal tuft resorption with cutaneous calcification. She also has erosion of
the distal IP joints in the hands and the first carpometacarpal (CMC) joint. She has ulnar
deviation deformity to the MCP joints in both hands on the Norgaard views, which corrects on
the antero-posterior (AP) views. She also has an MRI scan of the pelvis which shows uniform
high T2WI signal in the gluteal muscles bilaterally. From the list of connective tissues diseases
below, select the one paired with a feature that is atypical for the disease but present in this
patient? [B1 Q56]

A. Systemic sclerosis and high signal changes in muscles
B. SLE and deforming arthropathy
C. Systemic sclerosis and acro-osteolysis
D. Polymyositis and erosive arthropathy
E. Polymyositis and soft tissue calcification

A

Polymyositis and erosive arthropathy.

There is a lot of overlap in the features of connective tissue diseases, with several patients
labelled as mixed-connective tissue disease due to this. There are features of these conditions
that can help differentiate them. SLE is a great mimic and can manifest in a myriad of ways.
Erosive disease is not typically seen in SLE, but SLE does classically give a reducible
deforming arthropathy of the hands, which is most pronounced on Norgaard views, but can
appear completely normal on PA hand views. Avascular necrosis (AVN) and deforming
arthropathy are not typically seen in systemic sclerosis. Acro-osteolysis and soft tissue
calcifications, especially in the fingertip pulps, are classical features of this disease.
Polymyositis classically gives high T2WI signal in muscles due to myositis. It does not
typically give erosions.

148
Q

A patient is referred from the dialysis unit with a history of joint and muscular pain. They
complain of bilateral hand pain and hip pain. The plain films of both hands show a loss of
distinction of the radial aspect of the phalanges of the index and middle fingers. There is an
area of para-articular soft tissue calcification noted adjacent to the middle finger metacarpal of
the right hand. The pelvic x-ray is distinctly abnormal. There is a large expansile lucent lesion
in the right iliac bone, which has a narrow zone of transition and no evidence of internal matrix. The bony definition of the rest of the pelvic bone reveals a coarsened trabecular pattern, but no
evidence of expansion of the bones. There are multiple small linear lucencies noted along the
medial aspect of the femurs bilaterally, which demonstrate a periosteal reaction. What
condition do you think this patient has? [B1 Q61]

A. Primary hyperparathyroidism.
B. Secondary hyperparathyroidism.
C. Paget’s disease.
D. Osteomalacia.
E. Renal osteodystrophy.

A

Renal osteodystrophy.

The first important observation to note is the referral route. A patient from the dialysis unit is
going to have renal failure, thereby excluding tertiary hyperparathyroidism and making
primary less likely. This leaves Paget’s disease, osteomalacia, and renal osteodystrophy. The
patient clearly has features of both hyperparathyroidism (subperiosteal resorption, brown
tumour in iliac bone) and osteomalacia (Looser’s zones), giving the diagnosis of renal
osteodystrophy, which has features of both. Another feature of renal osteodystrophy is the soft
tissue calcification noted in the hand. The Looser’s zones are small stress fractures on the load-
bearing aspect of a bone, caused by osteomalacia. This contrasts with the stress fractures seen
on the tensile aspect of bones (i.e., lateral aspect of femur) seen in Paget’s disease and fibrous
dysplasia. Whilst Brown tumours are more closely associated with primary
hyperparathyroidism, the majority occur in secondary hyperparathyroidism as this disease is
much more prevalent.

149
Q

A 62-year-old male with a known diagnosis of bronchogenic carcinoma presents with pain and
swelling of his wrists. What radiographic features are consistent with hypertrophic pulmonary
osteoarthropathy? [B1 Q63]

A. Metaphyseal lamellar periosteal reaction.
B. Irregular epiphyseal periosteal proliferation.
C. Asymmetrical, thick ‘feathery’ periosteal reaction.
D. Cortical thickening and trabecular coarsening.
E. Symmetrical, solid periosteal new bone formation.

A

Metaphyseal lamellar periosteal reaction.

Hypertrophic pulmonary osteoarthropathy is a paraneoplastic syndrome secondary to the
release of vasodilators. It typically causes burning pain and swelling, with the ankles and wrists
being most affected. Pulmonary causes include bronchogenic carcinoma, mesothelioma, and
pleural fibroma. Radiographs demonstrate cortical thickening and lamellar periosteal
proliferation in a dia-metaphyseal location. Bone scintigraphy will demonstrate patchy linear
increased uptake along the cortical margins

150
Q

Option B describes pachydermoperiostosis, a self-limited condition in adolescents. Option C
is typical of thyroid acropachy. Cortical thickening and trabecular coarsening is a feature of
Paget’s disease and symmetrical, solid periosteal new bone formation is described in
hypervitaminosis A.

A 40-year-old chronic smoker with recently diagnosed bronchogenic carcinoma presents with
bilateral leg pains. A plain radiograph shows lamellar periosteal reaction in bilateral tibia. A
bone scan demonstrates diffusely increased uptake along the cortical margins of the tibial
diaphysis. he most likely diagnosis is? [B5 Q14]

(a) Bilateral tibial metastases
(b) Osteomyelitis
(c) Chronic venous stasis
(d) Hypertrophic osteoarthropathy
(e) Acromegaly

A

Hypertrophic osteoarthropathy

This is seen in multiple conditions (e.g. malignant tumours of the lung, some benign lesions,
chronic chest infections). The condition manifests as cortical thickening and lamellar periosteal
reaction in the diametaphyseal regions of the long bones. Bone scanning shows symmetrical
uptake along the cortical margins of the diaphysis and metaphysis of tubular bones.

151
Q

A 22-year-old patient presents to casualty with a reduced GCS and hypotension. He is visiting
the UK from abroad and fellow backpackers in a local youth hostel state that he was
complaining of abdominal pain earlier that day. A CT abdomen reveals sclerosis in both
femoral heads and H-shaped vertebrae. The spleen is small and calcified. What is the patient’s
most likely underlying diagnosis? [B1 Q67]

A. Scheuermann’s disease.
B. Hereditary spherocytosis.
C. Gaucher disease.
D. Sickle-cell disease.
E. Primary bone lymphoma.

A

Sickle-cell disease.

Gaucher disease and SCD can both cause H-shaped vertebrae and avascular necrosis of the
humeral heads, but Gaucher disease causes splenomegaly, whereas by adulthood SCD will
usually have caused splenic infarction, resulting in a small, calcified spleen. Films illustrating
the complications of these diseases are beloved by examiners in the 2B exam: remember to
look for the mediastinal mass (extramedullary haematopoiesis) and AVN of the proximal
humeri on the chest film.

Other musculoskeletal manifestations of SCD include osteomyelitis (particularly salmonella
species), septic arthritis, and medullary bone infarcts. Infarction in SCD is common throughout
the body and is responsible for the acute pain crisis. Infarction can occur in the liver, spleen,
and kidneys, and can result in stroke.

Scheuermann’s disease is osteochondrosis of the apophyses of the thoracic vertebrae and
results in end-plate irregularity, Schmorl’s nodes, loss of disc space, and kyphosis. True H-
shaped vertebrae are not a feature. Hereditary spherocytosis is an autosomal dominant condition. It produces splenomegaly and,
as with other haematological conditions, can result in widening of the diploic spaces of the
skull.

152
Q

A 60-year-old woman presents to her GP with renal colic and hypercalcaemia. She has the
following findings on plain film: subperiosteal bone resorption of the proximal phalanges of
the hands, chondrocalcinosis of the articular cartilage at the knee joints, and a well-defined
lytic lesion in the body of the mandible. The most likely unifying diagnosis is: [B2 Q6]

a. Parathyroid adenoma
b. Parathyroid carcinoma
c. Renal osteodystrophy
d. Osteomalacia
e. Myeloma

A

Parathyroid adenoma

Parathyroid adenoma would be the most likely cause of primary hyperparathyroidism.
Parathyroid carcinoma would produce a similar radiographic picture but is much less common.
Brown tumours are seen in both primary and secondary hyperparathyroidism and are most
common in the mandible, ribs, and pelvis; they have a variable appearance on MRI and may
simulate primary or secondary neoplasms.

153
Q

In a 26-year-old woman with sickle cell disease, which one of the following would not be
considered a typical musculoskeletal manifestation of the disease? [B2 Q16]

a. Osteopenia and trabecular thinning
b. ‘Bone within bone’ appearance
c. Avascular necrosis of the femoral head
d. Posterior vertebral scalloping
e. Fish deformity of the vertebrae

A

Posterior vertebral scalloping

Posterior vertebral scalloping is not a feature. The remainder are all classic features of sickle
cell anaemia, along with ‘hair-on-end’ appearance of the skull due to coarse granular
osteoporosis and widening of the diploe. Osteomyelitis is a feature and is due to salmonella in
over 50% of cases.

154
Q

A 74-year-old woman with back pain presents to her GP. Initial plain radiographs of her spine
show multiple sclerotic metastatic lesions. The most likely primary tumour would be: [B2 Q22]

a. Renal cell carcinoma
b. Melanoma
c. Bronchial carcinoid
d. Bladder
e. Colorectal carcinoma

A

Bronchial carcinoid

The most likely from the above list is a bronchial carcinoid. In men the most likely cause would
be prostate. All the other conditions are more likely to produce lytic metastases than sclerotic.

155
Q

An elderly gentleman complaining of generalised aching in his lower limbs is shown to have
bilateral distal tibial periostitis. There is no underlying bone lesion identified. Which of the
following would be the most likely explanation? [B2 Q23]

a. Arterial insufficiency
b. Thyroid acropachy
c. Trauma
d. Pachydermoperiostosis
e. Hypertrophic pulmonary osteoarthropathy

A

Hypertrophic pulmonary osteoarthropathy

The most likely explanation is hypertrophic osteoarthropathy. Thyroid acropachy changes
usually occur in the upper limb. Venous stasis is a cause of periostitis rather than arterial
insufficiency. Trauma would be unlikely to be bilateral unless there was a specific history.
Pachy-dermo-periostitis is the idiopathic form of hypertrophic osteoarthropathy, it usually
presents around adolescence and is usually associated with clubbing.

156
Q

A 70-year-old male presents with increasing pain in his right hip over the past month. There is
no specific history of trauma. A plain radiograph demonstrates the presence of an incomplete
fracture of the femoral neck arising from the lateral (convex) side. What is the most likely
underlying abnormality of the femoral neck? [B2 Q29]

a. Osteomalacia
b. Metastasis
c. Osteoid osteoma
d. Infection
e. Paget’s disease

A

Paget’s disease

Incremental fractures (banana fracture) along the convex side of the bone are classically
associated with Paget’s disease. These most commonly occur in the femur where they cause
lateral bowing, and the tibia where they cause anterior bowing. Compression fractures of the
vertebrae are also associated with Paget’s.

157
Q

An elderly patient with history of urinary frequency and dribbling presents with right hip pain.
A radiograph of pelvis shows there is marked thickening of the iliopectineal line with
acetabular protrusion, coarsening of the trabecular pattern and increased sclerosis in the entire
right hemipelvis. The left hemipelvis appears normal. The most likely diagnosis is? [B5 Q15]

(a) Sclerotic metastases from prostate carcinoma
(b) Paget’s disease
(c) Lymphoma
(d) Normal variant
(e) Fluorosis

A

Paget’s disease

Asymmetrical cortical sclerosis and trabecular thickening is seen in Paget’s disease.

Metastatic disease of the prostate is unlike to affect only a hemipelvis and does not directly
cause acetabular protrusion. Fluorosis causes generalised sclerosis of the bones in the body

158
Q

A 75-year-old woman presents with symptoms of headache, right leg pain and raised serum
alkaline phosphatase. Plain radiography of the right leg shows cortical thickening of the
proximal tibia with coarse trabeculations and bowing. A bone scan demonstrates intense tracer
uptake in the calvarium and the right tibia. The most likely diagnosis is? [B5 Q22]

(a) Multiple myeloma
(b) Paget’s disease
(c) Secondary hyperparathyroidism
(d) Hypertrophic pulmonary osteoarthropathy
(e) Myelofibrosis

A

Paget’s disease

Paget’s disease can be monostotic or polyostotic and demonstrates intense tracer uptake on
bone scan. The area of uptake usually conforms well to the area of bone that is distorted or
expanded. The most common bones involved are the pelvis, spine, skull, femur, scapula, tibia,
and humerus. Many patients are evaluated after finding increased serum alkaline phosphatase.

159
Q

Considering the radiological features of thalassemia of the skull: [B3 Q4]

A. Dipolic space widening occurs first in the occipital region
B. Thickening of the outer table and thinning of the inner table
C. Hair-on-end appearance is common
D. There is reduced pneumatisation of the air spaces within the skull
E. Marrow hyperplasia in the maxilla causes medial displacement of the orbits and ventral
displacement of the incisors

A

D.

Hair-on-end appearance is recognised but is relatively uncommon. Increased haematopoesis is
associated with thinning of the outer table and thickening of the inner table. Characteristic
facies are produced by lateral displacement of the orbits.

160
Q

During reporting of a series of GP plain film, a chest radiograph shows superior rib notching.
Which of the following is a cause of superior rather than inferior rib notching? [B3 Q8]

A. Aortic thrombosis
B. Subclavian obstruction
C. Pulmonary AVM
D. Rheumatoid arthritis
E. Superior Vena Cava (SVC) obstruction

A

D
Other causes of superior rib notching include SLE, scleroderma, Sjögren’s syndrome,
hyperparathyroidism, Marfan’s syndrome, and osteogenesis imperfecta. NF can cause superior
and inferior rib notching.

161
Q

A plain radiograph of an adult pelvis shows a widened pubic symphysis. Which of the
following is a cause of widening rather than fusion of the symphysis pubis? [B3 Q11]

A. Hyperparathyroidism
B. Osteoarthritis
C. Late ankylosing spondylitis
D. Alkaptonuria
E. Fluorosis

A

A Hyperparathyroidism is a cause of widening of the pubic symphysis. Other causes of
widening include pregnancy, trauma, osteitis pubis, osteolytic metastases, infection, early
ankylosing spondylitis, rheumatoid arthritis.

162
Q

Which of the following is a cause of acro-osteo-sclerosis rather than acro-osteolysis? [B3 Q13]

A. Psoriassis
B. Diabetes
C. Polyvinylchloride work
D. Hodgkin’s
E. Hyperparathyroidism

A

D

Acrosteolysis is a lytic destructive process involving the distal and middle phalanges with no
periosteal reaction.

163
Q

In a 50-year-old man with joint pains, involvement of which of the following indicates
ochranosis rather than calcium pyrophosphate dihydrate crystal deposition disease (CPDD)?
[B3 Q15]
A. Annulus fibrosis of lumbar intervertebral discs
B. Nucleus pulposis of the lumbar intervertebral discs
C. Sacroiliac joints
D. Triangular fibrocartilage in the distal radioulnar joints
E. Glenohumeral joint

.

A

B

CPDD never involves nucleus pulposis, unlike ochranosis

164
Q

A 37-year-old female with generalised bone pain and a chronic disease has plain films of the
thoracic spine showing diffuse sclerosis and band-like areas of sclerosis involving the upper
and lower endplates. Anterior erosions are noted in the vertebral bodies. A small lytic lesion is
seen in the pedicle. Which is the diagnosis? [B3 Q37]

A. Fluorosis
B. Osteopetrosis
C. Systemic mastocytosis
D. Renal osteodystrophy and secondary hyperparathyroidism
E. Myelofibrosis

A

D

A rugger-jersey spine is described with subligamentous resorption and lytic brown tumour

165
Q

A 68-year-old man has a pelvic radiograph showing an enlarged right iliac bone with mixed
lucency and areas of increased density. There are coarsened trabeculae and cortical thickening.
There is thickening of the right ileopectineal line. Intense uptake is seen in this region on bone
scan. He is known to have prostate cancer, currently on active surveillance. Prostate Specific
Antigen (PSA) is stable at 1.7. Which is the diagnosis? [B3 Q38]

A. Melorheostosis
B. Metastatic prostate cancer
C. Post-radiotherapy changes from treated bony metastases
D. Mixed phase Paget’s disease
E. Myelofibrosis

A

D

Lytic, mixed and sclerotic phases are recognised in Paget’s, in which the lytic and mixed types
show increased activity, and a variable pattern in the sclerotic type.

166
Q

Regarding malignant fibrous histiocytoma: [B3 Q41]

A. Central mineralisation is common
B. Occur more often in bone than in soft tissue
C. Is the main primary malignant tumour of fibrous origin affecting bone
D. Most cases arise in Paget’s disease
E. Reactive changes are common

A

C

Occur much more commonly in soft tissue than bone. 25% occur in pre-existing conditions
such as Paget’s. They usually present with insidious onset pain and swelling, and central
mineralisation and reactive changes are uncommon.

167
Q

Considering imaging features of plasmocytomas: [B3 Q 42]

A. Are commonly sclerotic
B. Arise in the cortex
C. Usually has a poorly defined margin
D. Peripheral lesions are common
E. A soap bubble appearance is common

A

E
Plasmocytomas arise from the medulla in sites of persistent red marrow, frequently producing
a soft tissue mass. Other imaging features of plasmocytomas include destructive lytic lesions
with well-defined margins, cortical thinning with expansion and apparent trabeculae or a soap
bubble appearance.

168
Q

A 28-year-old female with severe pain in both hips has a plain radiograph which demonstrates
marked periarticular loss of cancellous bone. Considering transient regional osteoporosis,
which of the following is most correct? [B3 Q43]

A. Radiographic changes occur before bone scan changes
B. Slowly developing osteoporosis
C. Usually resolves in 4-6 weeks
D. Joint space narrowing occurs late
E. Acetabulum is usually involved

A

D

MR shows marrow oedema of the femoral heads, but does not usually involve the acetabulum

169
Q

Which of the following favours primary rather than secondary hyperparathyroidism?

A. Brown tumour
B. Osteosclerosis
C. Rugger-jersey spine
D. Soft tissue calcification
E. Vascular calcification

A

Brown tumour

Brown tumours and chondrocalcinosis are features of primary hyperparathyroidism.

170
Q

As seen on radiographs of a paediatric skeleton, generalized appendicular findings of poor
mineralization of the epiphyseal centres, widening of the growth plate and cupping/fraying of
the metaphyses are all frequently associated with which condition? [B4 Q20]

a. osteogenesis imperfecta
b. scurvy
c. fibrous dysplasia
d. rickets
e. mucopolysaccharidosis

A

Rickets

Rickets (vitamin D deficiency) results in failure of normal bone mineralization (osteomalacia)
during bone growth and may have a dietary, cultural or metabolic cause. A widened growth
plate in a child is due to rickets until proven otherwise. Other features include bowing
deformity, metaphyseal cupping and/or fraying, poor epiphyseal mineralization, delayed
closure of the fontanelles, enlargement of the costochondral junction (rachitic rosary) and
craniotabes. Scurvy is a deficiency of vitamin C and is a primarily a disorder of collagen production, resulting in osteopenia with dense metaphyseal lines and a sclerotic rim around the
epiphysis.

171
Q

Plain radiographs of the knees are performed in a teenage girl with growth retardation and
painful, deformed lower limbs. Which radiographic finding would suggest a diagnosis of
scurvy rather than rickets? [B4 Q69]

a. pathological fractures
b. bowing deformity
c. widened growth plate
d. frayed metaphysis
e. sclerotic epiphyseal rim

A

Sclerotic epiphyseal rim

Rickets is a deficiency of vitamin D in a child that results in osteomalacia of the immature
skeleton. Scurvy is a deficiency of vitamin C and is a disorder of collagen synthesis that can
occur in children or adults. Pathological fractures may be seen in both conditions. Ground-
glass osteoporosis is characteristic of scurvy, with other features including a sclerotic line in
the metaphyseal zone of preparatory calcification (white line of Frnkel), a radiolucent zone
immediately to the diaphyseal side of the white line (Trummerfeld’s zone), corner fractures
(Parke’s corner sign) and a sclerotic ring around the epiphysis (Wimberger’s sign). In addition,
bleeding diathesis is seen in scurvy; therefore, subperiosteal haematoma and haemarthrosis are
also features

172
Q

Looser’s zones– transverse linear lucencies representing areas of poorly mineralized osteoid–
are seen with which underlying pathological process of bone? [B4 Q25]

a. fracture
b. osteomyelitis
c. osteoporosis
d. osteopetrosis
e. osteomalacia

A

Osteomalacia

Looser’s zones or Milkman’s pseudo-fractures are seen as linear lucencies in the bone due to
incomplete fractures that have nonmineralized osteoid deposited within them. The underlying
failure of bone to mineralize is termed ‘osteomalacia’ (which means bone softening) and is
most often due to vitamin D deficiency. Rickets is osteomalacia in an immature skeleton. The
most common conditions resulting in an osteomalacic process and inadequate bone
mineralization include renal osteodystrophy and vitamin D deficiency due to
malnutrition/malabsorption of vitamin D or phosphate.

173
Q

Osteoporosis circumscripta– well-defined geographic lytic lesions in the skull– represents the
early stages of which condition? [B4 Q28]

a. Paget’s disease
b. hyperparathyroidism
c. multiple myeloma
d. senile osteoporosis
e. sickle cell disease

A

Paget’s disease

Paget’s disease is a common progressive disorder of osteoclasts and osteoblasts resulting in
bone remodelling. It is usually polyostotic and asymmetrical and affects 10% of those aged
over 80. Osteoporosis circumscripta is seen in the initial phase of Paget’s disease, which is
characterized by an aggressive, predominantly lytic process with intense osteoclastic activity
causing bone resorption. Bone marrow is replaced by fibrous tissue with large vascular
channels. Geographic osteoporosis is seen in the skull and long bones, where the characteristic
feature is a flame-shaped radiolucency beginning in a subarticular location and progressing into
the diaphysis. The disease then progresses through a mixed phase to a quiescent inactive late
stage where bone turnover is decreased. The skull is involved in 29–65% of cases, most
commonly the anterior calvarium.

174
Q

A 35-year-old woman of African origin is admitted to accident and emergency with acute
abdominal and back pain with pyrexia. Radiographs of the chest, abdomen and lumbar spine
show rib thinning with notching, sclerosis of the right humeral head and biconcave, ‘fish-
shaped’, lumbar vertebral bodies. A subsequent radiograph of the skull reveals widening of the
diplo with hair-on-end striations. What is the most likely underlying condition? [B4 Q84]

a. neurofibromatosis
b. thalassaemia major
c. sickle cell disease
d. syphilis
e. Scheuermann’s disease

A

Sickle cell disease

Sickle cell disease is a haemoglobinopathy that results from the presence of abnormal b-globin
chains within haemoglobin, which may manifest as anaemia, infarction and superimposed
infection. It is much more prevalent in those of African–Caribbean origin. Over time, the
disease produces musculoskeletal abnormalities as a result of chronic anaemia, such as marrow
proliferation (which produces the characteristic changes in the skull), marrow reconversion and
extramedullary haematopoiesis. Other common skeletal complications include bone softening,
infection or infarction.

175
Q

Vertebral sclerosis confined to the upper and lower endplates with preservation of the
intervertebral disc space (‘rugger jersey spine’), is seen most with which underlying condition?
[B4 Q37]

a. osteoporosis
b. discitis
c. mucopolysaccharidosis
d. Paget’s disease
e. renal osteodystrophy

A

Renal osteodystrophy

The ‘rugger jersey spine’ appearance refers to sclerotic bands along the superior and inferior
endplates of the thoracic and lumbar vertebral bodies. These bands represent accumulation of
excess osteoid and result in a striped appearance of the vertebral bodies. Despite being poorly
mineralized, the accumulated osteoid appears opaque on plain radiographs because of its
increased volume compared with that of normal bone. The ‘rugger jersey spine’ is said to be
almost pathognomonic for the osteosclerosis seen with the secondary hyperparathyroidism of
chronic renal failure. Renal osteodystrophy is a term for the constellation of musculoskeletal
abnormalities occurring with chronic renal failure. Osteoporosis and Paget’s disease are more
likely to affect the whole of the vertebrae diffusely. Discitis usually causes a reduction in the
intervertebral disc space on radiographs, with indistinct endplates. The mucopolysaccharidoses
result in anterior vertebral body beaking rather than sclerosis.

176
Q

By the middle of the third decade, the adult pattern of haematopoietic and fatty marrow
distribution is established. Red marrow remains in all but which of the following locations?
[B4 Q83]

a. sternum
b. clavicles
c. ribs
d. proximal humeri
e. distal femora

A

Distal femora

MRI is superior to other imaging modalities in evaluating red (haematopoietic) and yellow
(fatty) bone marrow, because of its very high sensitivity for lipid, which is present in
significantly higher concentrations in yellow marrow than in red. At birth, haematopoietic
marrow is present throughout the entire skeleton. A predictable sequence of conversion of red
to yellow marrow begins distally in the hands and feet and migrates proximally (over a period
of 20 years) until red marrow remains only in the axial skeleton and the proximal humeral and
femoral metaphyses. Reversal of this process is called reconversion. Appreciation of the areas
of remaining red marrow in an adult is important, because malignant marrow infiltration can
have a similar appearance to haematopoietic marrow, with location being a major discriminator.

177
Q

On plain radiographs of the long bones or the spine, which of the following is not a recognized
cause of a ‘bone within a bone’ appearance? [B4 Q87]

a. infant physiology
b. sickle cell anaemia
c. nutritional disturbance
d. renal osteodystrophy
e. metastatic disease

A

Renal osteodystrophy

Bone within a bone’ is a term used to describe a radiographic appearance in which one bone
appears to arise within another. It can be a physiological finding in a neonate or infant due to
new bone formation. Pathological conditions that can cause the appearance include periosteal
new bone formation, cortical splitting, subcortical osteopenia, altered bone growth, impairment
of osteoclastic activity, altered bone metabolism, crystal deposition, and iatrogenic and technical radiological factors. It is not a feature of renal osteodystrophy but is seen in
hypervitaminosis D and in healing rickets

178
Q

Vertebral bone marrow oedema, seen as low signal on T1W and high signal on T2W MR
images, occurs typically in all but which of the following conditions? [B4 Q92]

a. degenerative disc disease
b. multiple myeloma
c. osteoporotic collapse
d. spondylolysis
e. ankylosing spondylitis

A

Multiple myeloma

Multiple myeloma is a malignant condition of plasma cells that commonly shows infiltration
of the bone marrow, best seen on MRI. Patterns of infiltration can be classified as focal, diffuse
or variegated. Although marrow infiltration returns similar signal to marrow oedema on T1W
and T2W images, infiltration will show diffuse enhancement following administration of
intravenous gadolinium. The pattern of infiltration also differs. Infiltration will be patchy and
randomly distributed throughout the vertebral bone. In contrast, bone oedema occurs adjacent
to its cause, being linear at the endplates in the case of degenerative disc disease, and linear
with a fracture line in osteoporotic collapse, in the pedicles adjacent to spondylolysis or at the
entheses in ankylosing spondylitis.

179
Q

An elderly man undergoes 99mTc-labelled diphosphonate bone scintigraphy. There is no
uptake of tracer in the soft tissues, urinary tract, or appendicular skeleton, but the axial skeleton
shows diffuse homogeneous tracer uptake. No focal lesions are seen. What is the most likely
cause of these appearances? [B4 Q93]

a. prostatic metastases
b. renal osteodystrophy
c. Paget’s disease
d. mastocytosis
e. myelofibrosis

A

Prostatic metastases

The resulting pattern following diffuse uptake of 99m Tc-labelled diphosphonates in the axial
skeleton, with little or no uptake of tracer in the soft tissues or urinary tract, is frequently
referred to as a super-scan. When there is little uptake in the limbs, the cause is most likely to
be diffuse metastases in the axial skeleton, most commonly prostatic or breast. Uptake in
metabolic bone disease is more uniform in appearance and extends into the distal appendicular
skeleton. Intense calvarial uptake disproportionate to that in the remainder of the skeleton may
also be seen. The most important factor is to recognize the scan as abnormal in the absence of
focal lesions. The lack of renal uptake (absent kidney sign) is a useful discriminator.

180
Q

Considering post-radiotherapy changes of soft tissue tumours on MRI: [B3 Q27]

A. Earliest radiation change is demonstrated by increased SI in marrow between 6-12 weeks
B. SI changes are due to replacement of marrow by fat
C. In most cases, complete replacement occurs in 12-14 weeks
D. Regeneration of normal marrow is common
E. Radiation field is usually poorly defined on MR

A

SI changes are due to replacement of marrow by fat

The earliest changes occur 1-6 weeks after therapy is initiated and is due to replacement of
marrow by fat. Complete replacement occurs within 6-8 weeks. Due to high radiation doses
used in soft tissue tumour treatment, regeneration of normal marrow is rare, but can occur in
young patients. The radiation field is usually well-defined.

181
Q

Considering soft tissue response to chemotherapy: [B3 Q28]

A. Neoadjuvant chemotherapy increases recurrence-free survival but not overall survival in
high grade soft tissue sarcoma
B. Chemotherapy may cause a substantial increase in tumour size initially
C. Tumour size is the most accurate predictor of biologic response to tumour treatment
D. Contrast enhancement limits interpretation of intralesional necrosis
E. Intralesional haemorrhage post-chemotherapy is very rare

A

B

Treatment induced necrosis is a more accurate predictor of tumour response than size. Tumour
size can indeed increase initially due to intralesional haemorrhage. The degree of necrosis is
best evaluated with gadolinium enhanced T1 fat suppressed sequences. Neoadjuvant
chemotherapy increases both recurrence-free and overall survival

182
Q

A 21-year-old man presents with right hip pain. He has a history of Ewing’s sarcoma of the
right hemi-pelvis when aged 11, which was treated with limb-sparing surgery and
chemoradiotherapy. Plain radiography shows well-defined regional sclerosis, and isotope bone
scan demonstrates a corresponding photopenic area. What is the most likely cause of his pain?
[B4 Q41]

a. recurrent Ewing’s sarcoma
b. heterotopic ossification
c. radiation necrosis
d. osteoarthritis
e. osteosarcoma

A

Radiation necrosis

Ewing’s sarcoma is a relatively common malignant bone marrow tumour related to, and sharing
a common chromosomal translocation with, peripheral neuroectodermal tumours. It is a very
aggressive tumour that is expressed in the radiological findings of permeative osteolysis, cortical erosion, periostitis and a soft-tissue mass. It principally affects the lower half of the
skeleton, with the most frequent location being meta diaphyseal in the femur, ilium, tibia,
humerus, fibula and ribs. Both radio- and chemotherapy are used in treatment. Sclerosis within
several months of treatment usually indicates bone healing or disease recurrence/persistence.
Radiation-induced osteonecrosis is mainly an effect on the osteoblasts and is dose dependent
(deterministic). It may be seen within the mandible within 1 year, but in other sites the latent
period is longer and can be several years.

183
Q

A middle-aged male patient who has previously undergone partial discectomy for
radiculopathy, has a lumbar spine MRI due to a recurrence of his symptoms. T1W images show
a low-signal area of tissue contiguous with the previously operated intervertebral disc and
impinging upon the adjacent exiting nerve root. Which single additional finding favours a
diagnosis of postoperative fibrosis over recurrent disc protrusion? [B4 Q43]

a. high signal on STIR sequence
b. enhancement with intravenous gadolinium
c. evolution at 6-month serial imaging
d. oedema in the surrounding bone
e. low signal on T2W images

A

Enhancement with intravenous gadolinium

In MRI of the spine in postoperative discectomy patients with recurrent or persistent
radiculopathy, a T1W sequence with intravenous gadolinium enhancement is added to
distinguish between postoperative epidural fibrosis (or scarring) and recurrent disc herniation.
Both can have similar, low-signal appearances on unenhanced T1W and T2W images, but
fibrosis will show enhancement with gadolinium whereas recurrent disc prolapse will not
enhance.
Difficulties arise where both conditions exist concurrently, and fibrosis that is not
causing nerve root irritation may also enhance. The importance of distinguishing between the
two is that surgical treatment is indicated for recurrent disc herniation but is of no value in
treating postoperative fibrosis, also known as failed back syndrome.

184
Q

The hypoperfusion complex, seen in patients who have suffered major blunt abdominal trauma,
includes all but which of the following radiological signs on contrast-enhanced CT? [B4 Q72]

a. hyperenhancement of the adrenal glands
b. hyperenhancement of the pancreas
c. hyperenhancement of the spleen
d. collapsed inferior vena cava
e. small aorta

A

Hyperenhancement of the spleen

The hypoperfusion complex is a marker of severe injury and is an important prognostic
indicator related to radiological signs on CT following blunt abdominal trauma. The features
are of hypovolaemia, with small arterial and collapsed venous vessels indicating reduced
circulating volume. Hyperenhancement of the kidneys, adrenal glands, pancreas and bowel
wall is seen, but the spleen may be small and hypodense. If injury to the vascular pedicle is not
present, nonenhancement of the spleen could be secondary to severe vasoconstriction and poor
perfusion.

185
Q

Which of the following locations is most often associated with post-traumatic osteolysis? [B4
Q60]

a. coronoid process of ulna
b. surgical neck of humerus
c. lateral clavicle
d. femoral neck
e. fibular head

A

Lateral clavicle

The lateral third of the clavicle is the most common location for posttraumatic osteolysis. It is
usually preceded by a severe injury to the shoulder, typically dislocation or subluxation of the
acromioclavicular joint. Changes may be evident radiographically after as little as 1 month. If
no bone loss was apparent on radiographs at the time of injury, the diagnosis is unequivocal.
However, if no such comparison can be made, then other

Causes of lateral clavicular osteolysis:
Rheumatoid arthritis
Scleroderma
Hyperparathyroidism
Post traumatic osteolysis

Other sites affected are the
pubic and ischial rami, distal portions of the radius or ulna, the carpus and femoral neck.
Widespread idiopathic osteolysis is termed Gorham’s or vanishing bone disease.

186
Q

A young woman attempts to commit suicide by jumping from a third-storey window, sustaining
a fall of 15 metres. In addition to bilateral lower limb and spinal fractures, she suffers a blunt
deceleration injury to the mediastinum. CT findings are of a large mediastinal haematoma and
a focal area of irregularity in the contour of the wall of the aorta, which appears otherwise
normal. Which segment of the thoracic aorta is most affected by tear or transection? [B4 Q79]

a. root
b. ascending
c. isthmus
d. arch
e. descending

A

Isthmus

Ninety per cent of traumatic thoracic aortic injuries occur at the aortic isthmus, just distal to
the origin of the left subclavian artery
. The isthmus is the section between the origin of the left
subclavian and the attachment of the ligamentum arteriosum and is about 1.5 cm in length in a
normal adult. The mechanism is usually rapid deceleration (but it can be due to direct trauma)
as in a fall from a height or a road traffic collision. The isthmus is thought to be particularly
vulnerable to the shearing forces that occur with deceleration compared with the descending
aorta, as it is relatively mobile and can be bent over the left bronchus main stem and left
pulmonary artery. A more recent theory is that this part of the aorta is particularly vulnerable
to being crushed by the surrounding bony thorax (manubrium, clavicle and first rib) at the point
of maximum deformation during high-energy injury. The ascending aorta is the site for injury
in only 5% of those who survive to reach hospital but is more prevalent in cadaveric studies
due to the high association of fatal cardiac injuries. The mechanism here is thought to be
torsional forces or a water hammer effect (a sudden increase in intrathoracic pressure).

187
Q

An 18-year-old motorcyclist is involved in an RTA in which he was dragged by the colliding
car. He is noted to have pain in his right shoulder and neck with associated paraesthesia. An
MRI is requested, suspecting brachial plexus injury. What finding is most suggestive of nerve
root avulsion? [B1 Q20]

A. Pseudomeningocoele.
B. Intradural nerve root enhancement.
C. Spinal cord T2WI hyperintensity.
D. T2WI hyperintensity within the paraspinal muscles.
E. Thickening of the brachial plexus.

A

Pseudo - meningocele.
Imaging in brachial plexus injury via CT myelography and/or MRI helps to determine whether
the injury is pre- or postganglionic, which has therapeutic implications.

Intradural nerve root enhancement suggests functional impairment of the nerve roots, despite
morphological continuity. This is not a common finding.

Signal intensity changes in the spinal cord are seen in only 20% of preganglionic injuries and
lack specificity. Traumatic pseudo meningocoele, although not pathognomonic, is the most
valuable sign of a preganglionic lesion.

T2WI signal intensity changes within the paraspinal muscles are observed in nerve root
avulsion, but this is less accurate than enhancement on T1WI post contrast. Abnormal
enhancement within the multifidus muscle is the most accurate of all paraspinal muscle findings
since it is innervated by a single nerve root.

Thickening of the brachial plexus, secondary to oedema and fibrosis, is seen in postganglionic
injury.

188
Q

A 17-year-old female is admitted with multiple penetrating injuries to her arms after shielding
her face from a nearby bomb blast while walking in the city centre. For which type of
penetrating foreign body is ultrasound most superior for detection? [B1 Q25]

A. Gravel.
B. Wood.
C. Plastic.
D. Windshield glass.
E. Bottle glass.

A

Wood.

Radiography is highly sensitive for foreign bodies considered radio-opaque. All glass material
is radio-opaque to some degree on radiographs and does not need to contain lead. The role for
ultrasound is limited to those foreign bodies that are radiolucent, such as wood and plastic.
Wood appears hyperechoic with marked posterior acoustic shadowing. Ultrasound can detect
wooden foreign bodies as small as 2.5 mm in length with 87% sensitivity and 97% specificity.
Plastic is also radiolucent, but less echogenic than other foreign bodies on ultrasound

189
Q

A young man is admitted to accident and emergency following an assault, during which he was
struck in the face with a heavy blunt object. Radiographs show multiple midface fractures.
Which supporting buttress of the face is disrupted in a Le Fort II fracture, but spared in a Le
Fort I fracture? [B4 Q89]

a. inferior lateral maxillary
b. inferior medial maxillary
c. superior medial maxillary
d. transverse maxillary
e. pterygomaxillary

A

Superior medial maxillary

Le Fort fractures involve separation of all or part of the maxilla from the skull base. For this to
occur, the posterior vertical maxillary (pterygomaxillary) buttress at the junction of the
posterior maxillary sinus with the pterygoid plates of the sphenoid must be disrupted. The
remaining facial buttresses are inspected to determine the class of Le Fort fracture. A Le Fort
I fracture involves the inferior portions of both the lateral and medial maxillary buttresses,
resulting in the maxillary arch floating free from the face. In a Le Fort II fracture, the inferior
lateral maxillary buttress is similarly injured, but, unlike type I, it is associated with fracture of
the superior portion of the medial maxillary buttresses, resulting in dissociation of the entire
maxilla from the skull base. A Le Fort III fracture results in the whole face floating free from
the skull with disruption of the superior portions of both the lateral and medial maxillary
buttresses and upper transverse maxillary buttress.

190
Q

A 28-year-old man is brought into the emergency department following an assault during
which he was stabbed in the left flank. He has a 1.3cm wound just below the left costal margin
in the mid-axillary line. No information regarding the knife has been obtained. His renal
function is within normal limits, and he has no contrast allergies. The optimal CT protocol for
scanning his abdomen would include the following contrast: [B2 Q24]

a. IV contrast only
b. Oral contrast and rectal contrast
c. IV contrast and oral contrast
d. Oral, rectal and IV contrast
e. IV contrast and rectal contrast

A

Oral, rectal and IV contrast

Oral, rectal and IV contrast A triple contrast technique has been advocated in penetrating
trauma where there may be concern regarding small bowel or colon trauma. If no oral or rectal
contrast are given, then a small bowel or colon injury can easily be missed.

191
Q

Which of the following local factors is not associated with an increased risk of fracture non-
union? [B4 Q70]

a. infection
b. fracture mobility
c. avascular fragments
d. impaction
e. open fracture

A

Impaction

Fracture sites that have a poor blood supply, either as a result of the original injury or due to
subsequent surgical treatment, may go on to develop atrophic non-union where the bone ends
become osteoporotic, thin and pointed (osteolysis) with no evidence of fracture healing. A
fracture site that is very mobile may develop hypertrophic non-union where there is attempted
healing denoted by excessive callus formation, but the fracture cleft persists. Open fractures
are often high energy, with soft-tissue damage and comminution of fracture fragments, and are
prone to infection, all of which predispose to non-union. Osteomyelitis in any fracture can
result in delayed, non- or malunion. Non-union in most skeletal locations should not be
diagnosed radiographically until 6 months have passed, particularly in the presence of
complicating factors.

192
Q

A 75-year-old man who is on warfarin for atrial fibrillation is involved in a high-speed road
traffic accident in which he sustains a head injury. He lost consciousness at the scene. On
arrival at the A&E department his GCS is 15. He has no other obvious injuries. According to
NICE guidelines, his management should include the following: [B2 Q25]

a. Skull radiograph
b. No immediate imaging but admission for regular neurological observations
c. CT head
d. Skull radiograph followed by CT head
e. MRI head

A

CT head

According to NICE guidance he should undergo CT head and the investigation should be
performed within the hour following referral. The fact he is anticoagulated, over 65 and
experienced a loss of consciousness would all be factors in warranting an urgent CT head. (Ref:
National Institute for Health and Clinical Excellence

193
Q

A young man sustains an obvious head injury during an assault, with clinically apparent, left
temporoparietal swelling and an underlying fracture on skull radiographs. The patient’s GCS
is initially normal but begins to deteriorate progressively after 4 hours of observation, and CT
of the brain is requested. Which finding other than a skull fracture would you expect to find on
CT to explain the patient’s condition? [B4 Q45]

a. diffuse axonal injury
b. haemorrhagic contusions
c. subarachnoid haemorrhage
d. subdural haematoma
e. extradural haematoma

A

Extradural haematoma

Extradural (or epidural) haematoma is the accumulation of blood in the potential space between
the inner table of the skull and the dura. Ninety per cent of cases are associated with a fracture
of the temporal bone which traverses and ruptures the middle meningeal artery (in 60–90%) or
vein. In children, vascular injury may occur here without a fracture. In half of cases, there is a
lucent interval between injury and the onset of deteriorating consciousness level, as opposed
to diffuse axonal injury where coma is immediate. CT appearances are of an extra-axial,
biconvex, high-attenuation collection. A subdural haematoma is caused by shearing forces on
small bridging veins and is not necessarily associated with a fracture of the calvarium, although
this may coexist. It differs from an extradural haematoma not in radiographic location but in
that it has a crescentic rather than a biconvex shape

194
Q

A20-year-old man is a restrained driver in a high-speed road traffic collision. On admission to
accident and emergency he undergoes CT of the brain for a reduced consciousness level.
Images show diffuse brain injury. Which of the following findings would support a diagnosis
of diffuse axonal injury rather than simple contusions? [B4 Q64]

a. corticomedullary petechial haematoma
b. anterior temporal petechial haematoma
c. baso-frontal petechial haematoma
d. intraventricular haemorrhage
e. brain oedema

A

Cortico-medullary petechial haematoma

Contusions are traumatic injuries to the cortical grey matter of the brain and make up
approximately half of primary intra-axial traumatic lesions. They are often multiple and
bilateral, with the most common locations being the inferior frontal lobes and temporal poles.
Diffuse axonal injury results from rotational shearing forces on cerebral white matter and
common locations are white matter-rich areas, such as the corticomedullary junction, centrum
semiovale, corpus callosum and cerebellum. In comparison to diffuse axonal injury, contusions
tend to be larger and more superficial, with a higher proportion being haemorrhagic due to the
increased vascularity of grey compared with white matter. Local or widespread oedema can be
seen in both conditions.

195
Q

A 24-year-old man suffers a short oblique fracture of his distal tibia from a direct blow during
a football game. He is treated with an intramedullary nail with a good reduction being achieved.
Fourteen days later the foot becomes very tender, red, and swollen but all haematological and
biochemical parameters remain normal. Plain radiographs show spotty osteoporosis and
subchondral erosions.
Which of the following is the most likely diagnosis? [B2 Q37]

a. Disuse osteoporosis
b. Charcot joints
c. Infection
d. Regional sympathetic dystrophy
e. Rheumatoid arthritis

A

Regional sympathetic dystrophy

This is the typical appearance, history, and imaging findings for regional sympathetic
dystrophy. This may occur following fractures or secondary to other pathologies such as
primary or secondary bone tumours. There is overactivity of the sympathetic nervous system
causing pain, swelling and hyperaemia with excessive bone resorption.
This is usually in a
periarticular distribution and may simulate other disease processes.

196
Q

A 65-year-old woman with chronic rheumatoid arthritis, had fracture of the lateral malleolus
which was treated by a cast immobilisation. Since removal of the plaster, the foot has been
swollen and painful on movements. Plain radiographs show that the fracture is united, but the
bones show diffuse osteopenia with endosteal scalloping and severe periarticular
demineralisation. Bone scan shows increased uptake on three phase scintigram. The most likely
cause of the patient’s symptoms is? [B5 Q7]

(a) Reflex sympathetic dystrophy
(b) Transient regional osteoporosis
(c) Myelomatosis
(d) Infection
(e) Disuse osteoporosis

A

Reflex sympathetic dystrophy

This is commonly associated with trauma. The condition causes hyperhidrosis skin changes
with excessive pitting oedema, sudomotor changes (hyperhidrosis and hypertrichosis), pain
and patchy osteopenia. Three-phase bone scan shows increased blood flow, increased blood
pool and increased periarticular uptake on delayed images.

197
Q

A 60-year-old man with history of diabetes, chronic renal failure and bilateral intermittent
claudication, had a left ankle injury 6 months ago. It was treated by open reduction and internal
fixation of the medial and lateral malleoli. He now complains of persistent pain and swelling
in the ankle and foot. Plain radiographs show marked osteopenia of the bones in the left ankle
and foot. Bone scan shows diffuse increased tracer uptake in the periarticular distribution in
left ankle and foot. The likely cause of the bone scan appearances is? [B5 Q21]

(a) Chronic ischemic feet
(b) Hypertrophic osteoarthropathy
(c) Secondary hyperparathyroidism
(d) Reflex sympathetic dystrophy
(e) Diabetic foot

A

Reflex sympathetic dystrophy

A history of trauma, periarticular uptake on bone scan and osteoporosis suggests reflex
sympathetic dystrophy.

All other conditions are likely to affect both feet.

198
Q

A young patient suffers a fractured femur and acetabulum in a road traffic collision and
undergoes intramedullary nailing and plate-and-screw internal fixation of the acetabulum. He
is well until 8 days postoperatively, when he develops acute shortness of breath and right-sided
chest pain. A chest radiograph shows only a small right-sided pleural effusion. What is the
most likely diagnosis? [B4 Q5]

a. fat embolism
b. bronchial pneumonia
c. pulmonary embolism
d. pneumothorax
e. hyperventilation due to pain

A

Pulmonary embolism

Pulmonary embolism is a common complication following immobility and major surgery,
particularly orthopaedic surgery of the pelvis. PE typically occurs 7–10 days post-surgery. Chest
radiograph findings can be normal but include small effusion, collapse or consolidation,
elevation of the hemidiaphragm, a prominent pulmonary artery and hyper-trans-radiance of the
affected side (Westermark sign). Fat embolism is preceded by long bone injury in 90% of cases
but usually occurs within 36 hours of the injury and is much less common than pulmonary
embolus from deep vein thrombosis even in the context of major trauma. Pneumonia and
pneumothorax do of course occur in postoperative patients, but it would be reasonable to expect associated findings on the chest film. Hyperventilation should be a diagnosis of exclusion once
other potentially serious causes have been excluded

199
Q

A 27-year-old man who attends A&E following an alleged assault is shown to have a left-sided
longitudinal temporal bone fracture. Which of the following is a correct association? [B2 Q43]

a. Facial nerve palsy in 50% of cases
b. Incudostapedial joint dislocation
c. Sensorineural hearing loss
d. Ophthalmoplegia
e. Rhinorrhoea

A

Incudo-stapedial joint dislocation

Longitudinal fractures of the temporal bone are more common than transverse fractures and
account for over 85% of temporal bone fractures. They are associated with otorrhea, conductive
hearing loss, pneumocephalus, herniation of the temporal lobe and incudo-stapedial dislocation.

Transverse fractures are associated with sensorineural hearing loss, and a higher percentage of
facial nerve palsies.

200
Q

Regarding scaphoid fractures, which of the following statements is correct? [B2 Q48]

a. 80% of scaphoid fractures occur at the waist
b. Approximately 5% of scaphoid fractures are complicated by avascular necrosis
c. Injury is typically due to hyperextension
d. Up to 60% of scaphoid fractures cannot be seen on initial radiograph
e. The specificity of CT in diagnosing scaphoid fractures is 60–70%

A

Injury is typically due to hyperextension

The scaphoid bone is the most fractured carpal bone, and the mechanism is usually a fall onto
the outstretched hand – ie. hyperextension of the wrist. The reported sensitivities and
specificities of CT are 89–97% and 85–100%, respectively. The high negative predictive value
of CT (96.8–99%) makes it very useful for ruling out a fracture. Scaphoid fractures are missed
on initial radiographs in up to 30% of cases

201
Q

A 24-year-old man is involved in a road traffic accident. On arrival in A&E he is
haemodynamically unstable and there is concern regarding pelvic fracture and associated
active extravasation. On multidetector CT, which of the following features is more suggestive
of pseudoaneurysm than active extravasation? [B2 Q49]

a. Ill-defined area of high attenaution on arterial phase imaging
b. Presence of a haemoperitoneum
c. Washout of the high-attenuation area on delayed imaging
d. Layering appearance on delayed imaging
e. Haemodynamically unstable patient

A

Washout of the high-attenuation area on delayed imaging
Washout of the high-attenuation area is one of the features of a pseudoaneurysm. A
pseudoaneurysm is likely to be adjacent to a vessel and whilst there will be a relatively well-
defined area of high attenuation on arterial phase imaging, this will diminish in intensity on
five-minute delayed imaging. In contrast to this, an area of active extravasation will often
appear as a jet of high attenuation which continues to collect and enlarge on delayed phase
imaging.

202
Q

A 27-year-old woman is brought into A&E following a road traffic accident in which she was
knocked down by a car. On arrival she has a GCS of 15 but is haemodynamically unstable and
on examination she has abdominal bruising. The A&E consultant has performed a FAST
(focused assessment with sonography in trauma) scan in resus and cannot see evidence of free
fluid. What is the approximate minimal detectable fluid volume by FAST scanning? [B2 Q50]

a. 10ml
b. 50ml
c. 100ml
d. 200ml
e. 500ml

A

200 ml

The approximate minimal detectable fluid volume is 200 ml. The distribution of free fluid will
be determined by both anatomical and physiologic factors and therefore the sensitivity of the
scan will depend upon the areas scanned. Ultrasound is often used in conjunction with
multidetector CT, particularly in the management of patients who have been involved in trauma.

203
Q

Which of the following is the most common type of Salter–Harris fracture, accounting for over
70% of growth-plate fractures of the immature skeleton? [B4 Q34]

a. type I
b. type II
c. type III
d. type IV
e. type V

A

Type II

The Salter–Harris classification originally described five types of growth plate injury. Type I
(6–8%) is slip of the epiphysis due to shearing forces, and the fracture line is a cleavage through
the growth plate only. Type II is by far the most common (75% of injuries) and is also a
shearing injury. The fracture line involves the physis and extends proximally into the
metaphysis separating a triangular fragment. Type III (6–8%) occurs in partially closed growth
plates, involves the physis and extends distally into the epiphysis, involving the articular
surface. Type IV (10–12%) is a fracture that begins in the metaphysis, crosses the physis and
also involves the epiphysis. Type V (1%) is a crush injury. The classification indicates
progressively poorer prognosis about future growth disturbance. There have been subsequent
additions to the classification.

204
Q

A 40-year-old man falls down the stairs and remains unconscious for several hours. On
admission to hospital, he is found to have bilateral upper limb weakness, patchy sensory loss, full power in the lower limbs and a normal level of consciousness. Plain radiographs of the
cervical spine and CT of the brain are normal. On MRI of the cervical spine, there is a small
area of oedema identified within the cord. Clinical symptoms persist for 4 days following injury.
What is the most likely diagnosis? [B4 Q36]

a. central cord syndrome
b. anterior cord syndrome
c. SCIWORA (spinal cord injury without radiological abnormality)
d. spinal shock
e. Brown-Se´quard syndrome

A

Central cord syndrome

In trauma, an incomplete spinal cord injury is one in which there is any degree of sparing of
motor or sensory function distal to the site of injury, whereas complete cord injury results in
complete lack of neurological function distal to the injury. The diagnosis can be made only in
the absence of spinal shock, a transient spinal cord concussion. Central cord syndrome is the
most common incomplete injury and is associated with hyperextension injury in middle-aged
patients; injury to centrally located grey matter in the cord causes a greater motor neurological
deficit in the upper than in the lower extremities. Sensory involvement can be variable, and
bowel and bladder function may be affected. Anterior cord syndrome, caused by anterior spinal
vascular insufficiency, causes complete motor paralysis with sparing of the posterior columns.
SCIWORA is seen in children, when the elastic cervical spine deforms sufficiently to cause
cord injury but without any radiological findings. Brown-Sequard syndrome results from hemi
transection and causes ipsilateral muscle paralysis and contralateral hyperaesthesia to pain and
temperature

205
Q

Following major trauma, which of the following fractures of the thoracic skeleton is most likely
to indicate a significant injury to the underlying intrathoracic viscera? [B4 Q44]

a. glenoid
b. scapular blade
c. clavicle
d. first rib
e. sternum

A

First rib

First rib fracture is considered a harbinger of major trauma, with approximately two-thirds of
fractures being associated with major chest injury and carrying a significant mortality. The
anatomy of the first rib is such that it is protected from the minor insults that often break other
ribs, and fracture of the first rib usually indicates violent blunt trauma to the thorax. Associated
local injuries include damage to the brachial plexus, major vascular structures and the
underlying lung and heart. There is also an association with significant abdominal injury, but
the major cause of death in patients with a first rib fracture is an associated head injury. It is
rare for a first rib fracture to be an isolated finding

206
Q

A young man is assaulted and attends accident and emergency with a painful left mandible and
inability to open and close his jaw without pain. Radiographs show a simple linear fracture
through the left body in the para-symphyseal region. A second fracture is most likely to be seen
at which of the following sites? [B4 Q39]

a. ipsilateral condylar neck
b. ipsilateral angle
c. symphysis menti
d. contralateral body
e. contralateral condylar neck

A

Contralateral condylar neck

The mandible is best considered as a closed ring, and as such approximately half of all
mandibular fractures are bilateral and multiple. The majority occur at the angle, and a
significant portion occur at the condylar neck, a common pattern of injury being an ipsilateral
body fracture from a direct blow, with a contralateral angle or condylar neck fracture due to
transmitted rotation force compressing that side. Fractures of the condylar neck have a limiting
effect on the opening and closing of the jaw and can be missed radiographically. Fractures in
the midline are also subtle and account for a significant minority. A flail mandible occurs when
the anterior support for the tongue is lost due to a bilateral fracture. This injury carries the risk
of the tongue prolapsing posteriorly and occluding the airway.

207
Q

Pelvic fractures - Actual exam question!!

An elderly woman falls down the stairs and suffers a Malgaigne fracture of the pelvis and a 1%
degloving injury to the left forearm. Due to significant medical co-morbidity, the decision is
made not to treat with surgery. The patient dies overnight on the ward. What is the most likely
mechanism of death? [B4 Q63]

a. pulmonary embolism
b. fat embolism
c. septicaemia
d. myocardial infarction
e. intra-abdominal haemorrhage

A

Intra-abdominal haemorrhage

A Malgaigne fracture of the pelvis is a fracture of the pubic rami and an ipsilateral
sacroiliac joint fracture
and occurs due to high-energy blunt trauma. It
represents complete disruption of the pelvic ring and therefore an unstable fracture. In such
fractures, distortion and disruption of the pelvic soft tissues and vascular injury involving the
rich blood supply in the pelvis will not be tamponade by the bony ring, as the pelvis will expand
to accommodate ever-increasing haematoma. Mortality rate from major pelvic trauma is 10%;
other common causes of death include multiorgan failure and sepsis, the latter expected to take
several days to evolve.

208
Q

A 75-year-old man has a cemented right total hip replacement. On routine follow-up imaging
he is noted to have a progressive well-delineated, rounded, focal area of lucency at the cement
bone interface adjacent to the tip of the femoral stem. Which of the following given reasons is
the most appropriate for this progressive lucency? [B1 Q18]

A. Aggressive granulomatous disease.
B. Primary loosening.
C. Cement fracture.
D. Normal finding.
E. Metal bead shedding.

A

Aggressive granulomatous disease.

Well-delineated, rounded, focal areas of lucency at the cement bone interface, which are
progressive, are suggestive of either infection or aggressive granulomatous disease. It can occur
with both cemented and non-cemented components. Its origin is thought to be multifactorial.
Metal, cement, or polyethylene fragments may penetrate the cement bone interface and induce
a focal inflammatory foreign-body reaction, leading to osteolysis.

Primary loosening usually manifests as a wide (>2 mm) radiolucent zone at the cement–bone
or metal–bone interface or a progressive radiolucent zone at the metal–cement interface. The
radiolucent zones are not typically rounded.

Cement fractures are thin linear lucent areas within the cement. They may be asymptomatic
but are important to identify as they may lead to component failure.

Metal bead shedding is defined as opaque micro-fragments separated from the porous coated
femoral stem. Metal beads can be seen on immediate postoperative radiographs, because of the
stem insertion. Bead shedding might later occur with loose non-cemented components,
reflecting micro-motion of the stem. These metal beads are seen in the soft tissue adjacent to
the hip replacement and their increase in number on follow-up indicates loosening.

209
Q

A 55-year-old man is noted on a plain x-ray of pelvis to have a right hip prosthesis. There is a
cemented acetabular component present with an uncemented stem. Of the following hip
arthroplasties, which is the most likely procedure that he has undergone? [B1 Q62]

A. Unipolar hemiarthroplasty.
B. Bipolar hemiarthroplasty.
C. Hip resurfacing.
D. Hybrid total hip replacement.
E. Reverse hybrid total hip replacement.

A

Reverse hybrid total hip replacement.
A combination of a cemented acetabular cup and an uncemented femoral stem is known as a
reverse hybrid hip total hip replacement. A hybrid total hip replacement is a combination of a
cemented femoral stem and an uncemented acetabular cup.

A unipolar hemiarthroplasty comprises a combination of a femoral component articulating
directly with the native cartilage surface of the acetabulum.

A bipolar hemiarthroplasty comprises a combination of a femoral component articulating with
a cup inserted into the native acetabulum without fixation. This cup is usually made of
polyethylene with a metal backing and can normally move within the native acetabular cavity
because of the absence of fixation.

Hip resurfacing consists of replacing the surface of the femoral head by a metallic ‘cap’ without
removing the femoral neck or instrumenting the femoral diaphysis. The cap used on the femoral
head is virtually the same size as the natural head and articulates with an acetabular prosthetic
cup, usually made of metal. This type of procedure is favoured in younger active patients and
may allow for easier revision to a total hip replacement in later years

210
Q

A 21-year-old long-distance runner complains of increasing right groin pain. Plain films show
no acute bony injury but demonstrate a pistol grip deformity of the femoral head, an osseous
bump deforming the femoral head–neck junction and an alpha angle of 70. The acetabulum
appears normal. The most likely diagnosis is: [B2 Q14]

a. Hip dysplasia
b. Pincer-type acetabular impingement
c. Cam-type acetabular impingement
d. Sportsman’s hernia
e. Avascular necrosis

A

Cam-type acetabular impingement

Femoro-acetabular impingement (FAI) occurs because of repetitive microtrauma due to an
anatomic conflict between the proximal femur and the acetabular rim at the extremes of motion.
An osseous bump at the femoral head–neck junction is present in 50% of cam-type FAI and
only 33% of pincer-type FAI.

The alpha angle, drawn on the AP pelvis radiograph, is formed by a line drawn from the centre
of the femoral head through the centre of the femoral neck, and a line from the centre of the
femoral head to the femoral head–neck junction, found by the point by which the femoral neck
diverges from a circle drawn around the femoral head. A normal patient’s alpha angle is around
45’, whereas for patients with FAI it may be around 70’.

211
Q

A 28-year-old physically active young man undergoes a hip MR arthrogram for chronic pain
that is worse during exercise. There is a history of several months of hip pain when the patient
was a teenager that was not investigated. Images show a loss of the femoro-acetabular sulcus
superiorly with an associated acetabular labral tear. What is the underlying condition? [B4 Q10]

a. pincer femoro-acetabular impingement
b. cam femoro-acetabular impingement
c. combined femoro-acetabular impingement
d. traumatic labral tear
e. osteochondritis dissecans

A

Cam femoro-acetabular impingement

Cam is the most common form of femoro-acetabular impingement in men, typically presenting
in the third or fourth decade. It is often related to a previous slipped upper femoral epiphysis
in the teenage years. A change in the rotational axis (increase in the alpha angle) causes the
proximal superior femoral neck to impinge upon the superior acetabular margin and labrum, in
turn causing intermittent pain, particularly in physically active individuals. Even without a
history of slipped femoral epiphysis, an osseous bump on the superior femoral neck obliterating
the femoro-acetabular sulcus can cause symptoms. Labral or articular cartilaginous tears can
follow repetitive microtrauma, leading to persistent pain and locking. The pincer type is more
common in women and is caused by an abnormally deep acetabulum.

212
Q

A 75-year-old woman presents with increasing pain in her left hip. She had a total hip
replacement eight years ago on this side which has been asymptomatic ever since. Plain
radiographs demonstrate a lucent line at the bone cement interface of the femoral component.
The likely cause for this is: [B2 Q30]

a. Infection
b. Metastasis
c. Loosening
d. Myeloma
e. Trauma

A

Loosening

Early changes (less than six months) are almost always due to infection. Up to four years,
infection remains the most likely cause, but after this point loosening becomes more common.
Every year, 38 000 hips are replaced in the UK. A routine postoperative film is usually
performed; an excessively varus stem may lead to loosening.

213
Q

A 70-year-old man attends a 6-week follow-up appointment after cemented total hip
arthroplasty, complaining of a poor range of motion. Radiographs taken during the appointment
show small areas of pericapsular bone, and formation of small bony spurs at the acetabular
margin. CT demonstrates these areas to have well-defined mineralization peripherally and
indistinct centres. Which of the following processes are responsible? [B4 Q95]

a. femoral component loosening
b. heterotopic ossification
c. periprosthetic fracture
d. postoperative infection
e. stress shielding

A

Heterotopic ossification

Heterotopic ossification, also known as myositis ossificans, is a benign, self-limiting process
of ossification occurring within skeletal muscle. Seventy-five per cent of cases are due to
trauma (including iatrogenic trauma), with other causes including paralysis, burns, tetanus and intramuscular haematoma. The areas of new bone are surrounded by fibrotic connective tissue,
which can be seen as a soft-tissue mass on MRI. Some heterotopic ossification is seen in half
of all total hip replacements, with one-third considered clinically significant. It is classified
radiographically according to the Brooker classification.

214
Q

A 28-year-old long-distance runner is to undergo MR arthrography of the hip joint for a
suspected labral tear. Which of the following statements is correct regarding MR arthrography?
[B2 Q46]

a. A solution of 20mmol/L gadopentetate dimeglumine is injected into the hip joint under
fluoroscopic guidance
b. Patients with developmental dysplasia of the hip are at increased risk of labral tears
c. A communication between the joint capsule and the iliopsoas bursa is always pathological
d. T2-weighted imaging is used to visualise the high signal of the gadopentetate dimeglumine
solution
e. The normal labrum has uniformly high signal on T1-weighted imaging

A

Patients with developmental dysplasia of the hip are at increased risk of labral tears

The increased risk of labral tears in developmental dysplasia is due to the increased stress
placed upon the acetabular rim and labrum. A communication between the joint capsule and
iliopsoas bursa has been described as a normal finding in 10–15% of patients. A dilute solution
of 0.2 mmol/L gadopentetate dimeglumine solution would usually be used for arthrography. A
normal labrum has uniformly low signal on T1-weighted imaging with slightly increased signal
on gradient echo imaging. Appearances on T2-weighted imaging can be more variable.

215
Q

A 42-year-old man with Type 1 diabetic nephropathy, neuropathy and retinopathy develops
sudden onset pain in the thighs. A diagnosis of diabetic myopathy is considered. Which is the
single best answer? [B3 Q31]

A. Fever is present in most cases
B. Both lower limbs are affected in most cases
C. Low signal on unenhanced T1 indicated haemorrhagic infarction
D. SI is usually low on T2
E. A focus of central low SI surrounded by a rim of high SI on contrast enhanced T1 indicates
muscle infarction and necrosis

A

A focus of central low SI surrounded by a rim of high SI on contrast enhanced T1 indicates
muscle infarction and necrosis

Diabetic myopathy typically presents with sudden onset pain and swelling of the affected
muscles and a palpable painful mass in 1 ⁄ 3 of cases. Fever is present in 10%. Other
complications of diabetes are usually present. Myopathy affects bilateral lower limbs in 18%.
It usually demonstrates diffusely high SI on T2 with areas of high SI on T1 indicating
haemorrhagic infarction

216
Q

A 27-year-old athlete develops groin pain. A pelvic radiograph suggests osteitis pubis. Which
is the single best answer? [B3 Q36]

A. Osteitis pubis usually demonstrates rapid radiographic changes
B. Is due to a single traumatic insult in most cases
C. Is associated with inflammatory mediated inappropriate osteoblastic activity
D. Is associated with pubic or perineal pain with resisted hip abduction
E. Radiographic changes include alternating osteopenia and sclerosis

A

Radiographic changes include alternating osteopenia and sclerosis

This is a self-limiting but often protracted condition secondary to repetitive microtrauma with
osteoclastic activity and osseous resorption. Radiographs show irregularity of subchondral
bone plate, erosions, fragmentation and alternating osteopenia and sclerosis. It may lead to
joint space widening

217
Q

On plain radiographs, which of the following is the most specific indicator of prosthetic
loosening following total hip replacement? [B4 Q40]

a. sclerosis at the tip of the femoral component
b. 3 mm, lucent line at the cement/prosthesis interface
c. heterotopic bone formation
d. periprosthetic fracture
e. femoral periosteal reaction

A

3 mm, lucent line at the cement/prosthesis interface

The artefact created by metallic prostheses on CT and MRI means that plain radiography has
an important role in the evaluation of postoperative arthroplasty joints. Cemented prostheses
may normally show a 1–2 mm lucent line at cement interfaces, but definite loosening is
diagnosed with progressive widening of this zone. Other specific indicators of loosening in
both cemented and uncemented prostheses include migration of components or a new
abnormality of alignment. Periosteal reaction and sclerosis can be normal findings, particularly
in uncemented prostheses. Serial imaging is often required to confirm the diagnosis of
loosening.

218
Q

The Catterall ‘head at risk’ signs are plain radiographic signs that indicate a poor prognosis
and increased risk of femoral head collapse in Legg–Calve´–Perthes disease. Which of the
following options is not such a sign? [B4 Q47]

a. Gage’s sign (wedge-shaped lysis of the femoral head)
b. calcification lateral to the epiphysis
c. lateral subluxation of the epiphysis
d. metaphyseal bone resorption
e. horizontal orientation of the growth plate

A

Metaphyseal bone resorption

Catterall described four radiographic signs that indicate a femoral ‘head at risk’ of avascular
necrosis (Perthes’ disease), to predict those cases in which considerable collapse of the femoral
head may occur.

  1. Gage’s sign is a radiolucent V-shape seen in the lateral side of the epiphysis on frontal
    pelvis radiographs.
  2. Calcification lateral to the epiphysis indicates a small area of lateral head collapse.
  3. The third sign is lateral subluxation, best seen as an increased inferomedial joint space,
    which when seen in conjunction with the first two worsens the prognosis.
  4. Finally, in some individuals the normal growth plate is more horizontally orientated, which
    results in normal weight-bearing forces exerting a shearing force on the growth plate, rather
    than a compressive force as seen in the more common transversely orientated growth plate.
    A horizontal growth plate is therefore considered a further poor prognostic sign in Perthes’
    disease
219
Q

A young adult male sustains an acetabular fracture in a high-speed road traffic collision. Which
type of acetabular fracture is most associated with significant neurological injury? [B4 Q52]

a. posterior rim/wall
b. anterior rim/wall
c. transverse T-shape
d. anterior and posterior column
e. central dislocation

A

Posterior rim/wall

Acetabular fractures are common in multiple or major trauma patients, particularly those
involved in road traffic collisions, and are classified according to the Letournel classification.
Fractures are often complex and require accurate delineation with CT, often following limited
or suboptimal initial radiographic investigation with or without oblique pelvic (Judet) views.
Isolated posterior rim or wall fractures are the most common type (27%) and are associated
with a high frequency of posterior dislocation of the femoral head causing sciatic nerve injury.
If the entire posterior column is involved in the fracture, there is a lower incidence of sciatic
nerve injury, as the femoral head may not be dislocated. Anterior injuries are uncommon (5%)
and may be associated with anterior femoral head dislocation and iliac wing fracture.
Transverse fractures account for 9%.

220
Q

A 30-year-old woman in the third trimester of pregnancy complains of a 4-week history of
gradual onset of pain in the left hip following minor trauma. Radiographs show a normal-
appearing joint space, mild osteopenia of the femoral head and neck, and an indistinct
subchondral femoral head. On subsequent MRI, the bone marrow in the affected regions
returns patchy but diffuse low signal on T1W and high signal on T2W images. There is a
similar small area of marrow abnormality in the acetabulum, and a small hip effusion is seen.
What is the most likely diagnosis? [B4 Q96]

a. septic arthritis
b. infarction
c. reflex sympathetic dystrophy
d. rheumatoid arthritis
e. transient osteoporosis

A

Transient osteoporosis

Transient osteoporosis of the hip is part of the bone marrow oedema syndromes that also
encompass migratory regional osteoporosis and transient bone marrow oedema. The condition
is spontaneous and self-limiting, clinical recovery occurring in several weeks to months with no specific treatment, although radiographic changes lag behind. It is seen in pregnant women
in the third trimester and middle-aged men. The radiographic hallmark is the loss of
subchondral cortex in the femoral head, and marrow oedema is seen on MRI with intense
uptake of 99m Tc-labelled diphosphonates on bone scintigraphy. The aetiology is uncertain,
but speculation has been made that the bone marrow oedema syndromes are related to reflex
sympathetic dystrophy. The appearance of transient osteoporosis of the hip may be mimicked
by osteonecrosis.

221
Q

An 80-year-old diabetic complains of left groin pain. He undergoes twice a week haemodialysis.
The plain radiograph shows large globular periarticular calcifications around both hip joints.
A bone scan shows absence of renal activity and ‘super-scan’ appearance. The calcifications
also show increased tracer uptake. The most likely cause of the calcifications is? [B5 Q6]

(a) Dermatomyositis
(b) Renal osteodystrophy
(c) Scleroderma
(d) Calcium pyrophosphate deposition disease
(e) Renal osteodystrophy

A

Renal osteodystrophy

Patients with renal osteodystrophy have extensive soft tissue calcifications, particularly in
periarticular distribution. ‘Super scan’ appearance is also a feature.

222
Q

A 14-year-old boy presents with persistent right hip pain after a recent injury. Radiographs
confirm the diagnosis of slipped capital femoral epiphysis. What is the Salter–Harris
classification of this condition? [B5 Q50]

(a) Type I
(b) Type II
(c) Type III
(d) Type IV
(e) Type V

A

Type I Salter–Harris injury

Slipped capital femoral epiphysis is classified as a type I Salter–Harris injury because there is
a slipped epiphysis due to the shearing force of an injury, separating the epiphysis from the
metaphysis. There is no fracture of the metaphysis or the epiphysis itself.

223
Q

A 40-year-old immigrant from south Asia with a history of sexually transmitted disease treated
20 years ago presents with a painless, swollen right knee. Radiograph of the right knee shows
collapse and fragmentation of the medial femoral condyle with subluxation of tibiofemoral
joint. There is calcified debris in the knee and a large joint effusion. The bones show excessive
sclerosis. What is the most likely diagnosis? [B5 Q45]

(a) Diabetes
(b) Haemophilia
(c) Charcot’s joint
(d) Calcium pyrophosphate deposition disease
(e) Osteoarthritis

A

Charcot’s joint

The radiographic features are those of Charcot’s joint. Given the history of treated sexually
transmitted disease, syphilis must be considered

224
Q

A 25-year-old male presents with a history of dislocation and spontaneous relocation of the
patella while playing football. An MRI of the knee is requested. Which of the following
findings is consistent with the clinical history of patellar dislocation? [B1 Q3]

A. Bone oedema involving medial facet of patella and medial femoral condyle.
B. Bone oedema involving posterior patella and anterior aspect of the tibial plateau.
C. Bone oedema involving the lateral facet of patella and lateral femoral condyle.
D. Bone oedema involving the lateral facet of patella and medial femoral condyle.
E. Bone oedema involving the medial facet of patella and lateral femoral condyle.

A

Bone oedema involving the medial facet of patella and lateral femoral condyle.

Transient dislocation of the patella typically occurs laterally because of a twisting injury in a
fixed and flexed knee. The medial facet of the dislocated patella impacts against the lateral
femoral condyle, resulting in the classic bone contusion pattern. Rarely oedema may also be
seen in the adductor tubercle of the medial femoral condyle due to avulsion of the medial
patello-femoral ligament.

225
Q

A16-year-old female gymnast sustains a twisting injury to the knee, which becomes
immediately painful and swollen, and she is unable to bear weight. Initial radiographs show an
effusion but are otherwise normal. MRI confirms a joint effusion with a torn medial
retinaculum, marrow oedema affecting the anterior aspect of the lateral femoral condyle, and
a chondral defect of the medial facet of the patella. What is the most likely injury? [B4 Q75]

a. lateral collateral ligament tear
b. medial meniscal tear
c. pivot shift injury
d. transient patellar dislocation
e. posterolateral corner syndrome

A

Transient patellar dislocation

Transient patellar dislocation always occurs laterally and was originally thought to be an injury
confined to teenage girls with abnormal patellofemoral anatomy but is now considered a
potential injury in anyone who partakes in athletic activity. The most common finding is
effusion and lateralization of the patella with or without an abnormally shallow femoral sulcus.
Other findings seen on MRI are contusions of the lateral femoral condyle and medial patella
with potential osteochondral defects, and disruption or sprain of the medial retinaculum. Less
specific findings include loose bodies and associated ligamentous or meniscal injury.

226
Q

You are looking at an MRI of the knees of a 16-year-old male. There is widening of the distal
femoral metaphyses, with a widened intercondylar notch bilaterally. There is mild loss of joint
space height in the medial tibiofemoral compartment, with subchondral cyst formation on the
left, with preserved joint space but subchondral erosions on the right. The ligaments are intact.
GE sequences reveal blooming artefact. Synovial enhancement causing joint erosion is noted
on the enhanced T1WI sequence. What is the likely diagnosis? [B1 Q17]

A. Juvenile arthritis.
B. Pigmented villo-nodular synovitis (PVNS).
C. Amyloid.
D. Haemophilia.
E. Tuberculous arthritis.

A

Haemophilia.

There are three salient features to this question. Firstly, widening of the intercondylar notch—
this is typically caused by haemophilia and JRA, but can also be caused by tuberculous arthritis.
Secondly, causes of arthritis with variable loss of joint space. This is seen in tuberculous
arthritis, amyloid, and PVNS, and may be present in haemophilia, although this can also cause
severe arthropathy. Thirdly, causes of blooming artefact on GE sequences. This is usually
caused by haemosiderin and is found in PVNS and haemophilia. Thus, when the whole picture
is considered, the diagnosis is haemophilia.

227
Q

A 32-year-old woman with a long history of right knee pain undergoes radiography for
atraumatic swelling of the joint and is found to have an effusion and soft-tissue swelling but
no other findings. MRI shows a large anterolateral lobular intra-articular mass of low signal on
T1W and T2W images, and a blooming artefact is seen on gradient echo sequences. What is
the most likely condition? [B4 Q22]

a. malignant fibrous histiocytoma
b. pigmented villonodular synovitis
c. synovial osteochondromatosis
d. Baker’s cyst
e. intra-articular haematoma

A

Pigmented villonodular synovitis

Pigmented villonodular synovitis is a benign proliferative disorder of the synovium that has a
propensity for young to middle-aged adults and typically has a long history. On plain
radiographs, joint space and bone mineralization are typically preserved until late in the disease,
but softtissue swelling or effusion may be apparent early on. Haemorrhage is relatively common and can result in haemarthrosis and blooming artefact seen on gradient echo MRI
sequences. Malignant fibrous histiocytoma is the most common soft-tissue sarcoma after age
50. Synovial osteochondromatosis is more common in men and is characterized by
proliferation of the synovium with formation of cartilaginous nodules (that often calcify) but
does not show haemorrhage. Baker’s cyst has synovial fluid characteristics on MRI and is
located posterior to the joint.

228
Q

A 40-year-old man presents with progressive pain and swelling of the left knee joint. MRI
shows extensive low-signal synovial masses around the right knee on T1, T2 and STIR
sequences. There is marked degenerative joint disease as well. What is the most likely
diagnosis? [B5 Q49]

(a) Synovial chondromatosis
(b) Pigmented villonodular synovitis
(c) Synovial hypertrophy
(d) Lipoma arborescens
(e) Degenerative arthritis

A

Pigmented villo-nodular synovitis

The MRI appearances described are typical of pigmented villonodular synovitis, showing low
signal of abnormal synovium on all sequences.

229
Q

An 18-year-old man presents with progressive swelling of right knee. Radiographs show large
joint effusion in the suprapatellar pouch. MRI shows marked synovial thickening and large
synovial fronds which return high signal on T1, T2 and proton density images. The lesions are
low signal on STIR images. The most likely diagnosis is? [B5 Q29]

(a) Synovial lipoma
(b) Synovial osteochondromatosis
(c) Hypertrophic synovitis
(d) Pigmented villonodular synovitis
(e) Lipoma arborescens

A

Lipoma arborescens

This condition is seen most in the suprapatellar pouch, with small to large frond-like masses
arising from synovium. On MRI, the masses show characteristic signal of fat on all sequences.
Saturation on STIR images is diagnostic

230
Q

A patient attends A&E following an RTA in which she was the driver of car involved in a head-
on collision. She complains of pain in both knees. Plain radiographs of the knees are
unremarkable. Which of the following findings on MRI is most likely? [B1 Q19]

A. Bruising in the posterior aspect of the lateral tibial plateau and middle portion of the lateral
femoral condyle.
B. Bruising at the anterior aspect of the tibia.
C. Kissing contusions in the anterior aspect of the distal femur and proximal tibia.
D. Bruising in the lateral femoral condyle with a second smaller area in the medial femoral
condyle.
E. Bruising in the inferior medial patella and the anterior aspect of the lateral femoral condyle.

A

Bruising at the anterior aspect of the tibia

Such bruising occurs in a dashboard injury when a posteriorly directed force is applied to the
anterior aspect of the proximal tibia with the knee in flexion, such as occurs in an RTA.
Bruising is also occasionally found in the posterior patella in this situation. Associated soft-
tissue injuries are disruption of the posterior capsule and posterior cruciate ligament (PCL).

The pattern of injury in option A is caused by the pivot shift injury (valgus load in flexion
combined with external rotation of the tibia or internal rotation of the femur). This will result
in anterior cruciate ligament (ACL) disruption and the resultant anterior subluxation of the tibia
causes impaction of the lateral femoral condyle against the posterolateral margin of the lateral
tibial plateau. Soft-tissue injuries that may occur are tears of the posterior capsule, the posterior
horn of the lateral or medial meniscus, and the medial collateral ligament (MCL). The kissing
contusions in option C: are because of hyperextension injury; resulting injuries may be to the
ACL, PCL, or menisci. Option D describes the pattern found in clip injury, which involves a
pure valgus stress while the knee is in mild flexion. The second area of bruising in the medial
femoral condyle in this situation is due to avulsive stress to the MCL. The findings in option E
are in keeping with transient lateral patellar dislocation, as discussed elsewhere in this chapter.

231
Q

A young footballer sustains a twisting injury to the right knee in training. He is able to continue
practising but complains of moderate medial knee pain. The following morning, he wakes with
a swollen stiff joint. Radiographs show an effusion only. Subsequent MRI confirms an effusion
and reveals a truncated medial meniscus with a ‘bow-tie’ configuration seen on only a single
sagittal image. Sagittal sequences reveal a ‘double’ appearance of the posterior cruciate
ligament. He has not had any previous surgery. What is the most likely injury or combination
of injuries? [B4 Q99]

a. torn medial meniscus
b. torn medial meniscus and anterior cruciate ligament
c. torn medial meniscus and posterior cruciate ligament
d. torn anterior cruciate ligament
e. torn posterior cruciate ligament

A

Torn medial meniscus

Truncation of a meniscus may be due to previous injury or surgical resection, but in the absence
of a relevant history it suggests meniscal tear with displacement of the body of the meniscus.
On sagittal sequences, one would normally expect to see a full ‘bow-tie’-shaped meniscus on
three or more contiguous images, as the meniscal body is approximately 11 mm in thickness
(this of course will depend on slice thickness). Any fewer suggests a meniscal body tear with
displacement of the fragment. The fragment often flips into the intercondylar groove of the
femur to lie anterior and parallel to the posterior cruciate ligament, giving the impression of
two similar structures. This injury is known as a bucket-handle tear.

232
Q

A 62-year-old man presents with sudden-onset pain after minor injury. Plain radiograph shows
subchondral sclerosis in the medial femoral condyle and joint effusion. MRI shows a diffuse
oedema in the subchondral bone of medial femoral condyle with a crescentic linear fracture in
a subchondral location. The most likely diagnosis is? [B5 Q28]

(a) Spontaneous osteonecrosis of knee
(b) Osteoarthritis
(c) Osteochondritis desiccans
(d) Calcium pyrophosphate deposition disease
(e) Gout

A

Spontaneous osteonecrosis of knee

Typical subchondral fracture in elderly patient after minor knee injury.

233
Q

A 40-year-old man with a history of dislocated left hip in a road traffic accident 2 years ago
presents with left hip pain. Radiography shows flattening and sclerosis in the superolateral part
of the femoral head. The most likely diagnosis is? [B5 Q37]

(a) Degenerative arthritis
(b) Calcium pyrophosphate deposition disease
(c) Avascular necrosis
(d) Paget’s disease
(e) Prostatic metastases

A

Avascular necrosis

Avascular necrosis of the femoral head is a complication after hip dislocation. It may be seen as a wedge-shaped or geographical area of subchondral ischemic focus in the weight-bearing area. On MRI, there is a hyperintense inner border parallel to a hypointense periphery.

234
Q

A 42-year-old man in remission for lymphoma complains of bilateral hip pain. Coronal T1
images on MRI show geographical areas of abnormality in bilateral femoral heads, which are
well demarcated from the normal bone by a thin rim of low signal on T1-weighted images.
What is the most likely diagnosis? [B5 Q46]

(a) Lymphoma recurrence
(b) Red bone marrow
(c) Avascular necrosis
(d) Osteomyelitis
(e) Stress fractures

A

Avascular necrosis

This appearance on MRI is typical of avascular necrosis.

235
Q

A 54-year-old man presents with a swelling in his right popliteal fossa. A Baker’s cyst is
suspected clinically, and an ultrasound scan is arranged. This confirms a complex cystic
structure with debris. To help confirm this is a Baker’s cyst, you look for a communication of
this cyst with fluid at the posterior aspect of the knee joint between which two tendons? [B1
Q21]

A. Semi-tendinosis and lateral head of gastrocnemius.
B. Semi-tendinosis and medial head of gastrocnemius
C. Semi-tendinosis and semi-membranosis.
D. Medial and lateral heads of gastrocnemius.
E. Lateral head of gastrocnemius and semi-membranosis.
F. Medial head of gastrocnemius and semi-membranosis.

A

Medial head of gastrocnemius and semimembranosis.

Identification of anechoic cysts communicating with fluid between the semimembranosis and
gastrocnemius tendons confirms the diagnosis of Baker’s cyst. It is important to perform further
imaging if the mass in the posterior compartment lacks signs of communication with fluid
between the semimembranosis and medial gastrocnemius tendons. If this is the case, there are
other possibilities for the lesion, including meniscal cyst or even a myxoid sarcoma.

236
Q

A 15-year-old boy presents with a history of knee pain. Plain radiographs demonstrate
calcification at the patellar tendon attachment to the inferior pole of the patella. MRI of the
knee demonstrates oedema at the patellar attachment of the patellar tendon. What is the
diagnosis? [B1 Q41]

A. Osgood–Schlatter disease.
B. Patellar sleeve avulsion.
C. Sinding–Larsen–Johansson syndrome.
D. Complete rupture of patellar tendon.
E. Partial tear of quadriceps tendon.

A

Sinding–Larsen–Johansson syndrome.

This is a traction tendonitis occurring at the attachment of the patellar tendon to the inferior
pole of the patella. Repetitive stress/microtrauma at the tendinous attachment results in
calcification or ossification of the tendon on the plain film. MRI demonstrates oedema within
the tendon and at the inferior pole of the patella.

Similar changes occurring at the tibial attachment of the patellar tendon is called Osgood-
Schlatter disease.

Patellar sleeve fracture is a unique paediatric injury in which the cartilage at the inferior pole
of the patella is avulsed along with a small bone fragment.

The quadriceps tendon inserts into the superior pole of the patella, therefore a partial tear
produces oedema at the superior pole of the patella

237
Q

A 47-year-old man presents with a progressive history of pain, swelling, and reduced range of
movement affecting his right knee. Symptoms have been ongoing for 2–3 years. Locking is
noted on examination. Radiography of his knee reveals multiple intra-articular calcifications.
A supra-patellar joint effusion is also present. The joint space is maintained. What is the most
likely diagnosis? [B1 Q50]

A. Neuropathic arthropathy.
B. Osteochondritis dissecans.
C. Osteochondral fracture.
D. PVNS.
E. Synovial osteo-chondro-matosis.

A

Synovial osteo-chondro-matosis.

The primary form of this represents an uncommon benign neoplastic process with hyaline
cartilage nodules in the subsynovial tissue of a joint, tendon sheath, or bursa. Secondary
synovial chondromatosis is associated with joint abnormalities, such as mechanical or arthritic
conditions, that cause intraarticular chondral bodies. The primary form of the disease
predominantly affects men in the third to fifth decades. The knee is the most common site, but
it is also seen in the hip, shoulder, elbow, and ankle; less commonly the MCP, IP, distal
radioulnar, and acromioclavicular joints are involved. It can rarely involve extra-articular sites,
the synovium about the tendons, or bursa. Clinical symptoms typically include pain, swelling,
and restriction of the range of motion of the joint. Radiologic findings are frequently
pathognomonic. Radiographs reveal multiple intraarticular calcifications (70–95% of cases) of
similar size and shape distributed throughout the joint, with typical ‘ring-and-arc’ chondroid
mineralization. Extrinsic erosion of bone is seen in 20–50% of cases. Juxtaarticular osteopenia
is not typically apparent in synovial chondromatosis unless it is the result of disuse. CT is often
diagnostic if the bodies are adequately mineralized and is particularly helpful for identifying
characteristic ring-and-arc or punctate mineralization and the multiplicity of nodules in cases
for which radiographic findings are normal or equivocal. Lack of mineralization of the bodies
does occur in which cases MRI is very helpful to distinguish from, for example, PVNS or
amyloid. The most common pattern on MRI (77% of cases) reveals low to intermediate signal
intensity with T1WI and very high signal intensity with T2WI with hypointense calcifications.

238
Q

A 35-year-old man presents with pain, swelling, and reduced movement of his knee. A plain
film reveals a joint effusion, well-defined erosions with preservation of joint space, and normal
bone mineralization. An MRI reveals, in addition, a mass in the region of the femoro-tibial
joint space with low signal on T1WI and T2WI, and blooming artefact on GE imaging. What
is the most likely diagnosis? [B1 Q75]

A. Synovial cell sarcoma.
B. Regional migratory osteoporosis.
C. Gout.
D. Synovial chondromatosis.
E. PVNS.

A

PVNS.

This is a monoarticular tumour-like proliferation of synovium that occurs in joints, bursae, and
tendon sheaths. It may be focal or diffuse. It occurs most frequently in the knee (80% of cases),
then the hip, ankle, shoulder, and elbow. The abnormal synovium is prone to haemorrhage,
thus producing blooming artefact on GE sequences, secondary to haemosiderin deposition.

In general the classic MRI appearance is variable low signal intensity on all sequences (T2WI
signal being more variable due to fat, oedema, and blood products). Early changes involve a
focal mass and joint effusion. Subsequently large erosions, synovial hypertrophy, and
subchondral cysts may occur. Joint space is preserved until advanced disease is present and
bone density is normal. After IV contrast at CT, PVNS shows variable enhancement, which can be striking. The differential diagnosis includes diseases causing recurrent haemarthroses,
e.g. haemophilia and haemochromatosis (PVNS is monoarticular) as well as gout, amyloid,
synovial chondromatosis, and tuberculosis.

Some 90% of synovial cell sarcomas do not originate from a joint. They are usually isointense
to muscle on T1WI, with heterogeneous high-signal intensity on T2WI. Regional migratory
osteoporosis would obviously involve loss of bone mineralization, as well as marrow oedema.
Gout demonstrates typically ‘rat’s bite’ para-articular erosions and soft-tissue calcification;
when it involves the knee it tends to affect the patello-femoral compartment.

239
Q

On reviewing a knee radiograph of a 17-year-old boy with knee pain, squaring of the patella,
widening of the intercondylar notch, flattening of the condylar surface and medial slanting of
the tibiotalar joint are noted. There is also patelallo-femoral joint space narrowing and a joint
effusion. Which of the following is most likely? [B3 Q20]

A. Psoriatic arthropathy
B. Haemophilic arthropathy
C. Juvenile idiopathic arthritis
D. TB arthritis
E. Rheumatoid arthritis

A

Although all are causes of a widened intercondylar notch, the radiographic features are of
haemophilic arthropathy. MRI shows low SI of hypertrophied synovial membrane on all pulse
sequences due to haemosiderin

240
Q

A 24-year-old man undergoes acute trauma to his right knee playing football. He is unable to
weight bear. An x-ray of the right knee is performed, and this demonstrates a large joint
effusion and a small, avulsed elliptical fragment of bone at the medial aspect of the proximal
tibia at the joint margin. Which knee structure is likely to be deranged in association with this
injury at a subsequent MRI? [B1 Q65]

A. Anterior cruciate ligament.
B. Posterior cruciate ligament.
C. Lateral collateral ligament.
D. Patellar tendon.
E. Lateral meniscus.

A

Posterior cruciate ligament.

The avulsion injury described is a reverse Segond fracture. This injury is known to be
associated with both mid-substance tears of the posterior cruciate ligament and avulsions of
the PCL from the posterior tibial plateau. They can also be associated with medial meniscus
injuries.

They are not to be confused with a Segond fracture, which is a small elliptical fragment
of bone avulsed from the lateral tibial plateau at the lateral joint margin, best seen on the AP
view of the knee. They have a strong association with tears of the anterior cruciate ligament
and meniscal tears.

241
Q

Plain knee radiographs performed in accident and emergency following a sports injury in a 20-
year-old footballer show an effusion, a small avulsion fracture immediately proximal to the fibular head, deepening of the lateral femoral sulcus and anterior translocation of the tibia.
What is the most likely underlying ligamentous injury? [B4 Q3]

a. complete posterior cruciate ligament rupture
b. complete anterior cruciate ligament rupture
c. partial anterior cruciate ligament rupture
d. tibial collateral ligament rupture
e. fibular collateral ligament rupture

A

Complete anterior cruciate ligament rupture

The avulsion fracture described is a Segond fracture, which is classically associated with
anterior cruciate ligament (ACL) rupture and represents avulsion of the meniscotibial portion
of the middle third of the lateral capsular ligament. Anterior translocation of the tibia occurs in
complete ACL rupture, and manifests clinically as the anterior draw sign. Also associated with
ACL rupture is an impaction injury of the lateral femoral condyle, which can be seen on
radiographs as a deepened lateral femoral condylar sulcus, although sometimes this cannot be
identified on acute films.

242
Q

A 24-year-old man injured his left knee whilst skiing. He presents with pain and swelling over
the lateral aspect of the knee joint. AP plain radiographs demonstrate an avulsion fracture of
the lateral aspect of the proximal tibia below the articular surface. A joint effusion is also seen.
The most likely associated ligamentous injury is to which of the following structures? [B2 Q10]

a. Posterior cruciate ligament
b. Anterior cruciate ligament
c. Medial collateral ligament
d. Lateral collateral ligament
e. Ligament of Humphry

A

Anterior cruciate ligament

The fracture described is a Segond fracture, originally documented by Dr Paul Segond in 1879
after a series of cadaveric experiments. The Segond fracture occurs most commonly in
association with anterior cruciate ligament injuries (75–100%) and medial meniscal injuries.
Due to the high rate of associated injuries, a patient who sustains a Segond fracture will require
further imaging, usually by way of MRI, to specifically investigate the ligaments and menisci.

243
Q

A 20-year-old student complains of a six-week history of pain and tenderness in his right thigh
associated with a soft-tissue mass. There is no definite history of trauma. CT of the region
shows a mass in the right distal femur with well-defined mineralisation at the periphery and a
less distinct lucent centre. On plain film, there is faint calcification within the lesion and a
radiolucent zone separating the lesion from bone. The most likely cause is: [B2 Q41]

a. Tumoural calcinosis
b. Osteomyelitis
c. Myositis ossificans
d. Parosteal sarcoma
e. Osteosarcoma

A

Myositis ossificans

Often there is no distinct history of trauma, although this is the most common cause. It usually
occurs in the large muscles of the extremities and in the early stages it can be difficult to
distinguish from soft-tissue sarcomas. It is, however, separate from bone, unlike parosteal
sarcoma and post-traumatic periostitis. This is a self-limiting condition, most commonly
occurring in young athletic adults, with resorption occurring in approximately one year.

244
Q

A 34-year-old man has a 3-month history of right knee pain. There is a remote history of
previous right leg trauma. He has an x-ray of the right knee performed, which demonstrates a
densely ossified mass immediately adjacent to the posterior cortex of the distal femur. You
determine that the differential diagnosis is between post-traumatic myositis ossificans or a par-
osteal osteosarcoma. Which of the following features on plain x-ray is likely to be most helpful
in distinguishing between these diagnoses? [B1 Q57]

A. Periosteal reaction in the adjacent bone.
B. Presence of lucent areas in the lesion.
C. Pattern of ossification in the lesion.
D. Size of the lesion.
E. Presence of lucent cleft between the lesion and adjacent bone.

A

Pattern of ossification in the lesion.

The pattern of ossification is likely to be the most helpful. In post-traumatic myositis ossificans, the ossification occurs classically first at the periphery.

whereas in par-osteal osteosarcoma, the ossification is diffuse, but predominantly central. Periosteal reaction is typically absent in both these lesions and both lesions may contain lucent areas on plain radiograph.

A lucent cleft between the mass and the bony cortex, representing periosteum, is characteristic
in par-osteal osteosarcoma, but frequently is not seen as the tumour envelops bone. A thick
lucent zone separating myositis ossificans from an adjacent bony cortex is typical but may not
be seen on plain radiograph if the lesion is immediately juxta cortical.

245
Q

A 60-year-old man is referred for an MRI of his left upper leg after noticing a slowly enlarging
firm mass measuring approximately 7–8cm in maximum diameter. The mass is located in the
quadriceps muscle group and is causing cortical erosion of adjacent bone. There are poorly
defined calcifications within it and MR shows a poorly defined lesion which is isointense to
muscle on T1-weighted imaging and hyperintense on T2-weighted imaging. The most likely
diagnosis is: [B2 Q42]

a. Malignant fibrous histiocytoma
b. Benign fibrous histiocytoma
c. Liposarcoma
d. Fibrosarcoma
e. Elastofibroma

A

Malignant fibrous histiocytoma

Soft-tissue malignant fibrous histiocytoma is the most common primary malignant soft tissue tumour of later adulthood. It is most seen in the lower extremities. It has a metastatic rate of 42% and most commonly metastasises to the lung. Osseous malignant fibrous histiocytoma presents as a painful, tender, rapidly enlarging mass and most commonly arises in the metaphysis of long bones.

246
Q

A 35-year-old man presents with knee pain. MRI shows a 1.5 cm homogenous ovoid lesion
which pushes the medial collateral ligament. It returns high signal on STIR and T2 images.
There is also a horizontal cleavage tear of the posterior horn of the medial meniscus and a radial
tear of the lateral meniscus. The most likely cause of the lesion is? [B5 Q16]

(a) Dissecting baker’s cyst
(b) Ganglion
(c) Pes anserinus bursa
(d) Haemangioma
(e) Meniscal cyst

A

Meniscal cyst

Meniscal cysts are very commonly associated with meniscal tears. MRI shows classic features
of the cyst, related to the parameniscal structures. Lateral meniscal cysts tend to be smaller but
are often more symptomatic than cysts in the medial counterparts.

247
Q

A 64-year-old woman undergoes MRI of her left knee for investigation of chronic knee pain.
Which of the following would be considered an abnormal finding on MR? [B2 Q57]

a. Bowing of the posterior collateral ligament on sagittal imaging
b. Low signal ACL on T1-weighted imaging
c. High signal around the MCL on T2* on coronal imaging
d. Low signal of the menisci on both T1- and T2-weighted imaging
e. Medial patellar plica

A

High signal around the MCL on T2* on coronal imaging

The only abnormal finding is the presence of high signal around the MCL on T2* imaging.
This would represent oedema or haemorrhage around the MCL and may be associated with a
tear. Bowing of the PCL occurs when the knee is extended. A medial patellar plica is a normal
finding in approximately 50% of the population. This is an embryological remnant from when
the knee was divided into three compartments

248
Q

A 20-year-old man sustains a knee injury after playing football. Regarding imaging of the
infrapatellar tendon, which is the best answer? [B3 Q3]

A. Normal infrapatellar tendon appears high signal on all sequences
B. A triangular area of high signal at the patellar enthesis when imaged on gradient echo
indicates tendon rupture
C. In infrapatellar tendnopathy, the tendon may be swollen and contain focal areas of reduced
echogenicity
D. The paratenon is more commonly the primary site of acute inflammation in infrapatellar
tendon than the Achilles tendon
E. An echo-rich halo around the tendon is seen in paratenonitis on US

A

C.
The normal infrapatellar tendon is homogenously low SI on all sequences. A triangular area of
high signal at the patellar enthesis when imaged on gradient echo is of no clinical significance.
The paratenon is more commonly the primary site of acute inflammation in the Achilles tendon
than the infrapatellar tendon. An echo-poor halo around the tendon is seen in paratenonitis on
US.

249
Q

Which of the following indicates grade 4 chondromalacia patellae on T1 MRI? [B3 Q16]

A. Loss of sharp dark margin between articular cartilage of the patella and trochlea
B. Focal hypointense areas extending to the articular surface but not down to the osseous
surface
C. Focal hypointense areas extending from subchondral bone to cartilage surface
D. Focal hypointense areas extending to the cartilage surface with preservation of sharp
cartilage margins
E. Focal hypointense areas not extending to cartilage surface

A

C

Grade 1: focal hypointense areas not extending to cartilage surface
Grade 2: focal hypointense areas extending to the cartilage surface with preservation of sharp
cartilage margins
Grade 3: loss of sharp dark margin between articular cartilage of the patella and trochlea and
focal hypointense areas extending to the articular surface but not down to the osseous surface.

250
Q

MRI of the knee in an 18-year-old man, performed for pain and limited joint movement, reveals
an osteochondral lesion of the medial femoral condyle. Other than displacement, which MRI
finding is the most specific indication of an unstable osteochondral fragment? [B4 Q30]

a. joint effusion
b. sub-fragmental bone resorption
c. 3 mm cyst deep to the lesion
d. underlying linear high signal on T2W images
e. multiple lesions

A

Underlying linear high signal on T2W images

High T2 signal in the bone underlying an osteochondral lesion has been described as the most
common of four MRI findings indicating instability of an osteochondral fragment, which is the
most important factor when considering treatment options. The reported accuracy of this sign
for predicting instability varies from 45% to 85%, with one study reporting an increased
accuracy when this sign is combined with the second sign of a cartilaginous defect on T1W
images. However, another study states that often only a single indicator is present. The other
indicators of instability are high signal in the articular cartilage and a cystic lesion in the bed
(but this needs to be 5 mm or larger)

251
Q

O’Donoghue’s unhappy triad consists of injuries to which three internal structures of the knee
that are commonly injured together? [B4 Q50]

a. anterior cruciate and lateral collateral ligaments, medial meniscus
b. anterior cruciate and lateral collateral ligaments, lateral meniscus
c. anterior cruciate and medial collateral ligaments, medial meniscus
d. anterior cruciate and medial collateral ligaments, lateral meniscus
e. posterior cruciate ligament, medial and lateral menisci

A

Anterior cruciate and medial collateral ligaments, medial meniscus

O’Donoghue’s unhappy triad consists of injuries to the anterior cruciate and medial collateral
ligaments and the medial meniscus, and is an injury associated with contact sports. The
mechanism is indirect trauma causing deceleration, hyperextension and twisting forces. The
combination of external rotation of the tibia on the femur, knee flexion and valgus stress can
produce an anterior cruciate ligament injury combined with additional medial collateral
ligament injury. The meniscus and collateral ligament medially are attached to one another,
unlike their lateral counterparts, resulting in a higher frequency of concordant injury to the
other medial structure when one is injured

252
Q

A 24-year-old man was involved in a road traffic accident. CT of the left knee shows isolated
5 mm depression of the lateral tibial plateau. What is the Schatzker classification of this
fracture?

(a) Type 1
(b) Type 2
(c) Type 3
(d) Type 4
(e) Type 5

A

Schatzker type 3

Type 1 is lateral condylar split, type 2 is lateral split with depression, type 3 is pure lateral
depression, type 4 is medial condylar fracture and depression, type 5 is bicondylar fracture and
type 6 is fracture extending to metadiaphysis.

253
Q

A 30-year-old female runner presents with a history of pain in the legs on running. Plain
radiographs are unremarkable. An isotope bone scan reveals subtle, longitudinal, linear uptake
on the delayed bone scan images, with normal angiogram and blood pool images. What is the
diagnosis? [B1 Q26]

A. Stress fracture.
B. Shin splints.
C. Osteoid osteoma.
D. Osteomyelitis.
E. Hypertrophic osteoarthropathy.

A

Shin splints.

Excessive exertion of tibialis and soleus muscles of the legs causes periostitis along the muscular attachments. This shin splints results in longitudinal linear uptake on delayed bone scan images. The angiogram and blood pool images are usually normal compared to:

Stress fracture, which is associated with hyper-perfusion and hyperaemia. On delayed images, focal fusiform uptake is seen with stress fracture.

Infection is associated with hyper-perfusion, hyperaemia, and focal increased uptake.

Osteoid osteoma demonstrates hyper-perfusion, hyperaemia, and focal double density due to
nidus and reactive osteosclerosis.

Paget’s disease is associated with increased uptake in an enlarged and deformed bone. Age and
clinical presentation are also against this diagnosis.
Hypertrophic osteoarthropathy is associated with irregular cortical uptake producing the
‘tramline’ sign.

254
Q

A 20-year-old runner presents with a history of right leg pain for 4 weeks. Radiography of the
right leg shows a transverse cortical lucency and cortical thickening in the anterior cortex of
the mid shaft of the tibia. What is the most likely diagnosis? [B5 Q41]

(a) Stress fracture
(b) Nutrient artery foramen
(c) Osteomalacia
(d) Normal variant
(e) Hypertrophic pulmonary osteoarthropathy

A

Stress fracture

This is commonly seen in runners and other athletes, in marching soldiers and in other patients
in whom repetitive prolonged muscular action and stress are applied to a bone that is not
accustomed to such action.

255
Q

A 45-year-old man attends the Accident & Emergency Department with a 1-week history of
foot pain. He is a regular runner and recently completed the London marathon. There is a
history of lymphoma 15 years ago treated with chemotherapy. He is also diabetic and has
chronic renal failure. Radiographs demonstrated a subtle periosteal reaction at the second
metatarsal shaft. Bone scan shows focal tracer uptake in the second metatarsal region. What is
the most likely diagnosis? [B5 Q1]

(a) Lymphoma deposit in second metatarsal
(b) Stress fracture
(c) Osteomyelitis
(d) Neuropathic foot
(e) Osteomalacia

A

Stress fracture

This is the most likely diagnosis as this is a very common site for stress fractures in the feet of
runners, especially affecting the second and third metatarsals. Bone scan is almost 100%
sensitive showing abnormal uptake within 6–72 hours of injury. MRI is also a very sensitive
modality showing intermediate signal intensity on T1 and high signal on STIR and T2-
weighted images.

256
Q

An 81-year-old male diabetic is referred from the endocrinology team for an MRI of foot. This
patient was seeing a podiatrist, who became concerned that the foot had become increasingly
deformed and was acutely red and swollen around the tarso-metatarsal joints. The patient is
asymptomatic as he has peripheral neuropathy. The clinical query is whether this patient has
osteomyelitis/septic arthritis in this region, or neuropathic arthropathy. Which of these MRI
features would be more typically associated with osteomyelitis than acute neuropathic
arthropathy? [B1 Q42]

A. Focal involvement.
B. Predominant midfoot involvement.
C. Associated bony debris.
D. High T2WI and STIR, low T1WI. Enhancement present.
E. Bony changes are in a periarticular and subchondral location.

A

Focal involvement.

Whilst differentiating these conditions can be difficult and they frequently overlap, there are
certain features that can be of value. Neuropathic arthropathy (NA) seldom affects a single
bone/ joint in the foot and is most common in the midfoot region. As such a more focal
abnormality, or abnormality affecting the metatarsal heads, or other points of pressure, should
indicate osteomyelitis. Whilst high T2WI/STIR, low T1WI and enhancement are seen in
osteomyelitis, it is also seen in acute neuropathic arthritis and as such is not a good
differentiating factor. The converse is not true, where low signal on T1WI and T2WI, typical
of chronic NA, would make the presence of osteomyelitis unlikely.

257
Q

A 40-year-old female presents with a small lump in her foot. An MRI of the foot demonstrates
a small soft tissue mass, which has homogenous low signal on T1WI and T2WI. The mass
enhances with gadolinium. What is the most likely diagnosis? [B1 Q46]

A. Morton’s neuroma.
B. Lipoma.
C. Ganglion cyst.
D. Plantar fibromatosis.
E. Hemangioma

A

Plantar fibromatosis

Fibrous masses containing mature collagen are homogenously low in signal on T1WI and
T2WI sequences and demonstrate enhancement with gadolinium. Common fibrous masses in
the foot are plantar fibromatosis and fibroma of the tendon sheath.

Morton’s neuroma is typically intermediate in signal on T1WI and low on T2WI with variable
contrast enhancement.

Lipomas follow fat signal intensity. They are high on T1WI and T2WI, and low on fat-
suppressed sequences.

A ganglion cyst follows fluid signal. Ganglion cysts are low on T1WI and high on T2WI with
rim enhancement.

Haemangiomas are of mixed signal on T1WI and T2WI due to the presence of vessels, fat, and
fibrous tissue. The vascular portions of hemangiomas enhance homogenously

258
Q

A 25-year-old marathon runner presents with a history of right calf pain during exercise.
Popliteal artery entrapment is suspected clinically. Which of the following statements
regarding imaging of popliteal artery entrapment syndrome (PAES) is true? [B1 Q60]

A. In PAES, the popliteal artery is compressed with the ankle in the neutral position.
B. A normal Doppler ultrasound of the popliteal artery excludes the diagnosis.
C. In the normal popliteal fossa, the popliteal artery and vein pass lateral to the medial head of
the gastrocnemius and are surrounded by fat.
D. The anatomical abnormality is invariably unilateral.
E. Catheter angiography is the gold standard for the diagnosis.

A

In the normal popliteal fossa, the popliteal artery and vein pass lateral to the medial head of the
gastrocnemius and are surrounded by fat
.

Popliteal artery entrapment syndrome is a developmental abnormality resulting from an
abnormal relationship between the popliteal artery and neighbouring muscles. It is commonly
seen in healthy young adults and can present with symptoms of intermittent claudication or
thromboembolism. The anatomical abnormality occurs bilaterally in 27–67%.

The popliteal vessels normally pass lateral to the medial head of gastrocnemius. An anomalous
origin of the medial head or an anomalous course of the popliteal artery may result in extrinsic
compression of the artery

Doppler and digital subtraction angiography (DSA) findings may be non-specific with a wide
spectrum of findings. A normal Doppler or DSA with neutral ankle position does not exclude
the diagnosis. Provocative measures with ankle dorsiflexion and plantar flexion may be useful
in confirming the diagnosis on Doppler and DSA, but they do not demonstrate the underlying
anatomical cause. Non-invasive assessment with CT angiogram or MR angiogram is preferred
as they also demonstrate the anatomical abnormality.

259
Q

A 30-year-old male presents with a history of painful heels after a fall from a height. Plain
radiograph demonstrates calcaneal fractures. Which of the following statements regarding
calcaneal fractures is true? [B1 Q69]

A. Extraarticular fractures represent 75% of all calcaneal fractures.
B. Calcaneal fracture classification is based on fracture line location at the posterior facet.
C. Bilateral fractures are present in 30% of cases.
D. The flexor hallucis longus tendon passes inferior to the sustentaculum tali on the lateral
aspect of the calcaneus.
E. Normal Boehler’s angle is less than 20°.

A

Calcaneal fracture classification is based on fracture location at the posterior facet.

Calcaneal fractures represent 60% of fractures involving the tarsal bones. Axial loading
resulting from a fall from a height is the most common cause followed by motor vehicle
accidents. Treatment is based on accurate evaluation and classification of calcaneal fractures
using multidetector CT reformats.

Calcaneal fractures are classified into intra-articular and extra-articular based on the
involvement of the posterior facet of the subtalar joint.
Intra-articular fractures, accounting for
75% of all calcaneal fractures, are further classified into four types depending on the number
of fracture lines and fragments. Extra-articular fractures are classified into three types
depending on whether the fracture involves the anterior, middle, or posterior aspect of
calcaneus.

Bilateral fractures are seen in less than 10% of cases. Approximately 10% of calcaneal fractures
are associated with compression injuries of the spine, commonly at the thoracolumbar junction.

Boehler’s angle is formed by the intersection of (a) a line from the highest point of the posterior
calcaneal tuberosity to the highest point of the posterior facet and (b) a line from the latter point
to the highest of the anterior process. *Normal Boehler’s angle is 20–40°. An angle less than 20°
indicates collapse of the posterior facet.**

The sustentaculum tali is an eminence on the medial aspect of the calcaneus bearing the middle
facet of subtalar joint.*

260
Q

A patient is being assessed for a possible congenital foot deformity. Both AP and lateral
weight-bearing views of the hind and fore foot have been taken. You are lucky to have an
experienced radiographer working with you and she has carried out the standard measurements
in assessing for foot deformities. The tibio-calcaneal angle measures 60° on the lateral foot
image. On the AP hindfoot image, the talocalcaneal angle measures 30°. On the lateral foot image the metatarsals are superimposed, with the fifth metatarsal being in the most plantar
position. What sort of deformity does this patient have? [B1 Q71]

A. None.
B. Clubfoot.
C. Rocker bottom foot.
D. Flexible flat foot deformity.
E. Pes cavus.

A

None.

This is a complicated question, but one that is frequently posed to musculoskeletal radiologists.
A full description is beyond the remit of this section and the reader is referred to the excellent
synopsis referenced below. Basically, the foot is divided into the hind foot and fore foot when
assessing for congenital abnormalities. The tibio-calcaneal angle (angle made by a line drawn
along the length of both bones) should be between 60° and 90°. Less than this is due to
abnormal dorsiflexion (e.g. as seen with congenital vertical talus, or rocker bottom foot) and
more is due to equinus deformity. Secondly, degree of hindfoot varus or valgus is assessed.

The talo-calcaneal angle (on a frontal hindfoot x-ray this is the angle between the lines drawn
along the length of each bone) should be between 15° and 40°; less is varus deformity and more
is valgus.

On the lateral view, the fore foot can be roughly assessed with two observations. Normally the
metatarsals overlap. However, if the first metatarsal is the most plantar, then forefoot valgus is
present (as the foot is too flat). If the metatarsals are not superimposed, then varus is present.

The final step is to remember the features of the common conditions. These are clubfoot
(hindfoot equinus and varus), pes planovalgus (hind foot and fore foot valgus, not equinus),
and pes cavus (hindfoot valgus).

261
Q

A 50-year-old woman presents with a mass on the plantar aspect of her right foot. Ultrasound
reveals a small oval-shaped lesion between the plantar portions of the metatarsal heads. MRI
characteristics of the lesion are low-to-intermediate signal on T1 and low signal intensity on
T2. Which of the following is the most likely diagnosis? [B2 Q19]

a. Lipoma
b. Morton’s neuroma
c. Plantar fibromatosis
d. Giant cell tumour of the tendon sheath
e. Ganglion cyst

A

Morton’s neuroma

Low-intermediate on T1 and low on T2

The description is that of a Morton’s neuroma. This occurs most commonly in the third
metatarsal space
and less commonly in the second space. There is often an associated
metatarsal bursitis which is a high signal on STIR imaging
. Ultrasound is usually the first
imaging modality; squeezing the metatarsal heads together during scanning will usually make
the lesion more prominent.

262
Q

A 35-year-old woman with moderate hallux valgus deformity, complains of pain between the
second and third toes of left foot. Ultrasound shows a 1 cm hypoechoic lesion in the region of
the distal intermetatarsal heads of the second and third toes. This is non-compressible and
shows no significant vascularity. The most likely diagnosis is? [B5 Q19]

(a) Bursitis
(b) Morton’s neuroma
(c) Ganglion from metatarsophalangeal joint
(d) Tenosynovitis from the flexor tendons
(e) Synovial sarcoma

A

Morton’s neuroma

This is the typical ultrasound appearance of a Morton’s neuroma. Treatment is conservative or
surgical removal. On MRI the lesion appears as intermediate signal intensity on T1 and T2.

Bursitis is compressible on ultrasound probe pressure

263
Q

A 56-year-old woman slips off the pavement onto the road and her outstretched foot is run over
by a passing car. She has immediate severe midfoot pain. Plain radiographs taken on arrival at
the emergency department confirm a Lisfranc fracture dislocation of the midfoot. Which two
bones does the Lisfranc ligament attach to? [B2 Q36]

a. First metatarsal and intermediate cuneiform
b. First metatarsal and medial cuneiform
c. Second metatarsal and medial cuneiform
d. Second metatarsal and intermediate cuneiform
e. First and second metatarsals to the medial and intermediate cuneiforms

A

Second metatarsal and medial cuneiform

The Lisfranc ligament attaches between the second metatarsal and medial cuneiform, which is
why an injury to this ligament allows the second to fifth metatarsals to drift laterally once they
have lost this stabilisation. This is therefore an unstable injury and requires rapid
immobilisation. This is a vital injury to detect as long-term sequelae will often result from a
delayed diagnosis.

264
Q

A 24-year-old man has injured his right ankle playing football. The A&E SHO has asked your
opinion on the plain radiographs. These show a widening of the medial joint space on the AP
ankle view but no evidence of fracture, and an oblique fracture of the proximal shaft of the
fibula. This is the typical appearance for which of the following fractures? [B2 Q52]

a. Weber B
b. Maisonneuve
c. Pilon
d. Dupuytren’s
e. Fibula stress fracture

A

Maisonneuve

This is the description of a Maisonneuve fracture (sometimes classified as Weber C3). This
injury is often overlooked as the patient may complain only of ankle pain and hence a full
tibia/fibula plain film is not taken. This fracture is often associated with ligamentous injury at
the ankle, most usually of the anterior talofibular ligament and the postero-inferior talofibular
ligament

265
Q

Which of the following is associated with anteromedial ankle impingement syndrome? [B3
Q21]

A. Usually occurs from eversion injury only
B. Well-defined signal intensity on T1 and T2 in the deep deltoid ligament related to scarring
C. Large corticated ossicles are seen
D. Lateral displacement of the tibialis posterior tendon
E. Post-traumatic synovitis

A

E.

Anteromedial ankle impingement syndrome was previously thought to be from eversion injury,
but recent studies are showing inversion is the causative injury. There is associated amorphous
SI in ATTL with heterotopic bone formation and small corticated ossicles. Post-traumatic
synovitis and fibrosis are recognised. The tibialis posterior tendon may be displaced medially

266
Q

An MRI of the ankle shows deep injury to the deltoid ligament. Which of the following belong
to the deep components of the deltoid (medial collateral) ligament of the ankle? [B2 Q22]

A. Tibiocalcaneal ligament
B. Tibionavicular ligament
C. Posterior superficial tibiotalar ligament
D. Anterior tibiotalar ligament (ATTL)
E. Tibiospring ligament

A

D The other answers all belong to the superficial components. The posterior deep tibiotalar
ligament is also deep. The superficial and deep components function almost synergistically and
stabilise against valgus and pronation as well as rotational force against the talus

267
Q

A 20-year-old long-distance runner has a several-week history of right lower leg pain. A plain
film was reported as normal, but CT showed multiple areas of osteopenia and cavities of the
anterior tibial cortex. Which is the diagnosis? [B3 Q34]

A. Stress fracture
B. Chronic external compartment syndrome
C. Medial tibial stress syndrome
D. Periostitis
E. Interosseous membrane injury

A

Medial tibial stress syndrome

A spectrum of lesions can occur from repetitive stress. These include periostitis, cortical
osteopenia, cancellous bone, and cortical fractures. Injuries are most frequently in the cortex
of the distal 2 ⁄ 3 of the tibiae and known as medial tibial stress syndrome.

268
Q

A 35-year-old woman with bilateral forefoot pain is investigated for Morton’s neuromas.
Which is the single best answer? [B3 Q40]

A. Low on T1 and high on T2
B. Do not enhance with contrast
C. Associated proximal fluid-filled bursae are seen
D. Occur most commonly between the heads of the first and second metatarsals
E. Occur most commonly between the heads of the second and third metatarsals

A

Associated proximal fluid filled bursae are seen.

Most commonly between third and fourth metatarsal heads

269
Q

Which of the following is a characteristic of plantar fasciitis?

A. Calibre of plantar fascia > 2mm
B. Increased reflectivity of ligament
C. Enthesal new bone formation
D. Low SI T1, high SI T2
E. Pain typically worse with progressive exercise

A

Enthesal new bone formation

Most common type of plantar fascia injury. Sharp pain, worse after rest, lessening with exercise.
US shows increased calibre > 4mm, loss of reflectivity of the ligament, specifically within the
central bundle. MR shows high to intermediate T1 and high T2 SI

270
Q

A 49-year-old woman presents to her general practitioner with a history of mild midfoot pain
exacerbated by walking and wearing tight shoes. Ultrasound scan demonstrates a hypoechoic,
7 mm, rounded lesion lying in the third tarsal interspace. The lesion is poorly demonstrated on
MRI, returning intermediate T1 and low T2 signal. Which of the following conditions best
explains these findings? [B4 Q38]

a. tendon sheath ganglion
b. tendon sheath giant cell tumour
c. synovial cyst
d. Morton’s neuroma
e. paraganglioma

A

Morton’s neuroma

Morton’s or interdigital neuroma is a benign lesion consisting of perineural fibrosis that entraps
a plantar digital nerve. It is frequently asymptomatic, and women represent 80% of cases.
Clinical presentation is with foot pain exacerbated by walking, and symptomatic lesions are
surgically excised. They are not usually demonstrated on plain radiography and are poorly seen
on MRI, returning intermediate T1 and low T2 signal (like surrounding tissues). Typical
ultrasound appearances are of a hypoechoic rounded lesion, larger in the axial than the sagittal
plane. Ganglia and cysts would return high signal on T2W images, and pathology arising from
the tendon sheath itself can also show high signal. Giant cell tumours are usually painless

271
Q

A young male patient sustains an external rotational injury to his left ankle and is unable to
bear weight. A plain radiograph of the ankle performed in accident and emergency shows no
fracture but does show soft-tissue swelling over the medial malleolus and widening of the ankle
joint space medially (lateral talar shift). Which of the following additional view(s) should be
performed? [B4 Q48]

a. mortise view
b. calcaneus
c. foot
d. knee
e. contralateral ankle

A

Knee

The Maisonneuve fracture is a spiral fracture of the upper third of the fibula associated with a
tear of the distal tibiofibular syndesmosis and the interosseous membrane. The medial
component of the injury may be an associated fracture of the medial malleolus or rupture of
the deep deltoid ligament. The ankle joint is effectively a bony ring that extends up to the knee.
Interruption of the ring in this way allows lateral displacement of the fibula and so disruption
of the congruence of the ankle mortise, resulting in an unstable ankle injury that requires
surgical fixation.

272
Q

Tarsal coalition is a common cause of foot pain. Which of the following joints is most affected?
[B4 Q49]

a. anterior subtalar
b. middle subtalar
c. posterior subtalar
d. calcaneo navicular
e. calcaneocuboid

A

Calcaneo navicular

MRI is valuable in the diagnosis of several musculoskeletal conditions of the ankle, including
osteochondral lesions of the talus, bone infarcts and bruising, stress fractures, osteoid osteoma
and tarsal coalition. Forty-five per cent of tarsal coalition occurs at the calcaneo-navicular joint,
with a further 45% at the subtalar joint, most commonly involving the middle facet.
Radiographic findings include joint space narrowing, indistinct articular margins, elongation
of the anterior calcaneus, a hypoplastic talus and reactive sclerosis of the involved bones. It is
commonly associated with pes planus. Treatment options include physical supports, anti-
inflammatory medication, local steroid injection, and surgical resection or arthrodesis

273
Q

On MRI of the foot performed for non-specific pain, which single feature is most specific for
a diagnosis of sinus tarsi syndrome? [B4 Q71]

a. subtalar joint effusion
b. subtalar sclerosis
c. loss of fat signal in the sinus
d. bone marrow oedema in the talus
e. flexor tendon high signal on T2W images

A

Loss of fat signal in the sinus

Sinus tarsi syndrome is a common complication of ankle sprains but may also result from an
inflammatory arthropathy. It is associated with abnormalities of one or more structures in the
tarsal sinus and tarsal canal that led to pain and a feeling of instability of the hindfoot. Most
patients with this syndrome present in the third or fourth decade of life with persistent lateral
foot pain, though the pathogenesis of the condition is poorly understood. Conventional
radiography generally is not valuable, but on MRI there is alteration of the fat signal, the most
common changes being diffuse low-signal-intensity infiltration on both T1W and T2W images.
Other common MR findings include synovial thickening and diffuse enhancement of the tarsal
sinus following intravenous gadolinium.

274
Q

A 55-year-old housewife attended her GP with a gradually growing soft tissue swelling on the
dorsum of her foot for 1 year. The swelling is tender and mobile in a side-to-side direction.
Ultrasound shows a 4 cm hyper-vascular lesion on the dorsum of the foot between the tendons
of extensor hallucis and extensor digitorum. MRI shows that the lesion is bright on STIR and
intermediate signal on T1
. It shows homogenous enhancement with gadolinium. What is the
most likely diagnosis? [B5 Q2]

(a) Soft tissue ganglion
(b) Peripheral nerve sheath tumour
(c) Lipoma
(d) Liposarcoma
(e) Callus from a previous fracture.

A

Peripheral nerve sheath tumour

The location and direction of mobility along with typical MRI features clinch the diagnosis of
peripheral nerve sheath tumour.

Ganglion, lipoma and callous would not appear as hyper-vascular lesions. Liposarcoma can
show increased vascularity but demonstrates heterogenous enhancement on MRI.

275
Q

A 51-year-old man with a palpable nodule on the planter aspect of the foot. Ultrasound shows
a 2 cm, vascular and hypoechoic lesion within the mid part of the plantar fascia. T he most
likely diagnosis is? [B5 Q30]

(a) Plantar lipoma
(b) Plantar fibromatosis
(c) Ganglion cyst
(d) Accessory muscle
(e) Haemangioma

A

Plantar fibromatosis

This presents as nodular thickening in the plantar fascia. There can be single or multiple lesions.
On MRI, the lesion is low signal on T1 and mild hyperintensity on T2, and the nodule enhances
with gadolinium.

276
Q

A 60-year-old man presents to the A&E department with acute onset lower back pain following
a relatively minor fall. A plain film reveals a collapse of the L4 vertebral body against a
background of osteopenia. He has a history of renal cell carcinoma and the clinical team request
an MRI to ‘rule out metastatic disease’. Which of the following features would most suggest a
malignant rather than a benign cause for a vertebral compression fracture? [B1 Q14]

A. Isointense signal to adjacent vertebral bodies on T2WI.
B. A band-like area of low signal adjacent to the fractured endplate on T1WI.
C. High signal intensity adjacent to the vertebral endplate on STIR imaging.
D. Retropulsion of a posterior fragment into the spinal canal.
E. A convex bulge involving the whole of the posterior cortex of the vertebral body.

A

A convex bulge involving the whole of the posterior cortex of the vertebral body.

The others are more in keeping with benign compression fractures. Retropulsion of a posterior
fragment into the spinal canal is a highly specific (100%) finding of benign compression
fracture but has a sensitivity of only 16%. Other features in keeping with malignant
compression fractures are complete replacement of normal marrow with low signal on T1WI,
involvement of the pedicles, and the presence of an epidural and/or paraspinal soft-tissue mass.
The presence of an epidural mass is said to have 80% sensitivity and 100% specificity for
malignant fractures. Convex bulging of the posterior cortex of the vertebra and involvement of
the pedicle have respective sensitivities and specificities of 70% and 94%, and 80% and 94%.
Beware that compression fractures due to multiple myeloma only rarely show MRI features of
malignant fracture and this diagnosis should be included in the differential of a non-traumatic,
benign-appearing vertebral compression fracture.

277
Q

A 74-year-old woman presents with back pain and no history of recent trauma. Lateral plain
radiographs show partial collapse of the L2 vertebral body. Which of the following findings
would be more suggestive of osteoporotic collapse than malignancy? [B2 Q17]

a. Complete replacement of the normal marrow signal within the vertebral body on T1 imaging
b. Bilateral pedicular involvement with expansion of the right pedicle
c. Bulging and convex appearance to the vertebral body
d. Nodular irregular epidural mass
e. Intervertebral vacuum phenomenon

A

Intervertebral vacuum phenomenon

Intervertebral vacuum phenomenon is highly specific for osteoporotic collapse, although it is
not common. The other features are all more suggestive of malignancy than osteoporotic
collapse. Pedicular destruction occurs in 50% of cases of malignant collapse but in less than 1%
of osteoporotic collapse.

278
Q

OnMRIofthe spine demonstrating vertebral body collapse, which additional feature favours an
underlying diagnosis of malignancy rather than osteoporosis? [B4 Q81]

a. bone fragment retropulsion
b. focal low signal in the vertebral body on T1W images
c. diffuse intermediate signal in the vertebral body on T2W images
d. no enhancement with gadolinium
e. convex posterior border to the vertebral body

A

Convex posterior border to the vertebral body

A convex bulge involving the whole posterior border of the vertebral body strongly suggests
vertebral body expansion by tumour invasion and is only very rarely a feature of osteoporosis.
Other findings on MRI suggestive of malignancy include a soft-tissue mass, involvement of
the pedicles, and heterogeneous high signal on T1W post-contrast or T2W images.
Retropulsion of bone fragments, focal T1 low signal or an isointense appearance on T1W or
T2W images suggests osteoporotic collapse.

279
Q

A 70-year-old patient complains of back pain and leg pain after walking 50 metres. Plain
radiographs show an anterior slip of L4 relative to L5. The spinous process of L4 is also noted
to have moved anterior to the L5 spinous process. What type of spondylolisthesis does this
represent? [B2 Q32]

a. Traumatic
b. Degenerative
c. Spondylolytic
d. Dysplastic
e. Pathological

A

Degenerative

This is the classical description of the symptoms and radiology of a degenerative
spondylolisthesis. Degenerative spondylolisthesis is usually symptomatic due to spinal stenosis
and narrowing of neural foramen. This most commonly occurs at the L4/5 level. Spondylolysis
is a defect in the pars interarticularis between superior and inferior articulating processes

280
Q

A 55-year-old woman with breast cancer and back pain undergoes investigation with MRI.
Which of the following indicates a malignant rather than osteoporotic vertebral fracture? [B3
Q5]

A. Convex posterior cortex
B. Normal Signal Intensity (SI) on all sequences
C. Retropulsion of a posterior bone fragment
D. Band-like low SI adjacent to the fracture
E. Homogenous ‘return to normal’ SI after contrast

A

Convex posterior cortex

B-E are osteoporotic features. Other malignant features include diffuse low SI on T1,
high/heterogeneous SI on T2, and involvement of the posterior elements.

281
Q

```

~~~

A 52-year-old woman presents to her GP with a longstanding history of lower back pain which
has suddenly worsened in severity over the past few days. An urgent MRI scan of the lumbar
spine shows a right paracentral disc protrusion at the L4/L5 level. The disc impinges on the
lateral recess at this level. The most likely nerve to be affected is the: [B2 Q26]

a. Cauda equina
b. Lumbar plexus
c. Right L4
d. Right L5
e. Right S1

A

Right L5

The right L5 nerve root is the most likely to be affected as it will be sitting in the right lateral
recess at the L4/5 level. The L4 nerve root will be at the exit foramen and therefore if the
protrusion affects only the lateral recess, then this nerve will already have exited and therefore
not be affected.

282
Q

You receive a referral while on call from the orthopaedic consultant regarding a middle-aged
woman with a long history of simple back pain. She has attended accident and emergency
complaining of worsening lower lumbar pain with a several-hour history of progressive urinary
retention, faecal incontinence, saddle anaesthesia and mild bilateral leg weakness. Which
method of imaging would you recommend as most appropriate? [B4 Q11]

a. plain radiography
b. myelogram
c. CT
d. CT myelogram
e. MRI

A

MRI

Bilateral lower limb involvement suggests a myelopathy rather than a radiculopathy. The
presence of urinary and bowel symptoms and saddle anaesthesia suggests compression of
lumbosacral nerve roots. This complex of symptoms is cauda equina syndrome and is
considered an orthopaedic emergency because of the likelihood of permanent neurological
impairment, particularly affecting the autonomic supply to the bladder or bowel, which can
result in permanent incontinence if surgery is delayed. The Royal College of Radiologists
recommends proceeding straight to MRI in patients who have ‘red flag’ signs.

283
Q

In a patient who presents with acute femoral nerve radiculopathy, which of the following MRI
sequences is the most useful in the diagnosis of a far lateral upper lumbar vertebral disc
protrusion? [B4 Q18]

a. sagittal STIR
b. sagittal T1
c. sagittal T2
d. axial STIR
e. axial T2

A

Axial T2

The far lateral disc protrusion is the least common type of symptomatic disc herniation. It
distinguishes itself from the posterolateral herniation in that the disc ruptures outside the spinal
canal, lateral to the root foramen. The disc, instead of tethering the traversing nerve root,
compresses the more rostral nerve root that has already exited the root foramen. The
neurological symptoms therefore correspond to a lesion at the upper disc level, often leading
to confusion in the diagnosis. It is also difficult to diagnose radiologically, as the far lateral
location is usually not detected on the sagittal images but only on axial images. STIR is an
inversion recovery sequence that suppresses fat and so highlights areas of increased fluid.
However, it is not sensitive when the herniation is outside the fluid-filled spinal canal; therefore,
the T2W gradient echo sequence is better at detecting far lateral disc herniation.

284
Q

On lateral radiographs of the thoracolumbar spine, a central anterior beak of the vertebral
bodies is most likely to suggest which of the following conditions? [B4 Q29]

a. Scheuermann’s disease
b. Morquio’s syndrome
c. Hurler’s syndrome
d. Down’s syndrome
e. achondroplasia

A

Morquio’s syndrome

Morquio’s syndrome is type IV and the most common of the mucopolysaccharidoses, a family
of inherited disorders of metabolism. A central vertebral beak is relatively specific for the
condition. Other spinal manifestations include odontoid hypoplasia with atlantoaxial
subluxation (which can be life threatening), platyspondyly, ovoid vertebral bodies, widened
intervertebral disc space and exaggeration of the normal lumbar lordosis. Other skeletal
findings include dwarfism, as well as skull, face, and appendicular abnormalities. Hurler’s
syndrome belongs to the same family of disorders but has an inferior vertebral beak, which is
also seen in achondroplasia and Down’s syndrome. Scheuermann’s disease does not show
vertebral beaking.

285
Q

In reviewing a fracture of the spine at the thoracolumbar junction in a major trauma case, which
single indicator on CT is most sensitive for inferring instability? [B4 Q67]

a. widened facet joints
b. two-column malalignment
c. soft-tissue swelling
d. rotational abnormality
e. increased intervertebral disc space

A

Two-column malalignment

The spine can be divided anatomically into three columns: the anterior column contains the
anterior longitudinal ligament, anterior half of the vertebral body and anterior annulus fibrosus;
the middle column contains the posterior half of the vertebral body, posterior longitudinal
ligament, and the posterior annulus fibrosus; and the posterior column contains the posterior
elements of the spine, facet joint capsule and interspinous ligaments. Two intact columns are
required for intrinsic spinal stability. Disruption of two columns can therefore be used to infer
instability. Usual traumatic patterns are anterior and middle, or posterior and middle, disruption Isolated middle column interruption can occur after trauma or surgery, or as a congenital
abnormality, and is also considered potentially unstable.

286
Q

A50-year-old mechanic with a long history of back pain presents to the spinal clinic
complaining of sudden onset of numbness and pain over the right lateral calf and dorsum and
sole of the right foot following heavy lifting. Which of the following spinal pathologies is most
likely to explain the patient’s symptoms? [B4 Q76]

a. lumbar spinal stenosis
b. paracentral L4–5-disc protrusion
c. paracentral L5–S1 disc protrusion
d. far lateral L4–5-disc protrusion
e. central L5–S1 disc protrusion

A

Paracentral L4/5 disc protrusion

Degenerative disc disease of the spine is one of the leading causes of functional incapacity and
chronic disability in the working population, affecting both men and women. Although there
is no universally established nomenclature for describing disc herniation, ‘protrusion’ is
commonly used if the herniation is broader than it is deep and ‘extrusion’ if it is deeper than it
is broad. A disc ‘bulge’ is used to describe a herniation that is very broad based and may even
be circumferential, with a generalised disc bulge being one that affects at least half of the
periphery. As a result of the strong posterior longitudinal ligament, posterior disc herniation is
often paracentral, i.e. to the side of the midline. This can result in compression of the transiting
nerve root in the lateral recess, which is the one that will exit at the level below. A lateral disc
herniation narrowing the neural foramen compresses the exiting nerve root. Therefore, for a
given intervertebral disc, a paracentral herniation will affect the nerve that exits one level below,
whereas a lateral protrusion affects the nerve root at that level.

287
Q

Which of the following is not an appropriate indication for percutaneous
polymethylmethacrylate cement vertebroplasty? [B4 Q81]

a. progressive osteoporotic deformity
b. painful osteoporotic collapse
c. painful haemangioma
d. painful osteoid osteoma
e. painful metastases

A

Painful osteoid osteoma

Percutaneous cement vertebroplasty is a treatment for vertebral compression fractures that
involves the injection of acrylic bone cement into the vertebral body to relieve pain, stabilize
fractured vertebrae or, in some cases, restore vertebral height. Current guidelines from the
National Institute for Health and Clinical Excellence (NICE), regarding the use of
vertebroplasty in the UK, state that it may be used for pain relief in patients with severe painful
osteoporosis with loss of height, compression fractures of the vertebral body, symptomatic
vertebral haemangioma, and painful vertebral body tumours (metastases or myeloma). Review
of current evidence indicates some level of pain relief in 58–97% of patients.

288
Q

A 63-year-old female is being worked up for a left total hip replacement. She has a history of
RA. As part of the routine pre-operative assessment in your hospital a cervical spine radiograph
is requested. This demonstrates that there is widening of the pre-dental space, with the anterior
arch of C1 located anterior to the lower part of the body of C2. The dens is not clearly visible.
This appearance is constant on the flexion view. The patient is asymptomatic. What do you
think these findings represent? [B1 Q4]

A. Degenerative change.
B. Pannus erosion of dens.
C. Atlanto-axial subluxation.
D. Erosion of the occipital condyles.
E. Atlanto-axial impaction.

A

Atlanto-axial impaction.

This is a more severe form of atlanto-axial subluxation where the C1-2 facets collapse and
there is invagination of the dens of C2 into the foramen magnum. As such, the dens is not
visible on the lateral radiograph. The key feature, apart from widening of the pre-dental space
(which can also be caused by pannus eroding the dens or more commonly atlanto-axial
subluxation), is that the anterior arch of C1 lies in front of the lower portion of C2, whereas it
normally lies anterior to the dens

289
Q

A 56-year-old motorcyclist has a trauma series of plain films following a road traffic accident.
On evaluation of the lateral cervical spine film, which of the following soft tissue parameters
would be a concerning feature? [B2 Q3]

a. Predental space of 3 mm
b. Nasopharyngeal space of 7 mm
c. Retropharyngeal space of 10 mm
d. Retro-tracheal space of 20 mm
e. Decreased disc space at the C5/6 level

A

Retropharyngeal space of 10 mm

This is too wide for the retropharyngeal space. The correct acceptable limits for soft-tissue
measurements are as follows:

  • Predental space 3 mm in adults, 5 mm in children.
  • Nasopharyngeal space (anterior to C1) 10 mm.
  • Retropharyngeal space (C2–C4) 5–7 mm.
  • Retro- tracheal space (C5–C7) 22 mm.

Disc spaces should be roughly equal throughout the cervical spine. Narrowing of a disc space
is usually due to degenerative change but widening would be a more concerning feature.

290
Q

A 23-year-old man sustains a Jefferson fracture to his cervical spine following an injury in
which he dived into a shallow swimming pool, hitting his head on the bottom. Which of the
following regarding his injury is incorrect? [B2 Q18]

a. Displacement of the lateral masses of C1 relative to the dens on an odontoid view indicates
a transverse ligament rupture
b. Associated C2 fracture will be present in up to 30% of cases
c. Jefferson fractures are usually associated with a neurological deficit
d. Up to 50% are associated with a further cervical spine injury
e. There may be associated vertebral artery injury

A

Jefferson fractures are usually associated with a neurological deficit

Jefferson fractures are not usually associated with neurological deficit. Although there may be
retropulsion of fragments into the vertebral canal, spinal cord injury is rare due to the large
dimensions of the canal at this level. Vertebral artery injury, however, must be considered and
if there is concern either CTA or MRA imaging should be considered

291
Q

A 70-year-old man presents after falling five stairs and sustaining injury to the neck. An open-
mouth view shows increased space between the dens and medial border of lateral masses of
C1. CT shows fracture of the anterior and posterior arch of the C1 vertebra. What is the most
likely diagnosis? [B5 Q43]

(a) Hangman’s fracture
(b) Clay shoveller’s fracture
(c) Jefferson fracture
(d) Extension teardrop fracture
(e) Flexion teardrop fracture

A

Jefferson fracture

Jefferson fracture involves the C1 vertebra. There is a comminuted fracture of the C1 ring, at
least through two places. Plain radiography using an open-mouth view demonstrates lateral
displacement of the lateral masses.

292
Q

A 25-year-old male is involved in a 60-mph road traffic accident with a head-on collision. He
was wearing a seatbelt, but his car did not have an air-bag. A screening lateral radiograph of
the cervical spine shows the following findings: an angular kyphosis centred at C4/C5, a 1mm
anterior slip of C4 on C5, and widening of the interspinous space posteriorly. What is the likely
mechanism for this injury? [B2 Q32]

a. Lateral compression
b. Flexion
c. Extension
d. Combination
e. Rotation

A

Flexion
This describes the typical appearance for a flexion injury as well as the typical mechanism.
This would represent a potentially unstable fracture and immobilisation would be essential
until further management decisions are made. Flexion teardrop injuries are more common in
the lower cervical spine and extension teardrop injuries are more common in the upper cervical
spine

293
Q

An A&E SHO has asked you to review a paediatric cervical spine plain film which has been
performed on a child who has been involved in a road traffic accident. He is unsure as to
whether the appearances are normal for a paediatric cervical spine film. Which of the following
findings is more likely to represent a true cervical spine injury than a normal variant? [B2 Q51]

a. Absence of usual cervical lordosis
b. Widening of the prevertebral soft tissues in expiration
c. Increased distance between the tips of the C1 and C2 spinous processes in flexion
d. Wedging of the anterior aspect of the C3 vertebral body
e. A 7 mm gap between the occipital condyles and the condylar surface of the atlas

A

A 7 mm gap between the occipital condyles and the condylar surface of the atlas

This is highly suggestive of cranio-cervical injury; these injuries are often fatal and are often
caused by sudden deceleration. Radiologic evaluation of this injury can be difficult but is
crucial in determining further management. The remainder of the findings above can all be
normal variants in the paediatric cervical spine and therefore should be interpreted with care.

294
Q
A
295
Q

On review of a casualty film of a patient involved in a road traffic accident, which of the
following is an unstable cervical spine fracture? [B3 Q47]

A. Unilateral facet dislocation
B. Clay shovellers
C. Jefferson
D. Burst
E. Extension tear drop

A

Jefferson

Unstable fractures include bilateral facet dislocation, flexion teardrop, hangman’s,
hyperextension dislocation, Jefferson, odontoid and atlanto-occipital dislocation.

296
Q

CT of the cervical spine is performed on an intubated emergency patient who was a restrained
driver in a high-speed motor vehicle collision. This reveals bilateral C2 pedicle fractures. What
is the most likely underlying mechanism of injury? [B4 Q4]

a. hyperflexion and rotation
b. hyperextension followed by hyperflexion
c. axial loading
d. hyperextension and traction
e. hyper-rotation

A

Hyperextension and traction

The fracture described is a hangman fracture. This involves either the pedicles or pars
interarticularis of C2 bilaterally. The mechanism is usually extension and traction (as caused
during hanging). Hyperflexion injuries produce anterior tear-drop or of a vertebral body wedge
fractures. Axial loading can produce a burst fracture of C1 (Jefferson’s fracture) or a vertebral
body elsewhere in the spine. Hyperflexion and extension are associated with longitudinal
ligament injury. Hyper-rotation is associated with soft-tissue injury or facet joint dislocation

297
Q

A child passenger is admitted to accident and emergency following a road traffic collision.
Radiographs of the spine show a horizontal fracture involving the vertebral body and pedicles
of L2. Associated injury to which of the following abdominal organs is most likely? [B4 Q7]

a. duodenum
b. pancreas
c. spleen
d. liver
e. rectum

A

Duodenum

The spinal injury described is a Chance fracture, a fracture through the vertebral body and
pedicles caused by hyperflexion, therefore causing compression of the spine anteriorly and
distraction posteriorly. This injury typically occurs in back-seat passengers wearing a lap seat
belt during a road traffic collision. In children, there is a 50% incidence of associated intra-
abdominal organ injury. Retroperitoneal organs are most vulnerable, being closest to the spine.
Duodenal injuries are most common and have a significant associated mortality. The pancreas
is also commonly injured due to its retroperitoneal location.

298
Q

On plain radiographs of the neck in a 60-year-old man, which feature is most likely to support
a diagnosis of diffuse idiopathic skeletal hyperostosis rather than ankylosing spondylitis? [B4
Q26]

a. enthesopathy
b. confluent intervertebral bony bridging
c. sparing of the posterior elements
d. sparing of the sacroiliac joints
e. changes limited to the thoracic spine

A

Sparing of the sacro-iliac joints

Diffuse idiopathic skeletal hyperostosis (DISH) is an ankylosing disorder of the spine. It is
most seen in the thoracic region but may involve cervical and lumbar regions. Diagnostic
criteria are:
Fowing calcification along the anterolateral border of at least four vertebral bodies
Relative preservation of intervertebral disc height
Absence of sacroiliac joint involvement

These three-criteria aid differentiation of spondylosis deformans, intervertebral osteochondromatosis and ankylosing spondylitis respectively.

Extra-spinal manifestations of DISH include Achilles tendinosis, tennis elbow, calcaneal and olecranon enthesopathy and dysphagia. Whiskering is seen radiographically at tendinous insertions, particularly of the pelvis.

299
Q

A patient undergoes skull radiographs for a suspected occipital depressed skull fracture after
falling backwards onto the pavement. You receive non-angulated AP frontal and lateral views.
Which additional view would you request from the radiographer to detect an occipital bone
skull fracture? [B4 Q77]

a. orthopantomogram
b. submentovertical
c. Towne’s
d. Caldwell’s
e. Water’s

A

Towne’s

In modern radiology departments with the widespread availability of CT, the indications for
skull radiograph are few, and it has little place in the evaluation of brain injury. Standard
practice will usually comprise a lateral and a PA frontal view. The frontal view may be angled
to show different areas of anatomy. Towne’s view is an AP projection through the frontal and
occipital bones (30’ caudal tube tilt) and is mainly used to show the occipital bone. The
Caldwell view is a similar AP projection but with only 15’ of caudal tube tilt to allow evaluation
of the orbits. An occipitomental (and therefore PA, unlike Towne’s or the Caldwell) projection
is called Water’s view. It is used to demonstrate the facial bones and sinuses and may be
angulated to highlight the zygomatic arches. Further angulation results in a submento-vertical
view, which is also used to evaluate the zygomatic arches. This requires extension of the neck
and is therefore contraindicated in patients with suspected cervical spinal injuries. The
orthopantomogram is a panoramic view of the mandible used primarily in dental radiography
and to evaluate the whole of the mandible in a single exposure.

300
Q

Which of the following can cause a false-negative result in performing an ultrasound scan of
the shoulder for suspected rotator cuff tear? [B4 Q97]

a. rotator interval
b. musculotendinous junction
c. limited joint mobility
d. anisotropy
e. acoustic shadowing

A

Limited joint mobility

A false-negative result in this context is failure to identify pathology and to report incorrectly
the ultrasound examination as normal. Limited shoulder mobility will not permit correct
positioning of the shoulder for best interrogation of the whole of each tendon and may lead to
non-visualization of a tear. Other causes of false negatives include technical factors such as
using an incorrect transducer (should be at least 7.5 MHz), poor focusing and poor transducer
handling. There are also anatomical causes, including non-diastasis of the tendon fibres, scar
tissue, bursitis, tendinosis, and massive tear, with complete retraction of the tendon ends
preventing their visualization. Anisotropy, poor transducer positioning or misinterpretation of
the rotator interval, musculotendinous junction, supraspinatus–infraspinatus interface, acoustic
shadowing and fibrocartilaginous insertion can all give rise to false-positive findings.

301
Q

Radiographic arthrography of the shoulder with injection of contrast into the glenohumeral
joint is performed for a painful joint with a globally reduced range of movement. Which single
finding is most likely to indicate a diagnosis of adhesive capsulitis? [B4 Q85]

a. pain on injection of contrast
b. small axillary recess
c. contrast tracking along the subscapularis muscle
d. contrast in the subacromial space
e. obliteration of the sub-coracoid fat

A

Small axillary recess

Adhesive capsulitis or frozen shoulder is clinically characterized by restriction of both active
and passive elevation and external rotation. Patients are commonly 40–70 years old and
predominantly female. It may be idiopathic, preceded by trauma, or associated with diabetes
mellitus or other conditions. Patients have been shown to have a significantly thickened
coracohumeral ligament and joint capsule, and an axillary recess significantly reduced in
volume. Obliteration of the fat triangle between the coracohumeral ligament and the coracoid
process is specific when seen on MR arthrography. Treatment options include physiotherapy,
intra-articular corticosteroid injection, manipulation under anaesthetic and surgical
capsulotomy.

302
Q

In a 65-year-old woman with a fracture of the neck of the humerus, which of the following
classification systems to describe the fracture would be useful in guiding the surgical
management? [B2 Q5]

a. Garden classification
b. Neer classification
c. Weber classification
d. Fryman system
e. Crosby–Fitzgibbon system

A

Neer classification

The Neer classification system is used to grade humeral neck fractures. This system describes
four parts – greater tuberosity, lesser tuberosity, humeral head and shaft of humerus. According
to Neer, a fracture is displaced if there is more than 1 cm of displacement and 45 angulation
between any two segments. Two-part fractures involve any of the four parts and include one
fragment that is displaced. Three-part fractures include a displaced fracture of the surgical neck
in addition to either a displaced greater tuberosity or lesser tuberosity fracture. Four-part
fractures include displaced fractures of the surgical neck and both tuberosities.

303
Q

A 21-year-old rugby player presents to the A&E department with right shoulder pain and
decreased range of movement following a tackle. There is obvious contour deformity on
examination. Plain radiographs confirm anterior dislocation. Which additional radiographic
finding is in keeping with a Hill–Sachs deformity? [B1 Q30]

A. Intra-articular loose body.
B. Greater tuberosity fracture.
C. Anterior glenoid rim fracture.
D. Anterior humeral head indentation.
E. Posterior humeral head indentation.

A

Posterior humeral head indentation.

Anterior dislocation occurs when the arm is forcibly externally rotated and abducted.
Radiographically, the humeral head lies inferior and medial to the glenoid on the AP view. The
Hill–Sachs deformity is an indentation on the posterosuperior portion of the humeral head and
indicates a greater likelihood of recurrence. A Bankart deformity is a bony fragment off the
inferior glenoid. Anterior humeral head indentation is a ‘reverse Hill–Sachs’ deformity seen in
posterior dislocations

304
Q

A 56-year-old woman is referred for MR arthrography of her right shoulder for query rotator
cuff tear. You are asked to explain the procedure to a group of medical students attached to the
department. What is the advantage of using a fat suppressed T1WI sequence? [B1 Q45]

A. Differentiating partial from full thickness tear.
B. Identify bursal fluid collections.
C. Differentiating inadvertent air injection from intra-articular loose body.
D. Diagnosing capsular laxity.
E. Detecting incidental bone marrow lesions.

A

Differentiating partial from full thickness tear.

MR arthrography is most helpful for outlining labral-ligamentous abnormalities in the shoulder
and distinguishing partial thickness from full thickness tears in the rotator cuff. The technique
involves injection of diluted gadolinium mixed with iodinated contrast, which allows
fluoroscopic confirmation of intra-articular needle placement. Partial and full thickness tears
may not be distinguishable on standard T1WI because fat and gadolinium have similar signal
intensities. This is especially the case when cuff tendons show contrast solution extending to
the bursal surface but not definitively through it. This problem can be overcome with use of fat
suppression. MR arthrography should include a T2WI sequence to identify bursal fluid
collections and tears. T2WI is also helpful in characterizing incidental bone marrow lesions.
Inadvertent injection of gas may lead to a false-positive diagnosis of intra-articular loose bodies,
but gas bubbles will rise to non-dependent regions, whereas loose bodies will gravitate to
dependent locations. No accurate MR imaging criteria are recognized in the diagnosis of
capsular laxity.

305
Q

A 24-year-old rugby player attends A&E following a tackle during which he felt his left
shoulder dislocate. Initial plain radiographs confirm an anterior inferior dislocation of the left
shoulder. Which of the following statements is true? [B2 Q15]

a. The humeral head lies inferior and lateral to the glenoid on the AP view
b. The presence of a Hill–Sachs’s defect indicates previous dislocation
c. Hill–Sachs lesions are more common than Bankart lesions
d. Anterior dislocation accounts for 50% of shoulder dislocations
e. A Hill–Sachs’s lesion affects the inferior aspect of the humeral head.

A

Hill–Sachs lesions are more common than Bankart lesions

A Hill–Sachs lesion affects the postero-superior aspect of the humeral head and whilst it does
often indicate a previous dislocation, this is not necessarily the case, and it can be present after
a single episode. A Bankart lesion affects the inferior glenoid. Almost 95% of all shoulder
dislocations are anterior.

306
Q

A 72-year-old woman presents to her GP with pain in her right shoulder which is worse on
movement. Plain films of the right shoulder show loss of subacromial space and superior
subluxation of the humeral head. She is referred for an ultrasound with a suspected
supraspinatus tear. Which is the best position of the arm for visualisation of the free edge of
the supra-spinatous tendon? [B2 Q39]

a. Adduction and internal rotation
b. Abduction and internal rotation
c. Adduction and external rotation
d. Abduction and external rotation
e. Flexion and internal rotation

A

Adduction and internal rotation

The best position for visualising the supra-spinatous tendon is with the patient’s arm in
adduction and internal rotation. Often the patient may be asked to place the back of their hand onto their back, or alternatively asking them to simulate putting the hand into the back pocket
of their trousers. The most medial part of the tendon when imaged transversely is the free edge
– this is where most supra-spinatous tears occur.

307
Q

An ultrasound scan of a patient’s left shoulder suggests subacromial impingement. Which is
the single best answer? [B3 Q1]

A. Type 2 concave pattern is the most common morphology
B. Type 2 is the most common type to be associated with rotator cuff tears
C. ACJ degeneration increases the supraspinatus outlet
D. Significant subacromial subdeltoid bursitis is diagnosed with thickness > 10mm
E. Is associated with the presence of bursal fluid lateral to the Acromio-clavicular joint (ACJ)

A

Type 2 concave pattern is the most common morphology

Type 1-flat (12%) Type 2-concave (56%) Type 3-hooked (29%) Type 4-inferior convex (3%)

Type 3 more than Type 2 is associated with increased incidence of rotator cuff tears. ACJ
degeneration can narrow the supraspinatus outlet. Significant subacromial subdeltoid (SASD)
bursitis include thickness > 3mm, presence of bursal fluid medial to the ACJ and presence of
fluid in the anterior aspect of the bursa.

308
Q

A 32-year-old keen tennis player has had shoulder pain for 6 weeks. An MRI confirms a
superior labral tear from anterior to posterior (SLAP) lesion. Regarding these lesions, which of
the following is the single best answer? [B3 Q2]

A. Are isolated tears of glenoid labrum with superior and inferior components
B. Tears are classically located at the biceps anchor
C. Occur with repetitive underarm activity
D. Is often diagnosed clinically alone
E. Begins in the anterior aspect of superior labrum and extends posteriorly

A

Tears are classically located at the biceps anchor

SLAP lesions are isolated tears of the glenoid labrum with anterior and posterior components.
These lesions occur with repetitive over-arm activity, begin in the posterior aspect of the
superior labrum and extend posteriorly.

309
Q

A 27-year-old man presents with posterior instability of the shoulder. Transaxial ts T1 MR
arthrogram shows posterioinferior labrum tear and tear of the posterior scapular periosteum.
Which is the diagnosis? [B3 Q33]

A. Reverse Bankart lesion
B. Bankart lesion
C. Bennett lesion
D. Posterior labrocapsular periosteal sleeve avulsion
E. Humeral avulsion of the posterior glenohumeral ligament

A

Reverse Bankart lesion
The posterior inferior labrum is detected from glenohumeral attachment and there is an
avulsion tear of the posterior scapular periosteum

310
Q

A 30-year-old man undergoes shoulder MRI for chronic anterior pain. There is no history of
trauma. Sagittal images reveal an absent anterior labrum with a thickened middle glenohumeral
ligament. What is the most likely diagnosis? [B4 Q9]

a. anterior labral tear
b. Bankart’s lesion
c. superior labrum anterior-to-posterior (SLAP) lesion
d. glenohumeral tendonitis
e. normal variant

A

Acutal exam question!!

Normal variant

The findings describe the Buford complex, a normal variant present in 1.5% of the population.
It consists of an absent anterior labrum with a thickened cord-like middle glenohumoral
ligament. It can be misdiagnosed as a torn or avulsed anterior labrum, resulting in unnecessary
shoulder arthroscopy.

311
Q

A young adult male sustains an anterior shoulder dislocation while playing rugby. There is no
associated fracture. Following apparently uncomplicated reduction in accident and emergency,
he is unable to abduct the arm and complains of numbness over the upper lateral arm. What is
the most likely cause? [B4 Q55]

a. supraspinatus tendon tear
b. axillary nerve palsy
c. musculocutaneous nerve palsy
d. shoulder impingement
e. deltoid muscle tear

A

Axillary nerve palsy

The axillary nerve is a large terminal branch of the posterior cord of the brachial plexus that
passes into the posterior aspect of the upper arm via the quadrilateral space, where it winds
around the surgical neck of the humerus to supply the deltoid and teres minor muscles. It has a
cutaneous distribution called the ‘regimental badge area’ over the lateral aspect of the deltoid
(where a soldier may wear his regimental badge). Due to its intimate relationship with the
humerus and its passage through the relatively small quadrilateral space, the axillary nerve is
by far the most injured nerve with shoulder dislocation or fractures. As loss of abduction may
be caused by pain rather than deltoid paralysis, it is good practice to assess the sensation in the
cutaneous distribution of the axillary nerve before and after any attempted shoulder reduction

312
Q

A young man is admitted in cardiac arrest following electrocution. Following successful
resuscitation in accident and emergency, he complains of an acutely painful right shoulder with
severely decreased range of movement. What is the most likely plain film finding? [B4 Q60]

a. anterior shoulder dislocation
b. posterior shoulder dislocation
c. acromioclavicular dislocation
d. fractured surgical neck of humerus
e. subacromial impingement

A

Posterior shoulder dislocation

Posterior shoulder dislocation is much rarer than anterior dislocation, accounting for only 5%
of dislocations. It can be caused by direct or indirect force and is most seen following seizure
or electrocution. The internal rotators of the shoulder are stronger than the external rotators,
resulting in internal rotation of the arm if all the shoulder muscles contract simultaneously.
This internal rotation predisposes to posterior dislocation in the same way that external rotation
does for anterior dislocation. Radiographic findings may be subtle on the AP projection and
include superior position of the humeral head relative to the glenoid, external rotation (the
humeral head appears symmetrical like a light bulb), a sharp angle to the scapulohumeral arc
and a compression fracture of the anterior humeral head (a reverse Hill–Sachs lesion)

313
Q

A40-year-old woman presents to the emergency department with a painful, stiff shoulder, 12
hours after undergoing arthrography of the same joint. She describes onset of symptoms 8
hours previously with progressive worsening. She feels otherwise well with a temperature of
37.3’C. There is no overt joint swelling or overlying erythema. What is the most likely cause?
[B4 Q65]

a. septic arthritis
b. chemical synovitis
c. joint haemarthrosis
d. joint effusion
e. allergic contrast reaction

A

Chemical synovitis

Post-arthrography pain due to sterile chemical synovitis is the most common complication of
the procedure, typically beginning after 4 hours and peaking at 12 hours. Other, less common,
immediate and short-term complications include allergic contrast reaction (rare in intra-
articular injections), introduction of infection and vasovagal reaction.

314
Q

A 72-year-old woman presents to the rheumatologist with a long history of shoulder pain
affecting her dominant arm that began at night with associated stiffness but has suddenly
worsened over the past few weeks. Radiographs show a superiorly subluxed humeral head
forming a pseudarthrosis with the acromion, glenohumeral joint space loss, humeral head
collapse with cysts and sclerosis, and periarticular soft-tissue calcification. Ultrasound scan
demonstrates an effusion with widespread degeneration of the rotator cuff and a complete tear
of the supraspinatus tendon. Examination of aspirated joint fluid shows calcium hydroxyapatite
crystals. What is the most likely diagnosis? [B4 Q80]

a. Milwaukee shoulder
b. pseudogout
c. myositis ossificans progressive
d. erosive osteoarthritis
e. scleroderma

A

Milwaukee shoulder
Milwaukee shoulder is a crystal deposition disease of basic calcium phosphate, predominantly
affecting elderly women and resulting in a dysfunctional shoulder from destruction of the
rotator cuff. It is often bilateral but always involves the dominant side. Radiographic findings
include superior subluxation of the humerus due to loss of the superior rotator cuff, often
forming a pseudarthrosis with the clavicle or acromion. Glenohumeral degeneration manifests
as sclerosis and collapse of the humeral head, joint space narrowing and osteophyte formation.
Erosion at the site of rotator cuff insertion and periarticular soft tissue calcification is also a
feature. Examination of effusion fluid is stereotypical, revealing spheroid-shaped aggregates
of hydroxyapatite crystals. The condition is also seen in the knee, where, unlike osteoarthritis,
it predominates in the lateral compartment.

315
Q

A 39-year-old male presents with tenderness and decreased range of movement of the right
elbow after falling on an outstretched arm while playing indoor football. A radial head fracture
is noted on his radiographs, but the A&E doctor asks for your opinion, suspecting an additional
injury. What is the most common associated fracture with this injury? [B1 Q15]

A. Olecranon fracture.
B. Coronoid process fracture.
C. Scaphoid fracture.
D. Proximal ulna fracture.
E. Capitellum fracture.

A

Coronoid process fracture.

Radial head fractures are common, accounting for approximately one-third of all elbow
fractures and up to 5% of all fractures in adults. A recent retrospective study found that
associated fracture of the upper extremity was seen in 10.2% of patients, with fractures of the
coronoid process the most common (4.1%). Radial head fracture, coronoid fracture, and medial
collateral ligament tear form the ‘terrible triad’ of the elbow, which requires operative fixation.

316
Q

A 34-year-old female presents to the A&E department after falling on an outstretched hand.
Examination reveals tenderness at the anatomic snuff box. A scaphoid radiograph series
confirms scaphoid fracture. Which of the following features is most associated with a poor
prognosis? [B1 Q7]

A. Fracture of the distal third.
B. Fracture of the middle third.
C. Fracture of the proximal third.
D. Horizontal oblique fracture orientation.
E. Displacement of the scaphoid fat stripe.

A

Fracture of the proximal third.

Scaphoid fracture is the most common of all carpal bone fractures and potentially serious due
to the high rate of avascular necrosis. This fracture can be difficult to detect on initial
radiographs. Wrist casting and repeat radiography after 1 week are typically advised if there is
ongoing suspicion. Fractures of the proximal third account for 20% of injuries but are
associated with failure to unite in 90%. Middle third fractures make up the majority (70%),
with up to 30% failing to reunite. Distal third fractures usually reunite. A vertical oblique
fracture is more unstable than a horizontal oblique fracture. Fracture displacement of greater
than 1mm is also a poor prognostic feature.

317
Q

A 24-year-old male presents to the A&E department with pain and swelling of his right thumb
after landing against his ski pole while practising at the local dry ski-slope. An avulsion fracture
at the base of the proximal phalanx is noted on a radiograph of the thumb. What underlying
soft tissue structure has been injured to result in this fracture? [B1 Q10]

A. Ulnar collateral ligament.
B. Radial collateral ligament.
C. Joint capsule.
D. Flexor pollicis longus tendon.
E. Extensor pollicis longus tendon

A

Ulnar collateral ligament.

The history and radiographic findings are typical of gamekeeper’s thumb, which is an injury to
the ulnar collateral ligament at its insertion site into the proximal phalanx of the thumb. This
injury usually requires internal fixation to secure the ligament. Radial collateral ligament
injuries of the thumb lead to painful deformity and articular degeneration. Rupture of flexor
pollicis longus results in loss of active flexion of the thumb. The thumb remains in flexion with
rupture of extensor pollicis longus. Thumb tendon injuries are typically seen in RA due to their
susceptibility to synovitis.

318
Q

A 26-year-old man presents to the A&E department with wrist pain and swelling after falling
from a ladder on an outstretched hand. The lateral radiograph demonstrates posterior
dislocation of the capitate relative to the lunate. What is the most associated fracture with this
injury? [B1 Q35]

A. Capitate.
B. Lunate.
C. Triquetral.
D. Scaphoid.
E. Radius.

A

Scaphoid.

The findings describe peri-lunate dislocation, which is the most common carpal dislocation. It
can occur without fracture (lesser arc injury) or with fracture (greater arc injury). Greater arc
injuries are twice as frequent as lesser arc injuries. When describing these injuries, the fracture
is named first with the prefix ‘trans’ followed by the dislocation. Trans-scaphoid peri-lunate
dislocation is the most common type of peri-lunate injury. Fractures of the trapezium, capitate,
hamate, and triquetrum are also part of the greater arc injuries. Other radiographic signs of this
injury include disruption of the carpal (Gilula) arcs and a triangular lunate on the AP view. An
early sign is widening of the scaphoid–lunate space (Terry-Thomas sign), which suggests
scapholunate dissociation. Lunate dislocation is the final stage of peri-lunate injuries and is
associated with the highest degree of instability.

319
Q

A 70-year-old man undergoes an x-ray of his right-hand following trauma. There is no evidence
of fracture, but incidental resorption of the middle portion of the distal phalanges is
demonstrated. Which of the following would be the most likely underlying cause? [B1 Q44]

A. Scleroderma.
B. Frostbite.
C. Leprosy.
D. Polyvinyl chloride.
E. Psoriatic arthropathy.

A

Polyvinyl chloride.

This results in resorption of the middle portion of the terminal phalanx. The other answers
cause resorption of the terminal tufts of the distal phalanges. Other causes of resorption of the
terminal tuft include Raynaud’s, diabetes, syringomyelia, burns, trauma, epidermolysis bullosa,
congenital phenytoin toxicity (in infants of epileptic mothers), and snake and scorpion venom.
Hyperparathyroidism can cause tuft, mid-portion, and periarticular resorption; psoriatic
arthropathy can cause tuft and periarticular resorption.

320
Q

A 34-year-old woman has chronic right wrist pain, with no documented history of previous
trauma. An x-ray of the right wrist shows sclerosis and irregularity of the scaphoid with early
bony fragmentation. What is the most likely eponymous disease that has resulted in this
abnormality? [B1 Q 48]

A. Sever disease.
B. Freiberg disease.
C. Kohler disease.
D. Iselin disease.
E. Preiser disease.

A

Preiser disease.

The x-ray appearances are typical for osteonecrosis within the scaphoid. This is usually
posttraumatic in aetiology, but when idiopathic it is known as Preiser disease. Postulated
mechanisms for the osteonecrosis are repetitive minor trauma or secondary to drug treatment
(e.g. steroids).

The remaining wrong answers refer to osteonecrosis affecting the foot. Freiberg disease affects
the head of the second metatarsal, Kohler disease the tarsal navicular, Iselin disease the base
of the fifth metatarsal and Sever disease the calcaneal apophysis.

321
Q

A 45-year-old woman falls onto her outstretched hand. The following findings on PA and
lateral wrist plain films indicate which pathology? A scapholunate angle of 70, a capito-lunate
angle of less than 20, and a 4mm gap between scaphoid and lunate on PA view. [B2 Q8]

a. Normal appearances
b. Scapholunate dissociation
c. Volar intercalated segment instability (VISI)
d. Dorsal intercalated segment instability (DISI)
e. Peri-lunate dislocation

A

Scapholunate dissociation

In scapholunate dissociation the scapholunate angle is >60 and there is a >3 mm gap between
the scaphoid and lunate on AP view of the wrist. In VISI, capitolunate angle is increased and
there is volar angulation of the lunate. In DISI, both scapholunate and capitolunate angles are
increased and there is dorsal angulation of the lunate.

322
Q

A young man presents to A&E following a fall onto his outstretched right arm. Plain films of
the right forearm show a fracture of the distal forearm with volar angulation of the distal
fragment with no intra-articular component. The carpal bones remain well aligned. Which of
the following injuries has he sustained? [B2 Q9]

a. Smith’s fracture
b. Barton’s fracture
c. Monteggia fracture
d. Galeazzi fracture
e. Colles fracture

A

Smith’s fracture

This description is of a Smith’s fracture. More common is a Colles fracture, which is a fracture
of the distal radius with dorsal angulation of the distal fragment. A Monteggia fracture is
fracture of the ulnar with dislocation of the radial head. A Galeazzi fracture is a fracture of the
radius with dislocation of the distal ulnar. Barton’s fracture is a fracture of the distal radius
with dislocation of the distal radiocarpal joint.

323
Q

A 10-year-old boy presents after falling downstairs and sustaining injury to his left forearm.
Radiographs show a displaced fracture of the proximal shaft ulna and anterior dislocation of
the radial head. What is the diagnosis? [B5 Q26]

(a) Galeazzi fracture dislocation
(b) Monteggia fracture dislocation
(c) Essex–Lopresti fracture complex
(d) Weber fracture
(e) Maisonneuve fracture

A

Monteggia fracture dislocation

This involves a fracture of the proximal ulna shaft with dislocated radial head

324
Q

Following a traumatic left elbow fracture, a young man complains of paraesthesia in his left
ring and little fingers. He also starts to notice weakness of his left hand. A diagnosis of ulnar
nerve entrapment is made. Which of the following muscles will not be affected? [B2 Q20]

a. Abductor digiti minimi
b. Abductor pollicis brevis
c. Adductor pollicis
d. Flexor carpi ulnaris
e. Flexor digiti minimi

A

Abductor pollicis brevis

Abductor pollicis brevis is supplied by the median nerve and would therefore not be affected
in an ulnar nerve injury. Due to the anatomic location of the ulnar nerve at the elbow, it can
often be damaged leading to denervation and paralysis of the muscles supplied by the nerve.
This includes the intrinsic muscles of the hand, which can be very debilitating. Injury to the
ulnar nerve at the wrist would lead to severe muscle denervation sparing only the opponens
pollicis, the superficial head of the flexor pollicis brevis and the lateral two lumbricals.

325
Q

A 24-year-old woman presents with a painless mass on the dorsal aspect of the right index
finger measuring approximately 11cm. MRI shows a lobulated lesion which has low signal
intensity on both T1- and T2-weighted imaging. Which of the following is the most likely
diagnosis? [B2 Q44]

a. Haemangioma
b. Lipoma
c. Ganglion cyst
d. Giant cell tumour of the tendon sheath
e. Neurilemmoma

A

Giant cell tumour of the tendon sheath

This is a benign lesion thought to represent an extra-articular form of pigmented villonodular
hyperplasia. This is low signal on both T1- and T2-weighted imaging due to haemosiderin
deposition. It most commonly affects the fingers and characteristically lies along a tendon
sheath.

326
Q

A 27-year-old man falls onto his right hand during a game of rugby. He attends the A&E
department, and a plain film of the right hand shows a comminuted fracture through the base
of the thumb metacarpal with an intra-articular component. This is the description of which of
the following fractures? [B2 Q60]

a. Rolando’s fracture
b. Bennett’s fracture
c. Gamekeeper’s thumb
d. Boxer’s fracture
e. Barton’s fracture

A

Rolando’s fracture

This is the classic description of a Rolando’s fracture. A Bennett’s fracture is also a fracture of
the base of the first metacarpal but with no comminution; this fracture is often less stable than
a Rolando’s fracture and more often requires surgical fixation. A ‘gamekeeper’s thumb’ often
occurs as the result of forced abduction of the thumb and results in disruption of the ulnar
collateral ligament.

327
Q

In the spectrum of peri-lunate ligamentous injuries and instability, volar tilt of the lunate, seen
as a triangular or ‘pie-shaped’ lunate on the AP projection of the wrist, is most commonly a
feature of which of the following? [B4 Q14]

a. scapholunate dissociation
b. peri-lunate dislocation
c. lunate dislocation
d. volar intercalated segmental instability
e. dorsal intercalated segmental instability

A

Lunate dislocation

The lesser arc refers to the arc of ligamentous attachments around the lunate. These ligaments
become disrupted in a stepwise four-stage fashion. Stage I injury is to the scapholunate
ligament, leading to dissociation with rotary subluxation of the scaphoid. Stage II is
radiographically characterized by peri-lunate dislocation, caused by additional injury to the
capito-lunate joint. The carpus migrates dorsally, and the lunate maintains a normal
relationship with the radius. Stage III involves the triquetro-lunate ligaments, and stage IV is
complete disruption of the peri-lunate ligaments, allowing dislocation and rotation of the lunate.
It is this rotation that creates the triangular outline on AP radiographs. Segmental instabilities
relate to the spectrum of dynamic scaphoid instability.

328
Q

A 23-year-old man falls onto his outstretched right hand with his elbow flexed. AP and lateral
radiographs of the mid-forearm reveal a fracture of the middle third of the radius. Which
additional radiograph should be performed? [B4 Q33]

a. clavicle
b. shoulder
c. elbow
d. oblique forearm
e. wrist

A

Wrist

A Galeazzi fracture–dislocation is a pattern of injury sustained by falling on an outstretched
hand with a flexed elbow. It most commonly consists of a fracture of the radial diaphysis with
dislocation or subluxation of the distal radioulnar joint. It is associated with a high rate of non-
union, and one or both components are usually treated with surgical fixation. It is important
therefore that the radiologist can recognize potential patterns of injury and radiographically
demonstrate their full extent. As a rule, fractures should be viewed in two orthogonal planes,
as should the joint above and below any fracture. The opposite pattern, of an ulnar shaft fracture
with dislocation of the proximal radial head, is termed a Monteggia fracture–dislocation. A
mnemonic for remembering the two is Glasgow Rangers (Galeazzi, radius) and Manchester
United (Monteggia, ulna), which indicates for each injury which of the forearm bones is
fractured.

329
Q

On plain radiographs of the hands, hyperflexion of the proximal interphalangeal joint of the
index finger, with hyperextension of the distal interphalangeal joint of the same finger,
describes which deformity? [B4 Q46]

a. swan-neck
b. Boutonniere
c. mallet finger
d. baseball finger
e. Z-deformity

A

Boutonniere

The Boutonniere deformity is commonly caused by injury or inflammatory conditions such as
rheumatoid arthritis, and more commonly affects the index than middle fingers. It consists of
four stages. Stages 1 and 2 are mild and moderate, passively correctable extension lag, whereas
stages 3 and 4 are mild and advanced flexion contractures. The proximal flexion deformity is
due to disruption of the central slip of the extensor tendon, with the proximal phalanx herniating
through the defect and the lateral slips lying on either side. The position of the proximal phalanx
stretches the lateral slips and pulls the distal phalanx into extension. Z-deformity is the name
given to a Boutonniere-type deformity seen in the thumb.

Swan-neck deformity has similar causes but the opposite configuration, with extension at the
proximal interphalangeal joint and flexion distally.

Mallet (or baseball) fingers have a passively correctable flexion deformity of the distal
interphalangeal joint caused by avulsion of the extensor digitorum tendon by a hyperflexion
injury

330
Q

Bilateral hand radiographs performed in a 70-year-old man for painful and stiff joints reveal a
symmetrical periosteal reaction involving the metacarpals, increased soft tissue of the
fingertips, and an increase in the longitudinal curvature of the fingernails. Which additional
imaging investigation is most appropriate? [B4 Q56]

a. CT of the hands
b. MRI of the hands
c. isotope bone scan
d. radiograph of the chest
e. radiographs of the shoulders

A

Radiograph of the chest

Hypertrophic pulmonary osteoarthropathy (HPOA) is a clinical syndrome of osteitis of the long
bones, arthritis, and digital clubbing of the fingers and toes. It is most associated with lung
cancer (affecting 3–10% of patients) or other chronic pulmonary or pleural disease. The
underlying mechanism has not been established with certainty, but autonomic nervous or
endocrine stimulation by tumours is postulated, with hormones such as oestrogen,
adrenocorticotrophic hormone and growth hormone implicated. In patients presenting with
HPOA, approximately 80% have an underlying lung cancer, 10% a pleural tumour and 5%
another intrathoracic malignancy. Other causes include chronic, suppurative, pulmonary
inflammatory disease and congenital cyanotic heart disease. Typical radiographic appearances
are of a lamellar periosteal reaction affecting the dia-metaphyseal regions of the long bones,
particularly the dorsal and medial aspects. Bone symptoms and radiographic signs frequently
regress following treatment of the underlying cause.

331
Q

A 67-year-old man with history of lung cancer and renal transplant had a bone scan. There are
multiple focal areas of increased tracer uptake in the left ribs, arranged in a linear pattern.
Increased tracer uptake is also identified along the cortices of both humerus and radius bones
bilaterally. No renal uptake is seen. The most likely diagnosis for this appearance is? [B5 Q20]

(a) Hypertrophic osteoarthropathy with rib metastases
(b) Hypertrophic osteoarthropathy with rib fractures
(c) Normal uptake in lower limbs with rib fractures
(d) Normal uptake in lower limbs with rib metastases
(e) Diffuse skeletal metastases

A

Hypertrophic osteoarthropathy (HPOA) with rib fractures

On bone scan multiple areas of uptake in a linear pattern suggests traumatic injury to ribs.
HPOA is characterised by bilateral symmetrical tracer uptake on bone scanning, involving the
diaphyseal and metaphyseal regions of long bones. Characteristically a periosteal reaction is
seen along the shafts of involved bones. This pattern of uptake is called a ‘double-stripe’ or
‘parallel-track’ sign and is characteristic of HPOA.

332
Q

Of the following eponyms associated with fractures, which relates to a fracture–dislocation?
[B4 Q57]

a. Segond
b. Jones
c. Smith
d. Barton
e. Hutchinson

A

Barton
The use of eponymous names for fractures allows quick and accurate identification and
communication of bone injuries while simultaneously alerting clinicians to the potential
complications associated with a given fracture pattern. This is also particularly useful when
describing complex radiographic appearances to someone remote from the images. The full
value of such eponyms depends on accurate use and understanding of their meaning: Barton’s
fracture–dislocation is an intra-articular fracture of the dorsal margin of the distal radius with
dorsal dislocation of the radiocarpal joint; Segond’s fracture is an avulsion fracture of the
proximal lateral tibia; Jones’ fracture is a transverse fracture of the base of the fifth metatarsal,
at the junction of the diaphysis and metaphysis; Smith’s fracture is a distal radial fracture with
ventral displacement; and Hutchinson’s fracture is a triangular fracture of the radial styloid.

333
Q

A middle-aged woman falls on an outstretched hand, which becomes immediately painful and
swollen. A lateral radiograph shows a small fracture fragment dorsal to the carpus, and the AP
radiograph appears normal. Which carpal bone is most likely to be fractured? [B4 Q62]

a. scaphoid
b. lunate
c. triquetrum
d. capitate
e. hamate

A

Triquetrum

Carpal fractures in general are much less common than fractures to the distal radius. The two
bones most injured are the scaphoid (75%) followed by the triquetrum (14%), and these provide
a greater diagnostic challenge radiographically than distal radial fractures. Triquetrum fractures
generally occur on the dorsal surface due to avulsion of the dorsal radiocarpal ligament, or
shearing forces from impaction with the ulnar styloid or hamate in hyperextension. Less
commonly, the body of the bone can fracture in a transverse pattern. A posterior chip fragment
can often be seen with dorsal surface fractures but is only visualized on the lateral view. Such
an injury may be a primary triquetrum injury (such as avulsion) or related to a peri-lunate
fracture–dislocation.

334
Q

A middle-aged man has a history of an undiagnosed wrist injury interfering with his playing
golf. He presents with clinically apparent ulnar nerve compression at the wrist. Which of the
following causes is most likely to be identified following investigation with CT and MRI? [B4
Q74]

a. non-union of hook of hamate fracture
b. non-union of scaphoid wrist fracture
c. scapholunate dissociation
d. pisiform osteoarthritis
e. triangular fibrocartilage complex tear

A

Non-union of hook of hamate fracture

Fractures of the hook of the hamate are the most frequent type of hamate fracture, and most
often occur from the repetitive stress of swinging a bat, club, or racket, or from the direct blow
of a club on the ground. This may result in ulnar nerve compression at the wrist in Guyon’s
canal, which is particularly exacerbated in the context of non-union due to secondary
osteoarthritis or loose bodies in the piso-triquetral joint. Other causes of ulnar nerve compression at the wrist include adjacent masses, anomalous muscles, and tendons, fibrous
palmar arch, ulnar artery aneurysm, primary osteoarthritis of the pisotriquetral joint, os hamuli
proprium and dislocation of the pisiform bone.

335
Q

A 45-year-old, right-handed, male mechanic presents to orthopaedic clinic with intermittent
ulnar-sided wrist pain that is at its worst while he uses a screwdriver. Radiographs show
positive ulnar variance with a normal ulnar styloid. Subsequent MRI reveals a central
perforation of the triangular fibrocartilage complex with chondro-malacic changes in the lunate.
What is the most likely condition? [B4 Q86]

a. ulnar impingement syndrome
b. ulnar impaction syndrome
c. ulnar styloid impaction syndrome
d. hamato-lunate impaction syndrome
e. triangular fibrocartilage tear

A

Ulnar impaction syndrome

Ulnar-sided wrist pain is often caused by one of the spectrums of conditions known as
impaction syndromes. These include ulnar impaction syndrome (most common), ulnar
impingement syndrome, ulnocarpal impaction syndrome secondary to non-union of the ulnar
styloid process, ulnar styloid impaction syndrome and hamato-lunate impingement syndrome.
Ulnar impaction syndrome is a degenerative condition secondary to excessive loading across
the wrist and characteristically shows a positive ulnar variance that is accentuated in pronation
and during a firm grip. MRI is used to identify complications such as triangular fibrocartilage
complex tear or bone marrow oedema.

336
Q

A 40-year-old man presents with a lump in the right groin 2 months after a laparoscopic
inguinal hernia repair. Ultrasound shows a well-defined homogenous, hyperechoic, avascular
soft tissue mass lateral to the inferior epigastric vessels in the right groin. It has no change on
pressure or with Valsalva manoeuvre. What is the most likely diagnosis? [B5 Q40]

(a) Recurrent direct inguinal hernia
(b) Recurrent indirect inguinal hernia
(c) Lymph node
(d) Lipoma
(e) Seroma

A

Lipoma

The lesion has typical sonographic characteristics of a lipoma.

337
Q
A
338
Q

A 35-year-old woman presents with swelling in the thigh. The radiograph shows a bony
excrescence from the femoral cortex without medullary continuity. On MRI there is a soft
tissue surrounding the bony excrescence, which returns high signal on T1 and T2 and
homogenous low signal on STIR. The most likely diagnosis is? [B5 Q31]

(a) Osteochondroma
(b) Osteosarcoma
(c) Liposarcoma
(d) Parosteal lipoma
(e) Intramuscular lipoma

A

Parosteal lipoma

These are benign tumours of adipose tissue which are intimately related to the periosteum.
They often contain bony excrescences that may resemble osteochondroma but, unlike
osteochondroma, they do not communicate with the medullary cavity of parent bone. MRI is
diagnostic, confirming the juxtacortical benign nature of the fatty lesion and non-
communication of the bony lesion with the medulla of the adjacent bone.